Монгол Бодлогын Сан

Эх хэлээрээ суралцаж, эх хэлээрээ мэдлэгээ түгээе.

Алгебрийн тэнцэтгэл биш

Адил чанартай тэнцэтгэл бишүүд
Илтгэгч тэнцэтгэл биш
Интервалын арга
Иррационал тэнцэтгэл биш
Квадрат тэнцэтгэл биш
Логарифм тэнцэтгэл биш
Модультай тэнцэтгэл биш
Модультай хялбар шугаман тэнцэтгэл биш
Параметртэй тэнцэтгэл биш
Рационал тэнцэтгэл биш
Систем тэнцэтгэл биш
Тригонометрийн тэнцэтгэл биш
Тэнцэтгэл биш батлах
Тэнцэтгэл биш бодох графикийн арга
Тэнцэтгэл бишийн шийдийн муж
Холимог тэнцэтгэл биш
Шугаман тэнцэтгэл биш, түүний систем

Адил чанартай тэнцэтгэл бишүүд

$x< y< z$ нь бодит тоонууд бол доорх илэрхийллүүдийн аль нь үнэн байх албагүй вэ?

A. $x+yz=zy+x$     B. $x-y< z-y$     C. $x+y< y+z$     D. $xy< yz$     E. $-z< - y< -x$    
$x< y< z$ нь бодит тоонууд бол доорх илэрхийллүүдийн аль нь үнэн байх албагүй вэ?

A. $x+yz=zy+x$     B. $x-y< z-y$     C. $x^2+y^2< y^2+z^2$     D. $x+y< y+z$     E. $- y< -x$    

Илтгэгч тэнцэтгэл биш

$\dfrac{3^{x} - 25}{x + 1} \le \dfrac{3^{x} - 25}{x - 3}$ тэнцэтгэл бишийг бод.
$5 \cdot 9^{x} - 18 \cdot 15^{x} + 9 \cdot 25^{x} > 0$
$3 \cdot 49^{x} - 16 \cdot 21^{x} + 21 \cdot 9^{x} < 0$
$9 \cdot 4^{ - {\frac{{1}}{{x}}}} + 5 \cdot 6^{ - {\frac{{1}}{{x}}}} < 4 \cdot 9^{ - {\frac{{1}}{{x}}}}$ бүхэл шийдүүдийг ол.
$6^{x} \ge 3\sqrt {3} \cdot 2x + 32 \cdot 2^{2} \cdot 3^{x} - 64\sqrt {108} $ хамгийн бага эерэг бүхэл шийдийг ол.
$15^{x} - \sqrt {243} \cdot 5^{x} - 25\sqrt {5} \cdot 3^{x} + 225\sqrt {15} \le 0$
$8^{{\frac{{2}}{{x}}} + {\frac{{2}}{{3}}}} + 2^{{\frac{{2}}{{x}}} + 1} \ge 3^{2} \cdot 4^{{\frac{{2}}{{x}}}}$ мужийн уртыг ол.
$\left( {\sqrt {5} + 2} \right)^{x - 1} \ge \left( {\sqrt {5} - 2} \right)^{{\frac{{x - 1}}{{x + 1}}}}$
$4^{x^{2} - x} - 10 \cdot 2x^{2} + 2^{2x + 4} \ge 0$
$9^{x^{2} + 2}-8\cdot 3^{x^2}-3^{2-4x}\ge 0$
$\left( {{\frac{{1}}{{5}}}} \right)^{{\left| {x - 2} \right|}} > \left( {{\frac{{1}}{{25}}}} \right)^{{\left| {x} \right|}}$
$6^{x + 2} \ge 4 \cdot 7^{{\left| {x + 1} \right|}}$
$3^{x + 1} > 7 \cdot 5^{{\left| {x - 1} \right|}}$
$25^{x + 1} \ge 10 \cdot 32^{{\left| {x - 1} \right|} + 1}$
$2^{x} + 2^{{\left| {x} \right|}} \ge 2\sqrt {2} $
${\left| {2^{{\frac{{x}}{{3}}}} - {\frac{{11}}{{2}}}} \right|} \le {\frac{{5}}{{2}}}$ бүхэл шийдүүдийн нийлбэрийг ол.
${\left| {\sqrt {3^{x}} - {\frac{{11}}{{2}}}} \right|} \le {\frac{{7}}{{2}}}$ бүхэл шийдүүдийн нийлбэрийг ол.
${\left| {4^{x - 1} - {\frac{{5}}{{4}}}} \right|} \le {\frac{{4^{x}}}{{8}}} + {\frac{{3}}{{4}}}$ бүхэл шийдүүдийн нийлбэрийг ол.
$\left( {2.5} \right)^{\left( {x + 1} \right)^{2}} \cdot \left( {0.4} \right)^{{\left| {4x - 4} \right|}} \ge \left( {{\frac{{25}}{{4}}}} \right)^{{\frac{{13}}{{2}}}}$
${\frac{{1}}{{3^{x} + 5}}} \le {\frac{{1}}{{3^{x + 1} - 1}}}$
${\frac{{2^{x + 1} - 22}}{{2^{x} - 2}}} \ge 1$
${\frac{{5}}{{2^{x + 2} - 1}}} > {\frac{{1}}{{2^{x} - 1}}}$ хамгийн бага бүхэл шийдийг ол.
${\frac{{4^{x}}}{{2^{x} - 1}}} \le {\frac{{2^{x} + 12}}{{3}}}$
${\frac{{2^{x} + 8}}{{2^{x} - 1}}} > 2^{x}$
${\frac{{2^{ - x}}}{{1 - 2^{1 - x}}}} + 2^{x} < 0$
${\frac{{9^{x} + 0.5 + 1}}{{3 - 3^{2x}}}} \le 3^{2x} + 1$
${\frac{{15 - 16^{x + 1}}}{{4^{2x} - 4}}} \ge 2^{4x + 1} - 3$
${\frac{{3 \cdot 2^{x + 2} - 27}}{{2^{x} - 1}}} \ge 2^{x} + 3$ хамгийн их бүхэл шийдийг ол.
${\frac{{9}}{{2^{{\frac{{1}}{{x}}}} + 1}}} \ge {\frac{{4}}{{2^{{\frac{{1}}{{x}}}} - {\frac{{2}}{{3}}}}}}$
${\frac{{3^{{\frac{{1}}{{x}}}} + 12}}{{3^{{\frac{{1}}{{x}}}} - 1}}} \ge {\frac{{4 \cdot 3^{{\frac{{1}}{{x}}}}}}{{3}}}$
${\frac{{1}}{{4^{\sqrt {x}} - 3 \cdot 2^{\sqrt {x}} + 2}}} < {\frac{{1}}{{6}}}$
${\frac{{2^{2 + \sqrt {x - 1}} - 24}}{{2^{1 + \sqrt {x - 1}} - 8}}} > 1$
$f\left( {g\left( {x} \right)} \right) < g\left( {f\left( {x} \right)} \right)$; энд $f\left( {x} \right) = 2^{x}$; $g\left( {x} \right) = 4^{x}$
$4^{x}\le 3\cdot 5^{\sqrt{x}+x}+4^{\sqrt{x}+1}$ тэнцэтгэл бишийн хамгийн их шийдийг ол.
$\left({x^{2}-x+1}\right)^{x^{2}-2.5x+1}< 1$ тэнцэтгэл бишийг бод.
$\left({x^{2}-x+2}\right)^{\log_{0.6}{\frac{2x-3}{x+1}}}> 1$ тэнцэтгэл бишийг бод.
$81^{x}-3\cdot 27^{x}-12\cdot 9^{x}-3^{x+2}+9>0$ тэнцэтгэл бишийг бод.
  1. $2(\log_2x)^2+3\log_24x< 8$ тэнцэтгэл бишийг бод.
  2. $x\geq 2, y\geq 2, xy=16$ бол $(\log_2x)\cdot (\log_2y)$-ийн хамгийн их ба хамгийн бага утгыг ол.
Дараах тэгшитгэл тэнцэтгэл бишийг бод.

  1. $3^x=27$
  2. $9^x-2\cdot 3^{x+1}-27=0$
  3. $\Big(\dfrac12\Big)^x< 8$
  4. $2\cdot 4^x-17\cdot 2^x+8< 0$
Тэгшитгэл, тэнцэтгэл бишийг бод.
  1. $2^x=4\sqrt{2}$
  2. $4^x>32$
  3. $(\frac12)^x\leq\frac18$
  4. $(\frac13)^x>9$
Тэнцэтгэл бишийг бод.
  1. $\big(\frac13\big)^{2x+1}\leq 3\sqrt{3}$
  2. $2^{4x}-4^{x+1}>0$
  3. $\big(\frac12\big)^{2x-1}+4<9\cdot\big(\frac12\big)^x$
$100^x-10^x+a=0$ тэгшитгэл өгөгдөв.

  1. тэгшитгэл бодит шийдтэй байх $a$-ын утгын мужийг ол.

  2. тэгшитгэл $x< -1$ бодит шийдгүй байх $a$-ын утгын мужийг ол.
$3^{(x+3)^2}+\dfrac19\le 3^{x^2-2}+27^{2x+3}$ тэнцэтгэл бишийг бод
$\dfrac{8\cdot3^{x-2}}{3^x-2^x}>1+\Big(\dfrac23\Big)^x$ тэнцэтгэл биш хэдэн бүхэл тоон шийдтэй вэ?

A. $1$     B. $4$     C. $5$     D. $2$     E. $3$    
$\dfrac{8\cdot3^x}{3^x-2^x}>9+\dfrac{2^x}{3^{x-2}}$ тэнцэтгэл биш хэдэн бүхэл шийдтэй вэ?

A. $3$     B. $4$     C. $5$     D. $2$     E. $1$    
$\log_{\frac15}(x-4)-\log_5(8-x)\ge-1$ тэнцэтгэл бишийг бод.

A. $]4; 9]$     B. $]4; 8[$     C. $[3; 4[$     D. $[3; 4[\cup]8; 9]$     E. $]8; 9]$    
$\log_{\frac{1}{2} } \left(x+3\right)-\log_{\frac{1}{4} } \left(-x\right)\ge -1$ тэнцэтгэл бишийг бод.

A. $\left[-9;-1\right[$     B. $\left[-9;-1\right]$     C. $\left]-3;-1\right]$     D. $\left[-3;-1\right[$     E. $\left]-3;0\right]$    
$3^{\frac{2+x}{x}}-28\cdot3^{\frac1x}+9\le0$ тэнцэтгэл бишийг бод.

A. $\big[-1;\frac12\big]$     B. $\big[\frac12;+\infty\big[$     C. $]-\infty;-1]$     D. $[-1;0[\cup\big]0;\frac12\big]$     E. $]-\infty;-1]\cup\big[\frac12;+\infty\big]$    
$\dfrac1{27}\le\Big(\dfrac13\Big)^{x-2}< 27$

A. $]0;4]$     B. $]-5;1]$     C. $]-1;5]$     D. $]1;5]$     E. $[1;4[$    
$\dfrac{3}{(2^x+1)(x-3)}\le 0$ тэнцэтгэл бишийн хамгийн их бүхэл шийдийг ол.

A. $3$     B. $-1$     C. $2$     D. $0$     E. $1$    
$\log_{0.25}4\le \log_{\frac14}(x-2)$ тэнцэтгэл бишийг бод.

A. $[2;2.25[$     B. $]-\infty;2]\cup[6;+\infty[$     C. $]-\infty;2]\cup[2.25;+\infty[$     D. $[2;+\infty[$     E. $]2;6]$    
$(x^2-3x+9)^{x^2-5x+6}\ge 1$ тэнцэтгэл бишийг хангах хамгийн бага 2 эерэг бүхэл тооны нийлбэр хэд вэ?

A. 0     B. 3     C. 2     D. 1     E. 6    
$2\log_{0.5}(x-2)>\log_{0.5}(x+4)$ тэнцэтгэл биш бод.

A. $0< x< 5$     B. $2< x< 5$     C. $-4< x< 5$     D. $x>5$     E. $2< x$    
$x\cdot 3^{\log_{x}4}> 12$ тэнцэтгэл бишийг бод.

A. $]1;3[\cup]4;+\infty[$     B. $]3;4[$     C. $]1;3[$     D. $]4;+\infty[$     E. $]1;2[\cup]4;+\infty[$    
$\dfrac{x^2+2x}{\log_{0.2}(x+2)}>0$ бод.

A. $]-1;0[$     B. $]-2;-1[\cup]0;\infty[$     C. $]-2;\infty[$     D. $]-2;1[$     E. $[-1;0[\cup]0;\infty[$    
$2\log_{\frac13}(x-2)>\log_{\frac13}(2x-1)$ тэнцэтгэл бишийг бод.

A. $]1;5[$     B. $]2;5[$     C. $]-\infty;1[\cup]5;+\infty[$     D. $]-\infty;2[\cup]5;+\infty[$     E. $]2;+\infty[$    
$\log_{0.5}(x+1)\ge\dfrac{1}{\log_{2x-1}\frac12}$ тэнцэтгэл бишийг бод.

A. $x>0.5$     B. $0.5< x\le 2$     C. $1< x$     D. $x\ge 2$     E. $x\le 2$    
$\log_{\frac12}(x^2+x-2)>-2$ тэнцэтгэл бишийг бод.

A. $(-3;-2)$     B. $(-3;-2)\cup(1;2)$     C. $(1; 2)$     D. $(-3;2)$     E. $(0;2)$    
$2^{x-1}>8 \cdot \sqrt{2}$ тэнцэтгэл бишийн хамгийн бага бүхэл шийдийг ол.

A. 3     B. 4     C. 5     D. 6     E. 7    
$2 \cdot 4^x-17 \cdot 2^x+8< 0$ тэнцэтгэл биш бод.

A. $(0.5;8)$     B. $(-1,3)$     C. $(0.5;3)$     D. $(3;8)$     E. $\varnothing$    
$(x^2-x+1)^{x-2}>1$ тэнцэтгэл биш бод.

A. $(2; +\infty)$     B. $(0;1)$     C. $(0;1) \cup (2; +\infty)$     D. $(0; +\infty)$     E. $(1;2)$    
$\log_8(x^2-4x+3)< 1$ тэнцэтгэл бишийг бод.

A. $(-1;1)\cup(3;5)$     B. $(-1;1)$     C. $(3;5)$     D. $(-1;5)$     E. $(1;3)$    
$\log_{\frac12}(x^2+x-2)>-2$ тэнцэтгэл бишийг бод.

A. $(-3;-2)$     B. $(-3;-2)\bigcup(1;2)$     C. $(1;2)$     D. $(-3;2)$     E. $\varnothing$    
$(x^2-x+1)^{x-2}>1$ тэнцэтгэл бишийг бод.

A. $(2;+\infty)$     B. $(0;1)$     C. $(0;1)\cup(2;+\infty)$     D. $(0;+\infty)$     E. $\varnothing$    
$(x^2-8x+13)^{x-5}< 1$ тэнцэтгэл бишийг бод.

A. $(2;+\infty)$     B. $(-\infty;4-\sqrt{3})$     C. $(2;4+\sqrt{3})$     D. $\{(-\infty;2)\bigcup(4+\sqrt{3};6)\}$    
$\log_2(x+2)>\log_{x+2}16$ тэнцэтгэл бишийг бод.

A. $(-1.75;-1)\cup(2;+\infty)$     B. $(-2;0)$     C. $(-1;0)$     D. $(2;+\infty)$     E. $(-1.75;-1)$    
$\log_2(x+3)< \log_{x+3}16$ тэнцэтгэл бишийг бод.

A. $\{(-3;-2)\bigcup(1;+\infty)\}$     B. $\{(-3;-2,75)\bigcup(-2;1)\}$     C. $\{(-3;-2)\bigcup(2;+\infty)\}$     D. $(-3;+\infty)$    
$\displaystyle \log_{0.5}(\frac2{x-3})\leq\log_{0.5}(6-x)$ тэнцэтгэл бишийг бод.

A. $3<x\leq4$     B. $3<x\leq4\bigcup5\leq x<6$     C. $5\leq x<6$     D. $3<x<6$    
$\displaystyle 5^{\frac1x}+5^{\frac1x+2}\geq650$ тэнцэтгэл бишийг бод.

A. $0<x\leq\dfrac12$     B. $\dfrac12\leq x<1$     C. $\dfrac12\leq x$     D. $1 < x$     E. $x > 1$    
$\displaystyle 3^{\frac5x}+3^{\frac5x+1}\leq36$ тэнцэтгэл бишийг бод.

A. $0<x\leq2.5$     B. $x\leq2.5$     C. $x<0\bigcup2.5\leq x$     D. $2.5\leq x$    
$\left(\dfrac13\right)^{x^2+2x}< \left(\dfrac19\right)^{16-x}$ тэнцэтгэл бишийг бод.

A. $x< -8$     B. $4< x$     C. $-8< x< 4$     D. $x< -8\cup4< x$     E. $\varnothing$    
$\displaystyle (\frac17)^{x^2+10x+14}\geq49$ тэнцэтгэл бишийг бод.

A. $x\leq-8$     B. $-2\leq x$     C. $-8\leq x\leq-2$     D. $x\leq-8\bigcup-2\leq x$    
$\displaystyle \log_{x+1}(5x^2-x)\geq2$ тэнцэтгэл бишийг бод.

A. $-\frac14\leq x<0$     B. $1\leq x$     C. $-\frac14\leq x\leq1$     D. $-\frac14\leq x<0\bigcup1\leq x$    
$\displaystyle \log_{x-1}(3x^2-x)\leq2$ тэнцэтгэл бишийг бод.

A. $-1\leq x<0\bigcup\frac12\leq x<1$     B. $1<x<2$     C. $\frac12\leq x<1$     D. $-1\leq x<0$    
$\displaystyle \log_{\frac12}\left(\frac{2x-1}{16-x^2}\right)\geq2$ тэнцэтгэл бишийг бод.

A. $x\leq-10$     B. $-10\leq x<\frac12$     C. $-10\leq x\leq2$     D. $x\leq-10\bigcup\frac12<x\leq2$    
$\displaystyle \log_{\frac13}\left(\frac{x-3}{25-x^2}\right)\geq2$ тэнцэтгэл бишийг бод.

A. $x\leq -13$     B. $3<x\leq 4$     C. $3<x<13$     D. $x\leq -13\bigcup3<x\leq 4$    
$\displaystyle\log_3x+\log_{\sqrt3}x+\log_{\frac13}x< 6$ тэнцэтгэл бишийг бод.

A. $x<0$     B. $0<x<27$     C. $27<x$     D. $0<x$    
$\displaystyle \log_2x+\log_4x+\log_{\frac12}x< 2$ тэнцэтгэл бишийг бод.

A. $x<0$     B. $x<16$     C. $0<x<16$     D. $16<x$    
$\displaystyle \frac1{\lg x}-\frac1{\lg x-1}>1$ тэнцэтгэл бишийг бод.

A. $x<1$     B. $x<10$     C. $1<x<10$     D. $10<x$    
$\displaystyle \frac1{1+\lg x}+\frac1{1-\lg x}>2$ тэнцэтгэл бишийг бод.

A. $0.1< x <1$     B. $1< x <10$     C. $0.1< x <10$     D. $0.1< x <1\cup 1 < x <10$     E. $\varnothing$    
$\displaystyle |3-\log_2x|< 2$ тэнцэтгэл бишийг бод.

A. $0<x<2$     B. $2<x<32$     C. $0<x<32$     D. $x>32$    
$|\log_3x-1|< 2$ тэнцэтгэл бишийг бод.

A. $0< x<\frac13$     B. $\frac13< x$     C. $\frac13< x<27$     D. $0< x<27$     E. $3< x<27$    
$\displaystyle \log_{0.5}^2x+\log_{0.5}x-2\leq0$ тэнцэтгэл бишийг бод.

A. $0< x\leq\dfrac12$     B. $\dfrac12\leq x\leq4$     C. $0\leq x\leq4$     D. $4\leq x$     E. $2\leq x\leq4$    
$\displaystyle \log_{\frac13}^2x+\log_{\frac13}x-6\leq0$ тэнцэтгэл бишийг бод.

A. $\frac19\leq x$     B. $x\leq27$     C. $\frac19\leq x\leq27$     D. $0\leq x\leq27$    
$\displaystyle f(x)=8^x-3\cdot4^x$ бол $\log_2f(x)\geq x+2$ тэнцэтгэл бишийг бод.

A. $x\leq 2$     B. $2\leq x$     C. $0<x\leq2$     D. $1\leq x$    
$\displaystyle f(x)=\log_2^2x+12\log_4\sqrt x-4$ бол $f(x)\leq24$ тэнцэтгэл бишийг бод.

A. $0<x\leq\frac1{128}$     B. $\frac1{128}\leq x\leq16$     C. $16\leq x$     D. $0< x\leq16$    
$\displaystyle \log_{0.5}\log_8\frac{x^2+8x}{x-3}< 0$ тэнцэтгэл бишийг бод.

A. $x<3$     B. $0<x<3$     C. $-8<x<3$     D. $3<x$    
$\displaystyle \log_{0.3}\log_6\frac{x^2+x}{x+4}< 0$ тэнцэтгэл бишийг бод.

A. $-4<x<-3$     B. $8<x$     C. $-4<x<-3\bigcup8<x$     D. $-3<x<8$    
$\displaystyle |5-\log_2(x-3)|< 1$ тэнцэтгэл биш хангах бүхэл тооны нийлбэрийг ол.

A. 2019     B. 2020     C. 2021     D. 2022    
$\displaystyle |\log_3(x+2)-2|\leq1$ тэнцэтгэл биш хангах бүхэл тооны нийлбэрийг ол.

A. 324     B. 325     C. 326     D. 327    
$5^x\le125$ тэнцэтгэл бишийг бод.

A. $]-\infty;3]$     B. $[3;5[$     C. $]-\infty;5[$     D. $]3;+\infty[$     E. $]-\infty;+\infty[$    
$2^x\le 16$ тэнцэтгэл бишийг бод.

A. $]-\infty;4]$     B. $[4;+\infty[$     C. $]-\infty;4[$     D. $]4;+\infty[$     E. $]-\infty;+\infty[$    
$3^{x^2-x}>27^{|x-1|}$ тэнцэтгэл бишийг бод.

A. $x<-3$     B. $-3< x<3$     C. $|x|>3$     D. $1\le x$     E. шийдгүй    
$3^x\geq27$ тэнцэтгэл бишийг бод.

A. $]-3;3[$     B. $]-\infty;3]$     C. $]3;+\infty[$     D. $[3;+\infty[$     E. $[3;5]$    
$2^x\leq 8$ тэнцэтгэл бишийг бод.

A. $]-\infty;3]$     B. $[3;+\infty[$     C. $]-\infty;3[$     D. $]3;+\infty[$     E. $]-\infty;+\infty[$    
$0.2\le\Big(\dfrac15\Big)^{x-4}<125$ тэнцэтгэл бишийг бод.

A. $]-5;1]$     B. $]-5;-1]$     C. $]-1;5]$     D. $]1;5]$     E. $[-5;-1[$    
$\dfrac{5}{(5^x+2)(4x-17)}\leq 0$ тэнцэтгэл бишийн хамгийн их бүхэл шийдийг ол.

A. $5$     B. $3$     C. $1$     D. $0$     E. $4$    
$\log_{0.5}5\le \log_{\frac12}(2x-1)$ тэнцэтгэл бишийг бод.

A. $[0.5;+\infty[$     B. $]-\infty;0.5]\cup[3;+\infty[$     C. $[3;+\infty[$     D. $]0.5;3]$     E. $]-\infty;0.5]$    
$\dfrac{1}{\log_2x}-\dfrac{1}{\log_2x-1}>1$ тэнцэтгэл бишийг бод.

A. $\frac12< x<1$     B. $\frac12< x<2$     C. $\frac14< x<\frac12$     D. $1< x<2$     E. $\varnothing$    
$\log_{\frac13}(x-1)-\log_{\frac19}(9-4x)>1$ тэнцэтгэл бишийг бод.

A. $\big]0;\frac94\big[$     B. $\big]1;\frac94\big[$     C. $]0;1[$     D. $\big]0;\frac{14}{9}\big[$     E. $\big]1;\frac{14}{9}\big[$    
$\log_x4\ge1$ тэнцэтгэл бишийн шийдийн олонлог аль нь вэ?

A. $]0;1[$     B. $]1;4]$     C. $]0;4]$     D. $]1;4[$     E. $]4;+\infty[$    
$\log_2(x+3)< \log_{x+3}16$ тэнцэтгэл бишийг бод.

A. $(-3;-2)\cup(1;+\infty)$     B. $(-3;-2.75)\cup(-2;1)$     C. $(-3;-2)\cup(2;+\infty)$     D. $(-3;+\infty)$     E. $(-3;1)$    
$\dfrac{3}{\sqrt{27^x}}>\dfrac{3}{9^x}$ бод.

A. $]-\infty;0[$     B. $]4;+\infty[$     C. $]-\infty;4[$     D. $]0;+\infty[$     E. $]0;4[$    
$\dfrac{3}{\sqrt{27^x}}>\dfrac{3}{9^x}$ бод.

A. $(-\infty;0)$     B. $(4;+\infty)$     C. $(-\infty;4)$     D. $(0;+\infty)$     E. $(0;4)$    
$\left(\dfrac15\right)^{-x^2+x+9}>125$ тэнцэтгэл бишийн шийдийг ол.

A. $(-3;4)$     B. $(-4;3)$     C. $(-\infty;-3)\cup(4;\infty)$     D. $(-\infty;-4)\cup(3;\infty)$     E. $[-3;4]$    
$\left(\dfrac17\right)^{-x^2-x+4}<49$ тэнцэтгэл бишийн шийдийг ол.

A. $(-2;3)$     B. $(-3;2)$     C. $(-\infty;-6)\cup(1;\infty)$     D. $(-\infty;-3)\cup(2;\infty)$     E. $(-1;6])$    
$\left(\dfrac13\right)^{-x^2+x+3}<27$ тэнцэтгэл бишийн шийдийг ол.

A. $(-\infty;-2)\cup(3;\infty)$     B. $(-3;2)$     C. $(-1;6)$     D. $(-2;3)$     E. $[-2;3]$    
$\left(\dfrac14\right)^{-x^2+x+3}>64$ тэнцэтгэл бишийн шийдийг ол.

A. $(-3;2)$     B. $(-2;3)$     C. $(-1;6)$     D. $(-\infty;-2)\cup(3;\infty)$     E. $(-\infty;-1)\cup(6;\infty)$    
$\left(\dfrac13\right)^{-x^2+x+3}<27$ тэнцэтгэл бишийн шийдийг ол.

A. $(-\infty;-2)\cup(3;\infty)$     B. $(-3;2)$     C. $(-1;6)$     D. $(-2;3)$     E. $[-2;3]$    
$(x^2-3x+9)^{x^2-5x+6}\le 1$ тэнцэтгэл бишийг хангах хамгийн бага 2 эерэг бүхэл тооны нийлбэр хэд вэ?

A. 0     B. 3     C. 2     D. 5     E. 6    
$4^x\le256$ тэнцэтгэл бишийг бод.

A. $]-\infty;0[$     B. $[0;4[$     C. $]-\infty;4]$     D. $]4;+\infty[$     E. $]-\infty;+\infty[$    
$\dfrac1{27}\le\Big(\dfrac13\Big)^{x-2}< 27$

A. $]0;4]$     B. $]-5;1]$     C. $]-1;5]$     D. $]1;5]$     E. $[1;4[$    
$9\le 3^n < 729$ нөхцлийг хангах $n$-ийн натурал тоон утгуудын нийлбэрийг олоорой.

A. $6$     B. $8$     C. $10$     D. $13$     E. $14$    
$\dfrac{4-7\cdot 5^x}{5^{2x+1}-12\cdot 5^x+4}\le\dfrac23$ тэнцэтгэл бишд $5^x=t$ орлуулга хийн хувиргавал $\dfrac{\fbox{ab}t^2-3t-4}{15t^2-36t+\fbox{cd}}\ge0$ хэлбэрт шилжинэ. Уг тэнцэтгэл бишээ бодвол шийдийн олонлог $t\le-\dfrac12, \dfrac{\fbox{e}}{5}< t\le\dfrac{4}{5},t>\fbox{f}$ болно. Дээрх орлуулгаа ашиглавал анхны тэнцэтгэл бишийн шийдийн олонлог $x>\log_5{\fbox{g}}, \log_5\dfrac{2}{5}< x\le\log_5\dfrac{\fbox{h}}{5}$ байна.
$\log_{\frac35}|x-4|>\log_{\frac35}7$ тэнцэтгэл биш $x\ne \fbox{a}$ байх бүх цэгүүд дээр тодорхойлогдоно. Уг тэнцэтгэл бишийг хангах бүхэл тоон шийдийн тоо $\fbox{bc}$ байна.
$\log_{\frac53}|x-3|\leq \log_{\frac53}7$ тэнцэтгэл биш $x\ne \fbox{a}$ байх бүх цэгүүд дээр тодорхойлогдоно. Уг тэнцэтгэл бишийг хангах бүхэл тоон шийдийн тоо $\fbox{bc}$ байна.
$\log_{0,1}\left(\dfrac2{x-4}\right)\leq \log_{0,1}(7-x)$ тэнцэтгэл биш $\fbox{a}< x< \fbox{b}$ мужид тодорхойлогдох ба шийдийн олонлог нь $\fbox{c}< x\leq 5, \fbox{d}\leq x< \fbox{e}$ байна.
$\log_{0,5}\left(\dfrac2{x-3}\right)\leq \log_{0,5}(6-x)$ тэнцэтгэл биш $\fbox{a}< x< \fbox{b}$ мужид тодорхойлогдох ба шийдийн олонлог нь $\fbox{c}< x\leq 4, \fbox{d}\leq x< \fbox{e}$ байна.
$|3-\log_{2}x|< 2$ тэнцэтгэл биш $x>\fbox{a}$ мужид тодорхойлогдох $\fbox{b}< x< \fbox{cd}$ шийдийн олонлогтой.
$|\log_{3}x-1|< 2$ тэнцэтгэл биш $x>\fbox{a}$ мужид тодорхойлогдох $\dfrac{\fbox{b}}{\fbox{c}}< x< \fbox{de}$ шийдийн олонлогтой.
$y=\log_{6}{\dfrac{9x+4}{5+8x}}-2 $ функц нь $]-\infty;-\dfrac{\fbox{a}}{\fbox{b}}[\cup]-\dfrac{\fbox{c}}{\fbox{d}};+\infty[$ мужид тодорхойлогдоно.
$x^2+2(1+\log_{7}{k})x+(3+\log_{7}{k})=0$ тэгшитгэл ялгаатай язгууруудтай байх $k$-ийн утга $\fbox{a}< k< \dfrac{\fbox{b}}{49}$, $k>\fbox{c}$ байна.
$2^{2x+1}-21(\frac{1}{2})^{2x+3}+2\geq0$ тэнцэтгэл бишийг $2^{2x+3}=y$- ээр үржүүлбэл $\fbox{a}(2^{2x+1})^2+\fbox{b}\cdot{2^{2x+1}}-21\geq0$ тэнцэтгэл бишд шилжих ба шийдийн олонлог нь $x\geq\dfrac{\fbox{c}}{\fbox{d}}\log_{2}{\fbox{e}}-1$ байна.
$3^{4-3x}-35(\frac{1}{3})^{2-3x}+6\geq0$ тэнцэтгэл бишийг $ 3^{3x-2}=y $-ээр үржүүлбэл $35\cdot{3^{6x-4}}-\fbox{a}\cdot{3^{3x-2}}-9\leq0$ тэнцэтгэл бишд шилжих ба шийдийн олонлог нь $ x\leq\log_{\fbox{b}}{\frac{3}{\sqrt[3]{\fbox{c}}}} $ байна.
$5^x-3^{x+1}>2(5^{x-1}-3^{x-2}) $ тэнцэтгэл бишийн шийдийн олонлог $x>\fbox{a}$ байна.
$ 7^x-2^{x+2}< 5(7^{x-1})-2^{x-1} $ тэнцэтгэл бишийн шийдийн олонлог $ x< \fbox{a} $ байна.
$ \dfrac{3}{2}\log_{4}{\sqrt[3]{x}}-\dfrac{1}{2}\log_{2}{x}>1 $ тэнцэтгэл бишийн шийдийн олонлог $ 0< x< \dfrac{\fbox{a}}{\fbox{bc}} $ байна.
$ \dfrac{5}{2}\log_{5}{\sqrt[5]{x}}-\dfrac{1}{3}\log_{\sqrt{5}}{x}>1 $ тэнцэтгэл бишийн шийдийн олонлог $0< x< {\fbox{a}}^{\fbox{b}} $ байна.
$\log_{5}{(3-8x)}>0 $ хангах $x$-ийн хамгийн их бүхэл утга $x=\fbox{a}$ байна.
$ \log_{3}{\big(\frac{1}{7}\big)^x}+\dfrac{1}{\log_{7}{3}}>0 $ бол $  x< \fbox{a} $ байна.
$2^{|x+2|}>16 $ тэнцэтгэл бишийн шийдийн олонлог $ x\in\left]-\infty;-\fbox{a}\right[\cup\left]\fbox{b};+\infty\right[ $ байна.
$ 3^{|x^2-x|}< 9 $ тэнцэтгэл бишийн шийдийн олонлог $ -\fbox{a}< {x}< \fbox{b} $ байна.
$|3^x-2|\leq1 $ тэнцэтгэл бишийн шийдийн олонлог $ \fbox{a}\leq{x}\leq{\fbox{b}} $ байна.
$ 5^{-|x+2|}< 0.2 $ тэнцэтгэл бишийн шийдийн олонлог $ x\in]-\infty;-\fbox{a}[\cup]-\fbox{b};+\infty[ $байна.
$(2^x+3\cdot2^{-x})^{2\log_{2}{x}-\log_{2}{(x+6)}}>1 $ тэнцэтгэл биш $ x>\fbox{a} $ мужид тодорхойлогдох ба шийдийн олонлог нь $ x>\fbox{b} $ байна.
$(4\cdot3^x+3^{-x})^{3\log_{3}{(x-1)}-\log_{3}{((x-1)(2x+1))}}>1 $ тэнцэтгэл биш $x>\fbox{a}$ мужид тодорхойлогдох ба шийдийн олонлог нь $ x>\fbox{b}$ байна.
$\log_{\frac{1}{2}}{(1+x-\sqrt{x^2-4})}\leq0 $ тэнцэтгэл биш $ x\leq{-\fbox{a}}, x\geq{\fbox{b}} $ мужид тодорхойлогдох ба шийдийн олонлог $ x\geq{\fbox{c}} $ байна.
$ \log_{3}{(\sqrt{x^2-9}-x+\dfrac{1}{3})}\geq{-1} $ тэнцэтгэл биш $ x\leq{-\fbox{a}}, x\geq{\fbox{b}} $ мужид тодорхойлогдох ба шийдийн олонлог нь $ x\leq{-\fbox{c}} $ байна.
$ \log_{2}{(x^2+3)}+\log_{\frac{1}{2}}{5}=2\log_{\frac{1}{4}}{(x-1)}-\log_{2}{(x+1)} $ тэгшитгэл $ x>\fbox{a} $ мужид тодорхойлогдоно. Тэгшитгэлээ хувирган потенциачилбал $(x^2+\fbox{b})(x^2-1)=\fbox{c}$ тэгшитгэлд шилжинэ. Эндээс $x=\sqrt{\fbox{d}} $ шийд олдоно.
$ \log_{9}{(x^2-5x+6)^2}=2^{-1}\log_{\sqrt{3}}{\frac{x-1}{2}}+\log_{3}{|x-3|} $ тэгшитгэлийн тодорхойлогдох муж $ x\neq3, x\neq\fbox{a}, x>\fbox{b} $ байх ба язгуур нь $\dfrac{\fbox{c}}{\fbox{d}} $ байна.
$\log_{x+1}{(x^2+x-6)^2}\geq4 $ тэнцэтгэл биш $-1< x\neq{0}, x\neq{\fbox{a}}, x\neq{-\fbox{b}} $ мужид тодорхойлогдох ба шийдийн олонлог $ 0< x\leq{\fbox{c}} $ байна.
$\log_{9x^2}{(6+2x-x^2)}\leq\dfrac{1}{2} $ тэнцэтгэл бишийн шийдийн олонлог $ \fbox{a}-\sqrt{7}< x\leq-1, \dfrac{-\fbox{b}}{3}< x< 0 , 0< x< \dfrac{1}{3}, \fbox{c}\leq{x}\leq1+\sqrt{7}$ байна.
$\log_{2}{(|x-2|-1)}< 1 $ тэнцэтгэл биш $ x>\fbox{a}, x< \fbox{b} $ мужид тодорхойлогдох ба шийдийн олонлог $x\in]-\fbox{c};\fbox{d}[\cup]\fbox{e},\fbox{f}[$ байна.
$\log_{0.5}{(3+2x-|x+1|)}>-1 $ тэнцэтгэл биш $ \dfrac{-\fbox{a}}{\fbox{b}}< x< -\fbox{c} , x\geq{-\fbox{d}} $ мужид тодорхойлогдох ба шийдийн олонлог $х\in\left]-\dfrac{\fbox{e}}{\fbox{f}};\fbox{g}\right[$ байна.
$2\log_{x-2}{\sqrt{3}}+(x-4)^2\log_{3}{(x-2)}=(x-4)^2\log_{x-2}{3}+2\log_{3}{\sqrt{x-2}} $ тэгшитгэл $ x>\fbox{a}, x\neq{\fbox{b}} $ мужид тодорхойлогдох ба $ x_1=\fbox{c}, x_2=\dfrac{\fbox{d}}{\fbox{e}} $ шийдүүдтэй байна.
$\dfrac{1-\log_2x}{1+\log_4x}\leq 2$ тэнцэл биш $x>\fbox{a}$ мужид тодорхойлогдох ба $\fbox{b}< x< \dfrac{1}{\fbox{c}},$ $x\geq \dfrac1{\sqrt{\fbox{d}}}$ шийдийн олонлогтой.
$\dfrac{1-\log_3x}{1+\log_9x}\leq 2$ тэнцэл биш $x>\fbox{a}$ мужид тодорхойлогдох ба шийдийн олонлог нь $\fbox{b}< x< \dfrac{1}{\fbox{c}},$ $x\geq \dfrac1{\sqrt{\fbox{d}}}$ байна.
$ \log_{x}{(10x+3)}\log_{10x}{(3x+10)}\geq0 $ тэнцэтгэл биш $ x>\fbox{a}, x\neq{\fbox{b}}, x\neq\dfrac{1}{10} $ мужид тодорхойлогдох ба шийдийн олонлог нь $ x>\fbox{c}, \dfrac{1}{\fbox{de}}>x>0 $ байна.
$\log_{2-x}{(x+2)}\log_{x+3}{(3-x)}\leq0 $ тэнцэтгэл биш $ x\neq{\fbox{a}}, -\fbox{b}< x< \fbox{c} $ мужид тодорхойлогдох ба шийдийн олонлог $-2< x\leq{-\fbox{d}}, \fbox{e}< x< 2$ байна.
$ \dfrac{\sqrt{x-5}}{\log_{\sqrt{2}}{(x-4)}-1}\geq0 $ тэнцэтгэл биш $ x\geq\fbox{a} , x\neq\fbox{b}+\sqrt{\fbox{c}} $ мужид тодорхойлогдох ба шийдийн олонлог нь $ x=\fbox{d}, x>\fbox{e}+\sqrt{\fbox{f}} $ байна.
$ \dfrac{(\log_{\sqrt{2}}{(x-3)})^2}{x^2-4x-5}\geq0 $ тэнцэтгэл биш $ x>\fbox{a}, x\neq{\fbox{b}}, x\neq{-1} $ мужид тодорхойлогдох ба шийдийн олонлог $x>\fbox{c}, x=\fbox{d}$ байна.
$\sqrt{2(5^x+24)}-\sqrt{5^x-7}\geq\sqrt{5^x+7} $ тэнцэтгэл биш $ x\geq\log_{\fbox{a}}{\fbox{b}} $ мужид тодорхойлогдоно. тэнцэтгэл бишээ $ \sqrt{2(5^x+24)}\geq\sqrt{5^x-7}+\sqrt{5^x+7} $ хэлбэртэй бичээд дараалан 2 удаа квадрат зэрэг дэвшүүлэн хувиргавал $ \fbox{cde}\geq5^{\fbox{f}x} $ хэлбэрт шилжинэ. Тодорхойлогдох мужаа тооцон тэнцэтгэл бишээ бодвол шийдийн олонлог $ \log_{\fbox{a}}{\fbox{b}}\leq{x}\leq\fbox{g} $ байна.
$\sqrt{13^x-5}\leq\sqrt{2(13^x+12)}-\sqrt{13^x+5} $ тэнцэтгэл биш $ x\geq\log_{13}{\fbox{a}} $ мужид тодорхойлогдоно. тэнцэтгэл бишээ $ \sqrt{13^x-5}+\sqrt{13^x+5}\leq\sqrt{2(13^x+12)} $ хэлбэртэй бичээд дараалан 2 удаа квадрат зэрэг дэвшүүлэн хувиргавал $ {\fbox{bc}}^{2x}\leq169 $ хэлбэрт шилжинэ. Тодорхойлогдох мужаа тооцон тэнцэтгэл бишээ бодвол шийдийн олонлог $\log_{13}{\fbox{d}}\leq{x}\leq\fbox{e}$ болно.
$ (\sqrt{2}+1)^{\frac{6x-6}{x+1}}\leq(\sqrt{2}-1)^{-x} $ тэнцэтгэл бишийн хувьд $ \sqrt{2}-1=\dfrac{\fbox{a}}{\sqrt{2}+1} $ учраас өгөгдсөн тэнцэтгэл биш $ \dfrac{x^2-\fbox{b}x+\fbox{c}}{x+1}\geq0-$тэнцэтгэл бишийг бодоход шилжинэ. Иймд шийдийн олонлог $ -\fbox{d}< x\leq\fbox{e} , x\geq\fbox{f} $ байна.
$(\sqrt{5}+2)^{x-1}\geq(\sqrt{5}-2)^{\frac{x-1}{x+1}} $ тэнцэтгэл бишийн хувьд $ \sqrt{5}-2=\dfrac{\fbox{a}}{\sqrt{5}+2} $ учраас дээрх тэнцэл биш $\dfrac{x^2+\fbox{b}x-\fbox{c}}{x+1}\geq0 $ тэнцэтгэл бишийг бодоход шилжинэ. Иймд шийдийн олонлог $ -\fbox{d}\leq{x}< -\fbox{e}, x\geq\fbox{f} $ байна.
$\left\{ % \begin{array}{c} \log_{2-x}{(2-y)}>0 \\ \log_{4-y}{(2x-2)}>0 \\ \end{array} % \right.$ системийн тодорхойлогдох муж $ \left\{ % \begin{array}{c} \fbox{a}< x< \fbox{b} \\ y< \fbox{c} \\ \end{array} % \right.$ ба шийдийн олонлог нь $\{ (x,y): \frac{3}{2}< x< \fbox{d}, \fbox{e}< y< \fbox{f} \} $ байна.
$\left\{ % \begin{array}{c} \log_{x-2}{(2y-4)}>0 \\ \log_{3-y}{(x-4)}>0 \\ \end{array} % \right.$ системийн тодорхойлогдох муж $ \left\{ % \begin{array}{c} x>\fbox{a} \\ \fbox{b}< y< \fbox{c} \\ \end{array} % \right.$ ба шийдийн олонлог $\{ (x,y): 4< x< \fbox{d}, \dfrac{\fbox{e}}{\fbox{f}}< y< \fbox{g} \} $ байна.
$\log_x(x+1)<\log_{\frac1x}(2-x)$ тэнцэтгэл бишийг бод. $$x\in]\fbox{a};\fbox{b}[\cup\Big]\frac{\sqrt{\fbox{c}}+1}{\fbox{d}};\fbox{e}\Big[$$
$\left\{\begin{array}{l}\log_{x+\frac{1}{12}}6>\log_{x+\frac{1}{12}}5\\ \log_{x+\frac{1}{14}}6>\log_{x+\frac{1}{14}}7\end{array}\right.$ тэнцэтгэл бишийн шийд нь $$\dfrac{\fbox{ab}}{\fbox{cd}}< x<\dfrac{\fbox{ef}}{\fbox{gh}}$$ байна.

Интервалын арга

$(x^2+1)(x^2+x+1)^3(x+1)^5>0$ тэнцэтгэл бишийн хамгийн бага бүхэл шийдийг ол.
$(x^3-1)(x^4-16)< 0$ тэнцэтгэл биш бод.
$\left( {4x - 1} \right) \cdot \log _{2} x \ge 0$хамгийн бага бүхэл шийдийг ол.
$\left( {x + 2} \right) \cdot \log _{1.5} \left( {4 - x} \right) \ge 0$
$\left( {4x^{2} - 16x + 7} \right) \cdot \log _{2} \left( {x - 3} \right) > 0$
${\frac{{\log _{0.1} \left( {x + 2} \right)}}{{\sqrt {5 - 4x - x^{2}} }}} \le 0$
${\frac{{\sqrt {2x + 1}} }{{2 + \log _{0.5} \left( {x + 1} \right)}}} \ge 0$
${\frac{{\log _{0.3} \left( {x - 1} \right)}}{{\sqrt {8 - 2x - x^{2}} }}} \le 0$
$2^{\log _{0.7} \left( {1 + 2x} \right)} > 4$ шийдийн мужийн уртыг ол.
$\left( {{\frac{{1}}{{2}}}} \right)^{\log _{3} \left( {1 - x} \right)} \ge 0.25$ бүхэл шийдүүдийн нийлбэрийг ол.
$\left( {{\frac{{1}}{{2}}}} \right)^{\log _{{\frac{{1}}{{9}}}} \left( {2x^{2} - 3x + 1} \right)} < 1$
$\left( {{\frac{{1}}{{2}}}} \right)^{\log 3\log _{{\frac{{1}}{{5}}}} \left( {x^{2} - {\frac{{4}}{{5}}}} \right)} > 1$
${\frac{{2x^{2} - 11x + 15}}{{2^{x} - 6}}} < 0$
${\frac{{x - 2\sqrt {x} - 8}}{{2^{x} - 4}}} \ge 0$
${\frac{{x^{2} \cdot \left( {x - 2} \right)^{2}}}{{\log _{0.5} \left( {x^{2} + 1} \right)}}} \ge 0$
$\left( {{\frac{{1}}{{3}}}} \right)^{\log _{5}^{2} x - \log _{5} x^{2}} > {\frac{{1}}{{81}}} \cdot 3^{2\log _{5} x - 5}$
${\frac{{3 \cdot 2^{{\frac{{x}}{{2}}}} - 7 \cdot 2^{{\frac{{x}}{{4}}}} - 20}}{{\sqrt {x - 3}} }} \le 0$ хамгийн бага бүхэл шийдийг ол.
$\sqrt {6 - x} \cdot \left( {2 \cdot 9^{2x} - 53 \cdot 3^{2x} - 27} \right) \ge 0$ хамгийн бага бүхэл шийдийг ол.
${\frac{{\sqrt {3^{2x + 1} - 4 \cdot 3^{x} + 1}} }{{x^{2} - x - 6}}} \le 0$
${\frac{{3x + \sqrt {x + 1}} }{{3x - \sqrt {x + 1}} }} \le 1$
$2\sqrt {x^{2} - x - 2} \ge {\left| {x + 1} \right|} - 2$ тэнцэтгэл бишийг бод.
$P(x)=x^3-5x^2+2x+8$ байв.
  1. $P(x)=0$ тэгшитгэл бод.
  2. $P(x)\leq 0$ тэнцэтгэл биш бод.
$(1-\sin x)(x-2)\ge 0$ тэнцэтгэл бишийн хамгийн бага эерэг шийдийг ол.

A. $\pi$     B. $\dfrac{\pi}{2}$     C. $2$     D. $3$     E. $0$    
$ (x^2-4)\cdot\ln x>0$ тэнцэтгэл бишийг бод.

A. $]-2;2[$     B. $]1;2[$     C. $]0;1[\cup]2;+\infty[$     D. $]0;1[$     E. $]2;+\infty[$    
$(x^2-4)(x+2)(x-4)\le 0$ тэнцэтгэл биш хэдэн бүхэл шийдтэй вэ?

A. 1     B. 2     C. 3     D. 4     E. 5    
$\dfrac{(x-3)(2x-3)}{(x+5)}\le 0$ тэнцэтгэл бишийг бод.

A. $]-\infty;-5]\cup\big[\frac32;3]$     B. $]-\infty;-5[\cup\big[\frac32;3\big]$     C. $\big]-5;\frac32\big]\cup[3;+\infty]$     D. $\big[-5;\frac32\big]\cup[3;+\infty]$     E. Шийдгүй    
$\dfrac{(x-2)(x+3)}{1+x}\ge 0$ тэнцэтгэл бишийг бод.

A. $[-3;-1[\cup[2;+\infty[$     B. $[2;+\infty[$     C. $]-\infty;-3]\cup]-1;2]$     D. $[3;+\infty[$     E. $]-1;2]$    
$\dfrac{1}{5 - t} + \dfrac{2}{1 + t} < 1$ тэнцэтгэл бишийн хамгийн бага эерэг бүхэл шийдийг ол.

A. $2$     B. $3$     C. $5$     D. $6$     E. $10$    
$(x^2-9)(x+2)(x-3)\le 0$ тэнцэтгэл биш хэдэн бүхэл шийдтэй вэ?

A. 1     B. 2     C. 3     D. 4     E. 5    
$(x^2-9)(x+2)(x+3)\le 0$ тэнцэтгэл биш хэдэн бүхэл шийдтэй вэ?

A. 1     B. 2     C. 3     D. 7     E. 9    
$x^2-8x+15<0$ тэнцэтгэл бишийг бод.

A. $]-\infty;3[\cup]5;+\infty[$     B. $[3;5]$     C. $]3;5[$     D. $]5;+\infty[$     E. $]-\infty;5[$    
$\dfrac{(x-2)(x+3)}{1+x}\ge 0$ тэнцэтгэл бишийг бод.

A. $[-3;-1[\cup[2;+\infty[$     B. $[2;+\infty[$     C. $]-\infty;-3]\cup]-1;2]$     D. $[3;+\infty[$     E. $]-1;2]$    
$ (x^2-4)\cdot\ln x>0$ тэнцэтгэл бишийг бод.

A. $]-2;2[$     B. $]1;2[$     C. $]0;1[\cup]2;+\infty[$     D. $]0;1[$     E. $]2;+\infty[$    
$(1-\sin x)(x-2)\ge 0$ тэнцэтгэл бишийн хамгийн бага эерэг шийдийг ол.

A. $\pi$     B. $\dfrac{\pi}{2}$     C. $2$     D. $3$     E. $0$    
$ (x^2-4)\cdot\ln x>0$ тэнцэтгэл бишийг бод.

A. $]-2;2[$     B. $]1;2[$     C. $]0;1[\cup]2;+\infty[$     D. $]0;1[$     E. $]2;+\infty[$    
$(1-\sin x)(x-2)\ge 0$ тэнцэтгэл бишийн хамгийн бага эерэг шийдийг ол.

A. $\pi$     B. $\dfrac{\pi}{2}$     C. $2$     D. $3$     E. $0$    
$(x^2-4)(x+2)(x-4)\le 0$ тэнцэтгэл биш хэдэн бүхэл шийдтэй вэ?

A. 1     B. 2     C. 3     D. 4     E. 5    
$f(x)=x^3-ax^2+x+6$ олон гишүүнтийн нэг язгуур нь $x=3$ бол $a=\fbox{a}$ байна. Түүнчлэн $f(x)\ge 0$ тэнцэтгэл бишийн шийд нь $$[\fbox{bc};\fbox{d}]\cup[\fbox{e};+\infty[$$ байна.

Иррационал тэнцэтгэл биш

$\dfrac{1}{\sqrt {1 + x}} \ge \dfrac{1}{2 - x}$
$\dfrac{1}{\sqrt {2 - x}} \ge \dfrac{1}{1 + x}$
$\dfrac{1}{\sqrt {6 - 3x} + x} \ge \dfrac{1}{2}$ тэнцэтгэл бишийг бод.
$\dfrac{1}{\sqrt{6 - 2x} + x} \le \dfrac{1}{3}$ тэнцэтгэл бишийг бод.
$\dfrac{2x + \sqrt {x + 2}}{2x - \sqrt {x + 2}} \ge 1$ тэнцэтгэл бишийг бод.
${\frac{{3x + \sqrt {x + 1}} }{{3x - \sqrt {x + 1}} }} \le 1$
$x\sqrt{4 - 3x - x^2} \ge \left(\dfrac{4}{x} - 3 \right)\sqrt {(4 + x)(1 - x)} $ бүхэл шийдүүдийн үржвэрийг ол.
$(x + 3)^{2} \ge (x + 3)(1 + \sqrt{2x^2 - 4x - 5})$ хамгийн их бүхэл шийдийг ол.
$\sqrt {x^{2} - 9x + 20} \le \sqrt {x - 1} - \sqrt {x^{2} - 13}$
$\sqrt {4 - 4x^3 + x^6} > x - \sqrt[3]{2}$
$\sqrt {x - 2} + {\left| {x - 8} \right|} \le 6$
$\sqrt {x + 2} + {\left| {x - 4} \right|} \le 6$
$3\sqrt {x^{2} + {\left| {x} \right|} - 2} \ge 1 - x$
$2\sqrt {x^{2} - x - 2} \ge {\left| {x + 1} \right|} - 2$ тэнцэтгэл бишийг бод.
${\dfrac{{\sqrt{1-x^{3}}-1}}{{1+x}}}\le x$ тэнцэтгэл бишийг бод.
${\dfrac{{\sqrt{1+x^{3}}+x-2}}{{x-1}}}\ge x+1$ тэнцэтгэл бишийг бод.
$\sqrt{x+\dfrac{1}{x^{2}}}+\sqrt{x-\dfrac{1}{x^{2}}}>\dfrac{2}{x}$ тэнцэтгэл бишийг бод.
$\sqrt{x^{3}+x^{2}-2x+1}\le x$ тэнцэтгэл бишийг бод.
$\sqrt{x-\dfrac{1}{x}}-\sqrt{1-\dfrac{1}{x}}>\dfrac{x-1}{x}$ тэнцэтгэл бишийг бод.
$\sqrt{x^{3}+3x}>x^{2}-6x+3$ тэнцэтгэл бишийг бод.
$2\cdot\sqrt{x^{3}+4x}> x^{2}-8x+4$ тэнцэтгэл бишийг бод.
$\sqrt{x}+\sqrt{x+7}+2\sqrt{x\left({x+7}\right)}< 35-2x$ тэнцэтгэл бишийг бод.
$\sqrt{4v^{2}-4v-84}+\sqrt{4v^{2}-6v-85}\le{\left|{2v+1}\right|}$ тэнцэтгэл бишийг бод.
$\sqrt{9v^{2}-48v-21}+\sqrt{9v^{2}-51v-15}\le\left|{3v-6}\right|$ тэнцэтгэл бишийг бод.
$\sqrt {x - 3} \le 3 - {\left| {x - 6} \right|}$ тэнцэтгэл бишийг бод.

A. $[7;+\infty[$     B. $]-\infty;4]$     C. $[3;7]$     D. $[4;7]$     E. $\{3\}\cup[4;7]$    
$x-7<\sqrt{x+5}$ тэнцэтгэл бишийг бод.

A. $4< x< 7$     B. $-5\le x$     C. $7\le x$     D. $-5\le x< 7$     E. $-5\le x< 11$    
$2\sqrt{5+2x}<8-x$ тэнцэтгэл бишийг бод.

A. $[-2; 3]$     B. $\big]-1; \frac32\big[$     C. $\big[-1; \frac32\big]$     D. $\big[-\frac52; 2\big[$     E. $\big[-\frac52; -1\big]$    
$\sqrt{x^2-2x}>x-2$ тэнцэтгэл бишийг бод.

A. $(2; \infty)$     B. $[2;\infty)$     C. $(-\infty ; 0)$     D. $(-\infty ; 0]$     E. $(-\infty ;0] \cup (2; \infty)$    
$x\le\sqrt{3-2x}$ тэнцэтгэл бишийг бод.

A. $[-3;1]$     B. $]-\infty;-3]$     C. $]-\infty;1]$     D. $[1;+\infty[$     E. $\varnothing$    
$\sqrt{16x+96}+\sqrt{4x+24}>6x$ тэнцэтгэл бишийг бод.

A. $]-2;3[$     B. $[-6;3[$     C. $]3;+\infty[$     D. $[-6;+\infty[$     E. $]-\infty;+\infty[$    
$\sqrt{x}-3\le\dfrac{2}{\sqrt{x}-2}$ тэнцэтгэл бишийг бод.

A. $[0;1]\cup]4;16]$     B. $]4; 16]$     C. $[0;16]$     D. Аль нь ч биш     E. $[0;1]\cup[4;16]$    
$\sqrt{x+1}< 2$ тэнцэтгэл бишийн бүхэл шийдүүдийн нийлбэрийг ол.

A. 2     B. 3     C. 4     D. 5     E. 6    
$\sqrt{5-2x}< 6x-1$ тэнцэтгэл бишийн хамгийн их бүхэл шийд ол.

A. $0$     B. $1$     C. $2$     D. $3$     E. $4$    
$\sqrt{x+2}>x$ тэнцэтгэл биш бод.

A. $[-2;2[$     B. $(-1;2)$     C. $[-2;0]$     D. $[0;2[$     E. $]-\infty;2[$    
$\sqrt{x+1} \cdot (x^3-4x)>0$ тэнцэтгэл биш бод.

A. $(-1; +\infty)$     B. $(-1;0)$     C. $(-2;0) \cup (2;+\infty)$     D. $(-\infty;-2) \cup (0;2)$     E. $(-1;0) \cup (2;+\infty)$    
$\sqrt{3x-12}< 3$ тэнцэтгэл бишийг бод.

A. $0\leq x<7$     B. $4>x$     C. $0\leq x<1$     D. $4\leq x<7$     E. $\varnothing$    
$\sqrt{6-2x}>2$ тэнцэтгэл бишийг бод.

A. $x<1$     B. $x\geq 1$     C. $4\leq x<7$     D. $x\geq 7$    
$\sqrt{2x+6}< 2\sqrt{3-x}$ тэнцэтгэл бишийг бод.

A. $x<2$     B. $-3\leq x<1$     C. $2\leq x\leq 3$     D. $x>3$     E. $-3\leq x\leq 3$    
$\sqrt{3x-6}\geq \sqrt{6-x}$ тэнцэтгэл бишийг бод.

A. $x\geq 3$     B. $x\leq 6$     C. $3<x<6$     D. $3\leq x\leq 6$     E. $3< x\leq 6$    
$\sqrt{6+x-x^2}< 6$ тэнцэтгэл бишийг бод.

A. $]-\infty,-5]$     B. $[6,\infty[$     C. $[-5,6]$     D. $[-2,3]$    
$\sqrt{x^2+2x+10}\leq 5$ тэнцэтгэл бишийг бод.

A. $]-5,3]$     B. $[-\infty,\infty[$     C. $[-5,3]$     D. $]-5,3[$    
$5\sqrt[3]{2x-5}+\sqrt[6]{2x-5}< 6$ тэнцэтгэл бишийг бод.

A. $[0,3[$     B. $[2.5,3[$     C. $[2.5,3]$     D. $]2.5,3[$     E. Шийдгүй    
$7\sqrt{3x-15}-4\sqrt[4]{3x-15}-3\leq 0$ тэнцэтгэл бишийг бод.

A. $[5,\frac{16}{3}]$     B. $[5,\frac{16}{3}[$     C. Шийдгүй     D. $]5,\frac{16}{3}[$    
$\sqrt{x^2-x-12}< x$ тэнцэтгэл бишийг бод.

A. $]-12;\infty[$     B. $]-\infty;-3]\cup [4;\infty[$     C. $]-12;-3]\cup [4;\infty[$     D. $[4;\infty[$     E. $]-\infty;4]$    
$\sqrt{x^2-3x+12}< 8-x$ тэнцэтгэл бишийг бод.

A. $]-\infty,4[\cup ]8,\infty[$     B. $]4,8]$     C. $]-\infty,4[$     D. $]-\infty,8]$    
$\displaystyle\frac{\sqrt{6x^2+7x-3}-4x+5}{|2x-3|}>0$ тэнцэтгэл бишийг бод.

A. $]4,\infty[$     B. $]-\infty,-\frac{3}{2}]\cup[\frac{1}{3},\frac{3}{2}[\cup]\frac32,4[$     C. $-\infty,-\frac{3}{2}[\cup]\frac{5}{4},\infty[$     D. $]-\infty,0.7[\cup]4,\infty[$    
$\displaystyle\frac{\sqrt{2x^2-3x-5}+1-x}{|2x+3|}>0$ тэнцэтгэл бишийг бод.

A. $]-\infty,-\frac{3}{2}[\cup]-\frac32,-1]\cup]3,\infty[$     B. $]-\infty,-1.5[$     C. $]-\frac32,-1]\cup]3,\infty[$     D. $]-1.5,-1]$    
$\sqrt{x-1}+\sqrt{x-2}>\sqrt{x+3}$ тэнцэтгэл бишийг бод.

A. $]\sqrt{\frac{28}{3}},6[$     B. $[2,6]$     C. $[2,\infty[$     D. $]\sqrt{\frac{28}{3}},\infty[$    
$\sqrt{x+3}+\sqrt{x-2}>\sqrt{4x+1}$ тэнцэтгэл бишийг бод.

A. $[2,+\infty[$     B. $[0,+\infty[$     C. $[0,2]$     D. $[2,6[$     E. $]6;+\infty[$    
$\sqrt{x^2+3x-4}>-2$ тэнцэтгэл бишийг бод.

A. Шийдгүй     B. $]-\infty;-4]\cup[1;\infty[$     C. $]-\infty;\infty[$     D. $]-3;0[$     E. $[-4;1]$    
$\sqrt{16-6x-x^2}>-4$ тэнцэтгэл бишийг бод.

A. $[-8,2]$     B. Шийдгүй     C. $]-6,0[$     D. $]1,\infty[$    
$\displaystyle\frac{\sqrt{5x+11}}{x+1}< 1$ тэнцэтгэл бишийг бод.

A. $\left]-\dfrac{11}{5},-1\right[\cup ]5,\infty[$     B. $\left[-\dfrac{11}{5},-1\right[\cup ]5,\infty[$     C. $]5,\infty[$     D. $\left[-\dfrac{11}{5},-1\right[$     E. $\varnothing$    
$\displaystyle\frac{\sqrt{3x+1}}{x-3}< 1$ тэнцэтгэл бишийг бод.

A. $]-\frac{1}{3},3[\cup ]8,\infty[$     B. $[8,\infty[$     C. $[-\frac{1}{3},3[\cup ]8,\infty[$     D. $]-\frac{1}{3},3[\cup [8,\infty[$    
$\sqrt{x^2-6x+8}-\sqrt{x^2-7x+10}< 1$ тэнцэтгэл бишийг бод.

A. $]-\infty,2]\cup ]\frac{11+\sqrt{28}}{3},\infty[$     B. $]-\infty,2[\cup]\frac{11+\sqrt{28}}{2},\infty[$     C. $]-\infty,2[$     D. $]\frac{11+\sqrt{28}}{3},\infty[$    
$\sqrt{x^2-4x+3}-\sqrt{x^2-5x+4}< 1$ тэнцэтгэл бишийг бод.

A. $]-\infty,1[\cup ]\frac{11+2\sqrt 7}{3},\infty[$     B. $]-\infty,1]\cup]\frac{8+2\sqrt 7}{3},\infty[$     C. $]-\infty,1[$     D. $]\frac{8+2\sqrt 7}{3},\infty[$    
$\sqrt{x^2-x-6}>x-2$ тэнцэтгэл бишийг бод.

A. $]-\infty,-2]\cup ]\frac{10}{3},\infty[$     B. $]-\infty,-2[\cup]\frac{10}{3},\infty[$     C. $]-\infty,-2[\cup [\frac{10}{3},\infty]$     D. $]-\infty,-2[$    
$\sqrt{x^2+2x-8}>x-1$ тэнцэтгэл бишийг бод.

A. $]-\infty,-4]\cup [\frac{9}{4},\infty[$     B. $]-\infty,-4]$     C. $]-\infty,-4[\cup [\frac{9}{4},\infty]$     D. $]-\infty,-4]\cup]\frac{9}{4},\infty[$    
$\sqrt{x+3}>x+1$ тэнцэтгэл бишийг бод.

A. $[-1,0]$     B. $[-2,1[$     C. $[-3,1[$     D. $[-3,1]$     E. $[-2,1]$    
$x+2< \sqrt{x+14}$ тэнцэтгэл бишийг бод.

A. $[-10,1]$     B. $[-13,2[$     C. $[-14,2]$     D. $[-10,2[$     E. $[-14,2[$    
$\sqrt{x+\sqrt{36-x^2}}< \sqrt{x+6}$ тэнцэтгэл бишийг бод.

A. $[-3\sqrt 2;0[\cup ]0;6]$     B. $[-3\sqrt 2;6]$     C. Шийдгүй     D. $]-\infty;\infty[$     E. $]-6;6[$    
$\sqrt{2x+\sqrt{16-x^2}}< \sqrt{2x+4}$ тэнцэтгэл бишийг бод.

A. $\left[-\frac{4}{\sqrt 5};4\right]$     B. $\left[-\frac{4}{\sqrt 5};0\right[\cup ]0,4]$     C. $]-\infty;\infty[$     D. $]-2;4[$     E. Шийдгүй    
$\left.\begin{array}{l} \sqrt{4x-4}< x-3 \\ \sqrt{x-5}+\sqrt{5-x}\leq 4 \end{array}\right\}$ тэнцэтгэл бишийн системийг бод.

A. Шийдгүй     B. $1\leq x\leq 5$     C. $x\leq 5$     D. $x=5$     E. $x\ge 5$    
$\left\{\begin{array}{c} \sqrt[3]{x^2+18}\leq 3 \\ \sqrt{2x-6}+\sqrt{x+6}\geq \sqrt{6-2x} \end{array}\right.$ тэнцэтгэл бишийн системийг бод.

A. Шийдгүй     B. $-6\leq x\leq 3$     C. $x\geq 3$     D. $x=3$     E. $x\le 3$    
$\sqrt{(x-4)(x+3)}\ge\sqrt{x^2-9}$ тэнцэтгэл бишийг бодоорой.

A. $]-\infty;-3]\cup[4;\infty[$     B. $[-3;4]$     C. $]-\infty;4]$     D. $]-\infty;-3]$     E. $[-4;3]$    
$\sqrt{|x+2|-2}>\sqrt{|x+2|-2015}$ тэнцэтгэл бишийн шийд болох хамгийн их сөрөг тоо ба хамгийн бага эерэг тоог ол.

A. $-2013$ ба $2013$     B. $-2013$ ба $2017$     C. $-2017$ ба $2013$     D. $-2017$ ба $2017$     E. Ийм тоонууд оршин байхгүй    
$\sqrt{x^2-x-6}>x-1$ тэнцэтгэл биш бод.

A. $(-\infty;-2]$     B. $(7;+\infty)$     C. $[-2;7)$     D. $(-\infty;-2]\cup(7;+\infty)$     E. $\varnothing$    
$\sqrt{x+3}>x+1$ тэнцэтгэл биш бод.

A. $[-1;0]$     B. $[-2;1[$     C. $[-3;1[$     D. $[-3;1]$     E. $[-2;1]$    
$\sqrt{|1-2x|}>1-2x$ бод.

A. $]0;0.5[$     B. $\{0\}$     C. $]0;5[$     D. $]0;0.5[\cup]0.5;+\infty[$     E. $[0;+\infty[$    
$\sqrt{x^2-3x-4}>x-2$ тэнцэтгэл биш бод.

A. $(-\infty;-1]$     B. $(8;+\infty)$     C. $[-1;8)$     D. $(-\infty;-1]\cup(8;+\infty)$     E. $\varnothing$    
$\sqrt{x^2+1}>x-1$ тэнцэтгэл бишийн шийдийн олонлог аль нь вэ?

A. $]-\infty;0[$     B. $]-\infty;1]$     C. $]1;+\infty[$     D. $]-\infty;+\infty[$     E. $[1;+\infty[$    
$x+2<\sqrt{2x^2+3x-5}$ бод.

A. $\big(\frac{1+\sqrt{37}}2;+\infty)$     B. $(-\infty;-2.5)$     C. $(-\infty;-2.5]\cup\big(\frac{1+\sqrt{37}}2;+\infty)$     D. $[-2.5;-2)\cup\big(\frac{1+\sqrt{37}}2;+\infty)$     E. $(-\infty;-2.5]\cup(-2;6+\sqrt{37})$    
$(x^2-9)\sqrt{x+2}\geq 0$ тэнцэтгэл биш $[-6,6]$ завсарт нийт хэдэн ширхэг бүхэл шийдтэй вэ?

A. $1$     B. $2$     C. $3$     D. $4$     E. $5$    
$(x^2-9)\sqrt{x+2}\ge 0$ тэнцэтгэл биш $[-6;6]$ завсарт хэдэн бүхэл шийдтэй вэ?

A. $1$     B. $2$     C. $3$     D. $4$     E. $5$    
$x+1<\sqrt{2x^2-x-6}$ бод.

A. $\big(\frac{3+\sqrt{37}}2;+\infty)$     B. $(-\infty;-1.5]$     C. $(-\infty;-1.5]\cup\big(\frac{3+\sqrt{37}}2;+\infty)$     D. $[-1.5;-1)\cup\big(\frac{3+\sqrt{37}}2;+\infty)$     E. $(-\infty;-1.5)\cup(-1;7+\sqrt{37})$    
$(x^2-7x+12)\cdot\sqrt{8-x}\le0$ бод.

A. $8$     B. $[3;4]$     C. $\{8\}\cup[3;4]$     D. $[3;8]$     E. $[-\infty;8]$    
$\sqrt{x^2-2x} < x-2$ тэнцэтгэл бишийг бод.

A. Шийдгүй     B. $[2;\infty)$     C. $(-\infty ; 2)$     D. $(-\infty ; 0]$     E. $(-\infty ;0] \cup (2; \infty)$    
$\sqrt{x^2-x-12}< x$ тэнцэтгэл бишийг бод.

A. $]-12;\infty[$     B. $]-\infty;-3]\cup [4;\infty[$     C. $]-12;-3]\cup [4;\infty[$     D. $[4;\infty[$     E. $]-\infty;4]$    
$\sqrt{x+3}+\sqrt{x-2}>\sqrt{4x+1}$ бод.

A. $(6;+\infty)$     B. $[2;+\infty)$     C. $[2,6]$     D. $[0,4]$     E. $\emptyset$    
$\sqrt{x-3}>-1$ тэнцэтгэл бишийн шийдийг ол.

A. $]-\infty;3[$     B. $]-\infty;+\infty[$     C. $[4;+\infty[$     D. $[3;+\infty[$     E. $]4;+\infty[$    
$\sqrt {x - 4} \le 6 - {\left| {x - 10} \right|}$ тэнцэтгэл бишийг бод.

A. $[13;+\infty[$     B. $]-\infty;4]$     C. $[4;13]$     D. $[5;13]$     E. $\{4\}\cup[5;13]$    
  1. $\sqrt{x}>2$
  2. $\sqrt{x-8}\ge3$
  3. $\sqrt{x+1}\ge\sqrt{2}$
  4. $0.25\sqrt{x-1}>2$
  5. $\sqrt{19+2x}\ge3$
  6. $\sqrt{2x-7}>1$
  7. $\sqrt{4+2x}<1.5$
  8. $\sqrt{2-4x}\le4$
  9. $\sqrt{1-0.5x}\le0.5$
  10. $\sqrt{0.2x+1}\le0.5$
  11. $\sqrt{14-x}>2-x$
  12. $2\sqrt{x-1}\ge{x-4}$
  13. $\sqrt{24-5x}>-x$
  14. $\sqrt{9x-20}\ge{x}$
  15. $2\sqrt{4-x^2}<{x+4}$
  16. $3\sqrt{1-x^2}\le3-x$
  17. $\sqrt{x^2-3x-10}\le8-x$
  18. $\sqrt{5x-x^2-6}\le3+2x$
  19. $\sqrt{5-2x}<6x-1$
  20. $\sqrt{2x^2-3x-5}\le{x-1}$

$\dfrac{\sqrt{8+x^3}-4}{x-2}\ge x$ тэнцэтгэл бишийг хялбарчилж, $\sqrt{\fbox{a}-\fbox{b}x+x^2}>0$ тул орхиж, $\dfrac{\sqrt{\fbox{c}+x}-\sqrt{x^2-\fbox{d}x+\fbox{e}}}{x-2}\ge0$ хэлбэрт шилжүүлж шийдийг олбол $x\in[-\fbox{f};\fbox{g}]$ байна.
$\dfrac{\sqrt{x^2-3x+5}+8x-3}{4x}\ge2$ тодорхойлогдох мужийг олбол $x\in]-\infty;\fbox{a}[\cup]\fbox{b};+\infty[$ болно. Иймд иррационал тэнцэтгэл бишийн шийдийг интервалын аргаар олбол $x\in[-\fbox{c};\fbox{d}[\cup]\fbox{e};+\infty[$ болно.
$\sqrt{5-\sqrt{x+3}}-\sqrt{4-x}< 0$ тэнцэтгэл бишийн тодорхойлогдох муж нь $\fbox{ab}\le x\le\fbox{c}$ байна. Шийд нь $\fbox{de}\le x< \fbox{f}$ байна.
$f(x)=\sqrt{2x+3}$ бол
  1. $f(x)=5$ тэгшитгэлийн шийд нь $x=\fbox{ab}$,
  2. $f(x)>-1$ тэнцэтгэл бишийн шийд нь $\Bigl[-\dfrac{\fbox{c}}{\fbox{d}};+\infty\Bigr[$ завсрын бүх цэг юм.
  3. $f(x)>3$ тэнцэтгэл бишийн шийд нь $\left]\fbox{e};+\infty\right[$ завсрын бүх цэг болно.
$f(x)=\sqrt{3x+7}$ бол
  1. $f(x)=5$ тэгшитгэлийн шийд нь $x=\fbox{a}$,
  2. $f(x)>-4$ тэнцэтгэл бишийн шийд нь $\Bigl[-\dfrac{\fbox{b}}{\fbox{c}};+\infty\Bigr[$ завсрын бүх цэг юм.
  3. $f(x)< 7$ тэнцэтгэл бишийн шийд нь $\Bigl]-\dfrac{\fbox{d}}{\fbox{e}};\fbox{fg}\Bigr[$ завсрын бүх цэг болно.
$f(x)=\dfrac{2-\sqrt{x+2}}{1-\sqrt{x+2}}< 0$ тэнцэтгэл бишийг "бодъё". $f$ функцийн тодорхойлогдох муж нь $\left[\fbox{ab};\fbox{cd}\right[\cup \left]\fbox{ef};+\infty\right[,$ язгуур нь $x=\fbox{g}$ болно. Иймд дээрх тэнцэтгэл бишийн шийдийг интервалын аргаар олбол $x\in \left]\fbox{hi};\fbox{j}\right[$ гэж гарна.
$f(x)=\dfrac{\sqrt{x^2-20}+2\sqrt{x}-9}{\sqrt{x^2-20}-\sqrt{x}}$ гэе. $f(x)\leq 1$ тэнцэтгэл бишийг "бодъё". $f$ функцийн тодорхойлогдох муж нь $\Bigl[\fbox{a}\sqrt{\fbox{b}};\fbox{c}\Bigr[\cup\Bigl]\fbox{d};+\infty\Bigr[,$ $f(x)=1$ тэгшитгэлийн шийд нь $x=\fbox{e}$ болно. Иймд дээрх тэнцэтгэл бишийн шийдийг интервалын аргаар олбол $x\in \Bigl]\fbox{f};\fbox{g}\Bigr]$ гэж гарна.
$\sqrt{9x-20}< x$ тэнцэтгэл бишийг "бодъё". $x>0$ учир тэнцэтгэл биш $\left\{\begin{array}{l} x^2-\fbox{a}x+\fbox{bc}>0\\ 9x-20\geq 0\end{array}\right.$ системтэй эквивалент байна. Үүнийг бодож $\dfrac{\fbox{de}}{\fbox{f}}\leq x< \fbox{g}, x>\fbox{h}$ гэж олно.
$\sqrt{3x-x^2}< 4-x$ тэнцэтгэл бишийг "бодъё". $4-x>0$ тул тэнцэтгэл биш $\left\{\begin{array}{l} 2x^2-\fbox{ab}x+\fbox{cd}>0\\ 3x-x^2\geq 0\end{array}\right.$ системтэй эквивалент байна. Үүнийг бодож $\fbox{e}\leq x\leq \fbox{f}$ гэж олно.
$\dfrac{\sqrt{6+4x-2x^2}}{(x-1)(x+2)}\leq 0$ тэнцэтгэл биш бодъё.
  1. $6+4x-2x^2=0$ тэгшитгэлийн шийд болох $x_1=\fbox{ab}, x_2=\fbox{c}$ тоонууд тэнцэтгэл бишийг хангах нь ойлгомжтой юм.
  2. $6+4x-2x^2>0$ үед $(x-1)(x+2)< 0$ байх ёстой. Иймд $\left\{\begin{array}{l} x^2-2x-3< 0 \\ (x-1)(x+2)< 0 \\ \end{array} \right.$ системээс $x\in \left]\fbox{de},\fbox{f}\right[$ шийд гарна.
Энэ бүхнээс ерөнхий шийд нь $\left[\fbox{hi},\fbox{j}\right[\bigcup\left\{\fbox{k}\right\}$ болно.
$\dfrac{\sqrt{4+3x-x^2}}{(x-2)(x+3)}\leq 0$ тэнцэтгэл биш бодъё.

а) $4+3x-x^2=0$ тэгшитгэлийн шийд болох $x_1=\fbox{ab}$, $x_2=\fbox{c}$ тоонууд тэнцэтгэл бишийг хангах нь ойлгомжтой юм.

б) $4+3x-x^2>0$ үед $(x-2)(x+3)< 0$ байх ёстой. Иймд $\left\{ \begin{array}{l} x^2-3x-4< 0 \\ (x-2)(x+3)< 0 \end{array} \right.$ системээс $x\in \left]\fbox{de},\fbox{f}\right[$ шийд гарна. Энэ бүхнээс ерөнхий шийд нь $\left[\fbox{hi},\fbox{j}\right[\bigcup\left\{k\right\}$ болно.
$\sqrt{x^2+8x+15}-\sqrt{x^2+3x+4}\le 2$ тэнцэтгэл бишийн тодорхойлогдох муж нь $\left]-\infty,\fbox{cd}\right]\bigcup\left[\fbox{ef},+\infty\right[$ ба шийдийн олонлог нь $\left]-\infty,\fbox{ab}\right]\bigcup\left[\fbox{cd},\dfrac{\fbox{e}}{\fbox{f}}\right]$ болно.
$\sqrt{x^2+10x+21}-\sqrt{x^2+4x+5}\leq 2$ тэнцэтгэл бишийн тодорхойлогдох муж нь $\left]-\infty,\fbox{ab}\right]\bigcup\left[\fbox{cd},+\infty\right[$ ба шийдийн олонлог нь $\left]-\infty,\fbox{cd}\right]\bigcup\left[\fbox{cd},\dfrac{\fbox{e}}{\sqrt{\fbox{f}}}-\fbox{j}\right]$ болно.
$3x+4>\sqrt{9+4x(x+3)}+\sqrt{-2x^2-8x+10}$ тэнцэтгэл бишийн тодорхойлогдох муж нь $\left[\fbox{ab},\fbox{c}\right],$ шийдийн олонлог нь $\left[\dfrac{\sqrt{\fbox{de}}-\fbox{f}}{\fbox{g}},\fbox{h}\right]$ байна.
$4x-5>\sqrt{1+4x(x-1)}+\sqrt{-8x^2+40x-32}$ тэнцэтгэл бишийн тодорхойлогдох муж нь $\left[\fbox{a},\fbox{b}\right],$ шийдийн олонлог нь $\left[\dfrac{\fbox{c}+\sqrt{\fbox{de}}}{\fbox{f}};\fbox{g}\right]$ байна.
$\sqrt{x+2-2a}\geq x+2$ тэнцэтгэл бишийн шийдийн олонлог нь

а) $a\leq \fbox{a}$ үед $\left[\fbox{b}a-\fbox{c},\dfrac{\sqrt{1-8a}-\fbox{d}}{2}\right],$

б) $\fbox{a}< a\leq \dfrac{\fbox{e}}{\fbox{f}}$ үед $\left[\dfrac{-\sqrt{1-8a}-\fbox{g}}{2},\dfrac{\sqrt{1-8a}-\fbox{d}}{2}\right]$;

в) $a>\dfrac{\fbox{e}}{\fbox{f}}$ үед хоосон байна. Иймд шийдийн олонлогийн урт нь 4 байхын тулд $a=\fbox{hi}$ байна.
$\sqrt{x+3-4b}\geq x+3$ тэнцэтгэл бишийн шийдийн олонлог нь

а) $b\leq\fbox{a}$ үед $\left[\fbox{b}\cdot b-\fbox{c},\dfrac{\sqrt{1-16b}-\fbox{d}}{2}\right];$

б) $\fbox{a}\le b\leq \dfrac{\fbox{1}}{\fbox{ef}}$ үед $\left[\dfrac{-\sqrt{1-16b}-\fbox{g}}{2};\dfrac{\sqrt{1-16b}-\fbox{d}}{2}\right]$;

в) $b>\dfrac{\fbox{1}}{\fbox{ef}}$ үед хоосон байна. Иймд шийдийн олонлогийн урт нь 16 байхын тулд $b=\fbox{hi}$ байна.
$\sqrt{2-\sqrt{1-x}}-\sqrt{2-x}>0$ тэнцэтгэл бишийн
  1. Тодорхойлогдох муж нь $-\fbox{a}\le x\le \fbox{b}$
  2. Эерэг шийд нь $\dfrac{-\fbox{c}+\sqrt{\fbox{d}}}{\fbox{e}}< x\le \fbox{f}$ байна.

Квадрат тэнцэтгэл биш

$(x-2)^2+3>(x+5)^2$ тэнцэтгэл бишийн хамгийн их бүхэл шийдийг ол.
$2x^2+1\le x(x+2)$
$2\sqrt {x^{2} - x - 2} \ge {\left| {x + 1} \right|} - 2$ тэнцэтгэл бишийг бод.
$x^2-9x+14\le 0$ тэнцэтгэл бишийн хамгийн их бүхэл шийдийг ол.

A. $9$     B. $7$     C. $5$     D. $2$     E. $0$    
$x^2+8x<20$ тэнцэтгэл бишийн бүхэл шийдийн тоог ол.

A. $5$     B. $10$     C. $11$     D. $15$     E. $17$    
$x^2+2x-5\ge x+7$

A. $[3;+\infty)$     B. $(-\infty;-4[\cup]3;+\infty)$     C. $\{-4;3\}$     D. $[-4;3]$     E. $(-\infty;-4]\cup[3;+\infty)$    
$x^2-x-20>0$ тэнцэтгэл бишийг бод.

A. $x>0$     B. $x>5$     C. $x< -4$     D. $x< -4\cup x>5$     E. $-4< x< 5$    
$x^2-8x+15<0$ тэнцэтгэл бишийг бод.

A. $]-\infty;3[\cup]5;+\infty[$     B. $[3;5]$     C. $]3;5[$     D. $]5;+\infty[$     E. $]-\infty;5[$    
$x^2+7x<18$ тэнцэтгэл бишийн бүхэл тоон шийдийн тоог ол.

A. $10$     B. $11$     C. $12$     D. $13$     E. Бүхэл шийд байхгүй    

Логарифм тэнцэтгэл биш

$\dfrac{1}{1 - \lg x} < \dfrac{2\lg x - 5}{1 + \lg x}$
$\dfrac{\log _{2} x - 5}{1 - 2\log _{x} 2} \ge 2\log _{2} x$
$2 + \dfrac{\log _{2}^{2} x}{1 + \log _{2} x} > \log _{2} x$
$\dfrac{3\log_{0.5} x}{2-\log _{0.5} x}\ge 2\log_{0.5}x+1$
$\left(\log _{\frac{1}{2} x + 2}\right)\left(2 - \log _{\frac{1}{4}} x^{2} \right) \le \log _{\frac{1}{2}} \dfrac{x^3}{64}$
$4\log _{2} x + \log _{2} {\dfrac{x^{2}}{8\left(x - 1\right)}} \le 4 - \log _{2} \left(x - 1\right) - \log _{2}^{2} x$
$\log _{2} \left( {\log _{3} \left( {\log _{5} x} \right)} \right) > 0$
${\dfrac{{\log _{x} \left( {\left( {x - 2} \right)\left( {x - 3} \right)} \right)}}{{\log _{x} 2}}} < \log _{2} \left( {x + 1} \right)$
$\log _{\sqrt {11} - \sqrt {5}} \left( {x^{2} + 2x + 16 - 2\sqrt {55}} \right) < 2$
$\log _{2\sqrt {2} - \sqrt {3}} \left( {x^{2} - 4x + 14 - 4\sqrt {6}} \right) < 2$
$\log _{\sqrt {6} - \sqrt {2}} \left( {x^{2} + 4x + 11 - 4\sqrt {3} } \right) < 2$
${\frac{{1}}{{2}}}\log _{tg{\frac{{\pi} }{{12}}}} \left( {x^{2}} \right) \ge \log _{tg{\frac{{\pi} }{{12}}}} \left( {\sqrt {2x + 3}} \right)$
${\frac{{1}}{{4}}}\log _{2\sin {\frac{{\pi} }{{7}}}} \left( {x^{2}} \right) > \log _{2\sin {\frac{{\pi} }{{7}}}} \left( {\sqrt[{4}]{{3x + 4}}} \right)$
$\log _{2} {\dfrac{{\left( {\sqrt {4x + 1}} \right)^{2} + 15}}{{x^{2} + 2}}} + \log _{{\frac{{1}}{{2}}}} {\dfrac{{28}}{{x + 5}}} > 0$
${\left| {\lg x - 1} \right|} < 1$
$\sqrt {1 - \log _{{\frac{{1}}{{2}}}} x} < 2$
$4^{\sqrt {9 - x^{2}} + 1} + 2 < 9 \cdot 2^{\sqrt {9 - x^{2}}} $
$4^{\log _{2} x} + x^{2} < 8$
$\log _{2} \log _{4} x + \log _{4} \log _{2} x \le 2$ мужийн дундаж цэгийг ол.
$3\log _{\sqrt {5}} 2 + \log _{\sqrt {5}} \left( {2^{x2 - 1} - {\frac{{1}}{{8}}}} \right) < \log _{\sqrt {5}} 7$
$1 + \lg x^{6} \cdot \log _{5} x > \log _{5} x^{2} + \lg x^{3}$
${\frac{{1}}{{4}}}\log _{2} \left( {x - 2} \right) - {\frac{{1}}{{2}}} \le \log _{{\frac{{1}}{{4}}}} \sqrt {x - 5} $
$\log _{2} \left( {x + \sqrt {x} - 2} \right) < 2$
$\log _{3} \left( {2^{2x - 1} - 3 \cdot 2^{x - 1} + 1} \right) < 1$
$\log _{3} \left( {\sqrt {x + 7} - x - 1} \right) < 0$ бүхэл шийдүүдийг ол.
$\log _{0.1} \left( {x - 1 + \sqrt {7 - x}} \right) > 0$ бүхэл шийдүүдийг ол.
$\log _{2} \left( {1 + \log _{{\frac{{1}}{{9}}}} \left( {{\frac{{x}}{{3}}}} \right) - \log _{9} \left( {{\frac{{x}}{{3}}}} \right)} \right) < 1$ мужийн уртыг ол.
$\log_4(3\cdot4^{x+1}-8) < 2x+1$
$\log _{2} \log _{{\frac{{1}}{{9}}}} \left( {\left( {{\frac{{3}}{{5}}}} \right)^{x} - {\frac{{2}}{{3}}}} \right) \le - 1$
$\log _{2} \left( {2^{x} - 2} \right) < 3 - x$
$\sqrt {\log _{2} {\frac{{3 - 2x}}{{1 - x}}}} < 1$ хамгийн бага бүхэл шийдийг ол.
$\log _{0.5} \left( {6{\left| {x} \right|} - 3} \right) \le \log _{0.5} \left( {4 - x^{2}} \right)$
$\lg {\left| {{\frac{{x - 1}}{{2x + 1}}} > 0} \right|}$ бүхэл шийдүүдийг ол.
$\sqrt {\lg x} \ge 2 - \sqrt[{4}]{{\lg x}}$
$\log _{5}^{2} x + {\left| {\log _{5} x} \right|} \ge 6$
$\log _{{\frac{{1}}{{3}}}}^{2} x + {\left| {\log _{{\frac{{1}}{{3}}}} x} \right|} > 2$ хамгийн бага бүхэл шийдийг ол.
$\log _{2} \left( {2^{x} - 1} \right) \cdot \log _{{\frac{{1}}{{2}}}} \left( {2^{x + 1} - 2} \right) > - 2$ бүхэл шийдүүдийг ол.
$\log _{2} \log _{3} {\frac{{x + 1}}{{x - 1}}} < \log _{{\frac{{1}}{{2}}}} \log _{{\frac{{1}}{{3}}}} {\frac{{x - 1}}{{x + 1}}}$ хамгийн бага бүхэл шийдийг ол.
$\log _{16} {\frac{{5x + 4}}{{x - 2}}} \le \log _{{\frac{{1}}{{6}}} - x} \sqrt {{\frac{{1}}{{6}}} - x} $
$\log _{3} \left( {2x^{2} - x} \right) - 1 \le \log _{3} \left( {6x - 3} \right) - \log _{3}^{2} x$
${\rm l}{\rm o}{\rm g}_{\sqrt {{\rm 2}}} {\rm x} + {\rm l}{\rm o}{\rm g}_{{\rm 7}} {\rm x}^{{\rm 5}} < {\rm 1} + {\rm 1}{\rm 0}{\rm l}{\rm o}{\rm g}_{{\rm 7}} {\rm x} \cdot {\rm l}{\rm o}{\rm g}_{{\rm 2}} {\rm x}$
${\frac{{\log _{{\frac{{1}}{{2}}}} \left( {1 - 2x} \right)}}{{\log _{2} \left( {{\frac{{8}}{{3}}}x} \right)}}} \le - {\frac{{1}}{{2}}}$
$2\log _{3} 3x - \log _{3} x^{x} \le x$
$\log_2x^x-3\log_2\dfrac{x}{2}\geqslant x$ тэнцэтгэл биш бод.
$\log _{x - 5.65} \left( {6.65 - x} \right) > 1$ мужийн дундаж цэгийг ол.
$\log _{x} \sqrt {\left( {4x - 4} \right)} \ge 1$ хамгийн бага бүхэл шийдийг ол.
$\log _{x} (1 - 2x) < 1$
$\log _{3 - 2x} x < 2$
$\log _{x} \left( {x^{2} - 2x - 3} \right) < 0$
${\rm l}{\rm o}{\rm g}_{{\rm 2}{\rm x}} \left( {{\rm x}^{{\rm 2}}{\rm -} {\rm 5}{\rm x} + {\rm 6}} \right) < {\rm 1}$
$\log _{x + 3} \left( {x^{2} - x} \right) < 1$
$\log _{x + 1} \left( {x^{2} - x + 1} \right) > 1$
$\log _{2x + 1} 16 > 2 + \log _{2x + 1} 25$
$2\log _{5} x - \log _{x} 125 < 1$
$\log _{t} 2 \le 1$ ; $t = {\frac{{x - 2}}{{x + 1}}}$ тэнцэтгэл бишийг x-ийн хувьд бод.
$\log _{x - 2} \left( {1 - 5x^{3} + x^{5}} \right) < 0$
$\log x\left( {20x + 3x^{2} - x^{3}} \right) \ge 3$
$\log _{x^{2} - {\frac{{3}}{{2}}}x} \left( {3 - 2^{x}} \right) > 0$
$\log _{ - 5x^{2} - 6x} 6x > 0$
$\log _{{\frac{{1}}{{x}}}} \left( {{\frac{{5}}{{2}}}x - 1} \right) \ge - 2$
$\log_{2x}(x-4)\cdot\log_{x-1}(6-x)< 0$
$\log _{x + 1} \left( {2 - x} \right) \cdot \log _{3x} \left( {2x - 1} \right) \ge 0$
$\log _{x} 9 \cdot \log _{3} {\frac{{1 - 5x}}{{6x - 4}}} \ge 2$
$\log _{x} 3 \cdot \log _{9} {\frac{{5 - 12x}}{{12x - 8}}} \le {\frac{{1}}{{2}}}$
$\log _{5} x + \log _{x} \left( {{\frac{{x}}{{3}}}} \right) < {\frac{{2 + \log _{3} x \cdot \log _{5} x}}{{\log _{3} x}}}$
$\log _{\left( {x + 1} \right)^{2}} 8 + 3\log _{4} \left( {x + 1} \right) \ge 9{\frac{{1}}{{4}}}$
${\frac{{6 - \lg x^{4}}}{{3 + 2\lg x^{2}}}} < 2$
$\left( {\log _{x} 2 - 1} \right) \cdot \log _{2} 2x \le {\frac{{3}}{{2}}}$
$\log _{2x + 4} 2 + {\frac{{2\log _{0.5} \sqrt {3 - 2x}} }{{1 + \log _{2} \left( {x + 2} \right)}}} \ge \log _{2x + 4} \left( {{\frac{{2}}{{3}}}x + {\frac{{4}}{{3}}}} \right)$
$\log _{3 - x} \left( {2x - 1} \right) + \log _{2x - 1} \left( {3 - x} \right) < - 2$
$\log _{9x} 3x + \log _{3x^{2}} 9x^{2} \le {\frac{{5}}{{2}}}$
$\log _{4x} 2x - \log _{2x^{2}} 4x^{2} \ge - {\frac{{3}}{{2}}}$
$\log _{8x^{2} - 0.5} \left( {\log _{0.5} x} \right) < 0$
$\log _{x} \left( {\log _{3} \left( {9^{x} - 6} \right)} \right) \ge 1$
$\log _{x} \left( {\log _{2} \left( {4^{x} - 6} \right)} \right) \le 1$
$\log _{x} \left( {\log _{2} \left( {4^{x} - 12} \right)} \right) \le 1$
$\log _{{\frac{{x}}{{2}}}} \left( {\log _{2} \sqrt {6 - x}} \right) > 0$
$\log _{{\frac{{x}}{{30}}}} \left( {\log _{x} \sqrt {30 - x}} \right) > 0$ бүхэл шийдүүдийг ол.
$\log _{{\left| {x} \right|}} \log _{2} \left( {4^{x} - 12} \right) \le 1$
$\log _{2x} \left( {\log _{3} {\frac{{x + 1}}{{x - 1}}}} \right) < \log _{{\frac{{1}}{{2x}}}} \left( {\log _{{\frac{{1}}{{3}}}} {\frac{{x + 1}}{{x - 1}}}} \right)$
$\log _{\log _{{\frac{{1}}{{5}}}} x} \left( {\log _{{\frac{{1}}{{2}}}} x} \right) > 0$
$\log _{\log _{2} \left( {{\frac{{x}}{{2}}}} \right)} \left( {x^{2} - 10x + 22} \right) > 0$
$\log _{\log _{{\frac{{1}}{{3}}}} \left( {{\frac{{x}}{{3}}}} \right)} \left( {x^{2} - 14x + 46} \right) < 0$
${\frac{{\log _{{\frac{{1}}{{5}}}} \left( {{\frac{{1}}{{x^{15}}}}} \right) - 2}}{{\log _{125} x^{12}}}} \le 4 - {\frac{{7}}{{\log _{x} 5}}}$
$\dfrac{1}{4}\cdot x^{\frac{1}{2}\cdot\log_{2}x}\ge2^{\frac{1}{4}\cdot\log_{2}^{2}x}$ тэнцэтгэл бишийг бод.
$3^{\frac{1}{4}\cdot\log_{3}^{2}x}\le\dfrac{1}{3}\cdot x^{\frac{1}{3}\cdot\log_{3}x}$ тэнцэтгэл бишийг бод.
$x\cdot 10^{\log_{x}11}< 110$ тэнцэтгэл бишийг бод.
$x\cdot 3^{\log_{x}4}> 12$ тэнцэтгэл бишийг бод.
$x\cdot 2^{\log_{x}3}\le 6$ тэнцэтгэл бишийг бод.
$3\cdot\log_{\left({2.5+x}\right)^{2}}\left({x^{2}+12x+32}\right)\le 4\cdot\log_{\left({-2.5-x}\right)}\left({x^{2}+12x+32}\right)$ тэнцэтгэл бишийг бод.
$\log_{\left({-0.5-x}\right)}\left({x^{2}+8x+12}\right)-5\cdot\log_{\left({x+0.5}\right)}\left({x^{2}+8x+12}\right)\ge 0$ тэнцэтгэл бишийг бод.
$\log_{3}\left({1+x}\right)>\left({1-\log_{x}\left({1-x}\right)}\right)\cdot\log_{3}x$ тэнцэтгэл бишийг бод.
$\log_{\left({2-5x}\right)}3+{\dfrac{{1}}{{\log_{2}\left({2-5x}\right)}}}\ge{\dfrac{{1}}{{\log_{6} \left({6x^{2}-6x+1}\right)}}}$ тэнцэтгэл бишийг бод.
$\log_{\left({7x-4}\right)}3+{\dfrac{{1}}{{\log_{2}\left({7x-4}\right)}}}\ge{\dfrac{{1}}{{\log_{6}\left({9x^{2}2x-2}\right)}}}$ тэнцэтгэл бишийг бод.
$\log_{{\frac{{x+2}}{{x-3}}}}\left({5-x}\right)^{4}\ge-4\log_{{\frac{{x-3}}{{x+2}}}}\left({4-x}\right)$ тэнцэтгэл бишийг бод.
$\log_{{\frac{{x-5}}{{x+6}}}}\left({x-8}\right)^{6}\le-6\log_{{\frac{{x+6}}{{x-5}}}}\left({x+7}\right)$ тэнцэтгэл бишийг бод.
$\log_{x^{2}}{\dfrac{{4x-5}}{{{\left|{x-2}\right|}}}}\ge{\dfrac{{1}}{{2}}}$ тэнцэтгэл бишийн хамгийн их шийдийг ол.
$\log_{3}{\dfrac{{{\left|{x^{2}-4x}\right|}+3}}{{x^{2}+{\left|{x-5}\right|}}}}\ge 0$ тэнцэтгэл бишийн хамгийн их шийдийг ол.
$\left({2^{x}-3^{x}}\right)\cdot\log_{x}\left({x^{2}-5x+7}\right)> 0$ тэнцэтгэл бишийг бод.
$\left|\log_{(x+5)}(x+2)^2\right|-2\le 0$
$\log_{\frac{1}{x^{2}}}{\left[{{\dfrac{{2\cdot \left({x-2}\right)}}{{\left({x+1}\right)\cdot\left({x-5}\right)}}}}\right]}\ge{\dfrac{{1}}{{2}}}$ тэнцэтгэл бишийг бод.
$\log_{{\left|{x-3}\right|}}{\left|{x^{2}-5x+6}\right|}< 2$ тэнцэтгэл бишийг бод.
$\log_{\sqrt{x+1}-\sqrt{x-1}}\left({x^{2}-3x+1}\right)\ge 0$ тэнцэтгэл бишийг бод.
$\log_{\sqrt{x+2}-\sqrt{x-1}}\left({x^{2}+x}\right)\ge 0$ тэнцэтгэл бишийг бод.
${\dfrac{{1}}{{2}}}\cdot\log_{\left({x-1}\right)}\left({x^{2}-8x+16}\right)+\log_{\left({4-x}\right)}\left({-x^{2}+5x-4}\right)> 3$ тэнцэтгэл бишийг бод.
$\log_{{\left|{x}\right|}}\left({\sqrt{9-x^{2}}-x-1}\right)\ge 1$ тэнцэтгэл бишийг бод.
$\log_{\left({x+1}\right)^{2}}8+3\cdot\log_{4}\left({x+1}\right)\ge{\dfrac{{259}}{{2\pi}}}\cdot\arcsin\left({\cos{\dfrac{{11\pi}}{{7}}}}\right)$ тэнцэтгэл бишийг бод.
${\dfrac{{2\cdot\log_{\left({1-3\cdot{\left|{x}\right|}}\right)}\left({42x^{2}-14\cdot{\left|{x}\right|}+1}\right)}}{{\log_{\left({1-3\cdot{\left|{x}\right|}}\right)}\left({x-{\dfrac{{5}}{{6}}}}\right)^{2}}}}\le 1$ тэнцэтгэл бишийг бод.
$\left({4^{-x}+3\cdot 2^{x}}\right)^{\log_{7}x-\log_{x}\frac{1}{7}-2}\le 1$ тэнцэтгэл бишийг бод.
$\log_{\frac{1}{3}}\left(2x+3\right)>2x-1$ тэнцэтгэл бишийн хамгийн бага бүхэл шийдийг ол.
  1. $\log_3x+\log_3(x-2)=1$ тэгшитгэл бод.
  2. $2\log_{\frac{1}{3}}(x-2)>\log_{\frac13} (2x-1)$ тэнцэтгэл биш бод.
  3. $y=\log_2(x+7)+\log_2(1-x)$ функцийн хамгийн их утгыг ол.
Тэгшитгэл, тэнцэтгэл бишийг бод.
  1. $\log_3 x=4$
  2. $\log_{\frac{1}{3}} x>2$
Тэгшитгэл, тэнцэтгэл бишийг бод.
  1. $\log_2(x^2+3x+4)=1$
  2. $\log_{\frac13}(-x)\geq 2$
  3. $\log_3(x-5)+\log_3(2x-3)=2$
  4. $\log_2x+\log_2(x-1)< 0$
  5. $\log_2(x+1)+\log_2(3-x)-\log_2(x-1)\leq \log_23$
  6. $\log_3x-\log_9(x+6)=0$
Тэгшитгэл, тэнцэтгэл бишийг бод.
  1. $(\log_3x)^2-2\log_3x=3$
  2. $(\log_2x)^2-\log_2x^4+3=0$
  3. $(\log_2x)^2-2\log_2x-4<0$
  4. $2(\log_{\frac13}x)^2+5\log_{\frac13}x-3>0$
Тэнцэтгэл бишийг бод.

  1. $2\log_2(10-x)\leq \log_2 (100-x^2)+1\leq\log_{\sqrt{2}} (10+x)-5$

  2. $\log_a (3x^2-3x-18)> \log_a(2x^2-10x)$
$\log_{\frac{1}{3}}^{2} (x-1) + 3 \ge -\dfrac{4}{5}\log_{\frac{1}{3}}(x - 1)^{5}$ тэнцэтгэл бишийг бод.

A. $(1;4]\cup[28;+\infty)$     B. $(1;+\infty)$     C. $[28;+\infty)$     D. $(1;4]$     E. $\varnothing$    
$\dfrac{1}{\log_{2} x - 4} > \dfrac{1}{\log_{2} x}$ тэнцэтгэл бишийн шийд аль нь вэ?

A. $]1;8[\cup]16;+\infty[$     B. $]16;+\infty[$     C. $]0;+\infty[$     D. $]0;1[\cup]16;+\infty[$     E. $]0;1[$    
$\dfrac{4}{\lg 10x} - \dfrac{5}{\lg 100x} \ge 0$ тэнцэтгэл бишийг бод.

A. $(0;1000]$     B. $(0;0.01)\cup(0.1;1000]$     C. $(0;0.01)$     D. $(0.1;1000]$     E. Шийдгүй    
$\dfrac{4\log_{0.3} x + 1}{\log_{0.3} x + 1}\le\log_{0.3}x + 1$ тэнцэтгэл бишийн шийдийг ол.

A. $\Big[\dfrac{9}{100};1\Big]$     B. $\Big(0;\dfrac{9}{100}\Big]\cup\Big[1;\dfrac{10}{3}\Big)$     C. $[0;+\infty)$     D. $\Big(0;\dfrac{9}{10}\Big]\cup\Big[1;\dfrac{100}{3}\Big)$     E. $\varnothing$    
$\dfrac{1}{5 - \lg x} + \dfrac{2}{1 + \lg x} < 1$ хамгийн бага бүхэл шийдийг ол.

A. $5$     B. $100$     C. $101$     D. $150$     E. $199$    
$\log_x(6x-1)-\log_x9\ge 2$ тэнцэтгэл бишийг бод.

A. $\big]\frac16;1\big[$     B. $\big]1;+\infty\big[$     C. $\big]0;\frac16\big[$     D. $\big]\frac16;+\infty\big[$     E. $\varnothing$    
$f(x)=\log_2^2x+12\log_4\sqrt{x}-4$ бол $f(x)\le 24$ тэнцэтгэл бишийг бод.

A. $0<x\le\frac1{128}$     B. $\frac1{128}\le x\le 16$     C. $16\le x$     D. $0<x\le 16$     E. $0<x\le 128$    
$\log_6 {(7x-9)} \leq 2$ тэнцэтгэл бишийн $[-2;5]$ завсарт байх бүхэл шийдүүдийн нийлбэрийг ол.

A. 12     B. 13     C. 14     D. 15     E. 16    
$2\log_{\frac{1}{3}}(x-2)>\log_{\frac{1}{3}}(2x-1)$ тэнцэтгэл биш бод.

A. $(0.5;1) \cup (5;+\infty)$     B. $(2;5)$     C. $(5;+\infty)$     D. $(-\infty;1) \cup (3;+\infty)$     E. $\varnothing$    
$\log_{0.1}\left(\dfrac2{x-4}\right)\leq\log_{0.1}(7-x)$ тэнцэтгэл бишийг бод.

A. $4< x\leq5$     B. $6\leq x<7$     C. $4< x\leq5\bigcup6\leq x<7$     D. $5< x<6$     E. $4< x<7$    
$\log_3 x\le -1$ тэнцэтгэл бишийг бод.

A. $]-\infty;+\infty[$     B. $]0;+\infty[$     C. $\big]0;\frac13\big]$     D. $]0;3]$     E. $]0;3[$    
$\log_3(5x-27)>\log_32x$ тэнцэтгэл бишийг бод.

A. $]-\infty;9[$     B. $]9;\infty[$     C. $]3;9[$     D. $]-\infty;3[$     E. $]2;9[$    
$\log_{0.5}5\le \log_{\frac12}(2x-1)$ тэнцэтгэл бишийг бод.

A. $[0.5;+\infty[$     B. $]-\infty;0.5]\cup[3;+\infty[$     C. $[3;+\infty[$     D. $]0.5;3]$     E. $]-\infty;0.5]$    
$\log_{\frac13}(2x+1)\le5^{\log_{25}4}$ тэнцэтгэл бишийг бод.

A. $\left[-\frac49;\infty\right[$     B. $\left]-\frac12;\frac49\right[$     C. $\left[-\frac12;\frac95\right[$     D. $\left[-\frac59;\infty\right[$     E. $\left[-\frac95;\infty\right[$    
$\log_{x+3}(x^2+3x+3)<1$ тэнцэтгэл бишийн шийд аль нь вэ?

A. $]-\infty;-2[\cup]-2;0[$     B. $]-2;0[$     C. $]-3;-2[\cup]-2;0[$     D. $]-3;-2[$     E. $]-3;0[$    
$\log_{x+1}(x^3+3x^2+2x)<2$ тэнцэтгэл бишийг бод.

A. $\varnothing$     B. $\big(-\frac{\sqrt5+1}{2};+\infty\big)$     C. $\big(-\frac{\sqrt5+1}{2};0\big)$     D. $\big(0;\frac{\sqrt5+1}{2}\big)$     E. $\big(0;\frac{\sqrt5-1}{2}\big)$    
$\log_3 x\le -1$ тэнцэтгэл бишийг бод.

A. $]-\infty;+\infty[$     B. $]0;+\infty[$     C. $\big]0;\frac13\big]$     D. $]0;3]$     E. $]0;3[$    

Модультай тэнцэтгэл биш

${\left| {4x^{2} + 35x + 38} \right|} > {\left| {12x^{2} + 33x + 32} \right|}$
${\left| {x^{2} - 4} \right|} - {\left| {9 - x^{2}} \right|} \ge 5$ тэнцэтгэл бишийг бод.
${\frac{{{\left| {x + 1} \right|} + {\left| {x - 2} \right|}}}{{x + 199}}} < 1$
$\dfrac{1}{x + 1} + \dfrac{2}{|x| - 1}\ge \dfrac{2}{x - 2}$ тэнцэтгэл бишийг бод.
$\dfrac{|2 - x| - x}{|x - 3| - 1} \le 2$ тэнцэтгэл бишийг бод.
$\dfrac{|x + 3| - 1}{4 - 2|x + 4|} \ge - 1$ тэнцэтгэл бишийг бод.
${\frac{{4x}}{{1 + x{}^{2}}}} < 1 + \sqrt {{\frac{{2x}}{{1 + x^{2}}}}} $
$\sqrt {{\frac{{1}}{{x^{2}}}} - {\frac{{3}}{{4}}}} < {\frac{{1}}{{x}}} - {\frac{{1}}{{2}}}$
${\frac{{\sqrt {3x - 10\sqrt {x} + 3}} }{{x - 3\sqrt {x} - 10}}} < 0$
${\frac{{1 - \sqrt {1 - 4x^{2}}} }{{x}}} < 3$
$\sqrt {x - 2} + {\left| {x - 8} \right|} \le 6$
$\sqrt {x + 2} + {\left| {x - 4} \right|} \le 6$
$3\sqrt {x^{2} + {\left| {x} \right|} - 2} \ge 1 - x$
$2\sqrt {x^{2} - {\left| {x} \right|} - 2} \ge x - 2$
$\sqrt {x^{2} - 5} + 3 > {\left| {x - 1} \right|}$
$3\sqrt {x^{2} + x - 2} \ge {\left| {x + 2} \right|} - 1$
$5 \cdot \sqrt {1 - {\frac{{1}}{{z}}}} > {\frac{{7z - 1}}{{z}}}$
${\frac{{\sqrt {x^{2} + x + 6} + 3x + 13}}{{x + 5}}} > 1$
$\left( {{\frac{{1}}{{5}}}} \right)^{{\left| {x - 2} \right|}} > \left( {{\frac{{1}}{{25}}}} \right)^{{\left| {x} \right|}}$
$6^{x + 2} \ge 4 \cdot 7^{{\left| {x + 1} \right|}}$
$3^{x + 1} > 7 \cdot 5^{{\left| {x - 1} \right|}}$
$25^{x + 1} \ge 10 \cdot 32^{{\left| {x - 1} \right|} + 1}$
$2^{x} + 2^{{\left| {x} \right|}} \ge 2\sqrt {2} $
${\left| {2^{{\frac{{x}}{{3}}}} - {\frac{{11}}{{2}}}} \right|} \le {\frac{{5}}{{2}}}$ бүхэл шийдүүдийн нийлбэрийг ол.
${\left| {\sqrt {3^{x}} - {\frac{{11}}{{2}}}} \right|} \le {\frac{{7}}{{2}}}$ бүхэл шийдүүдийн нийлбэрийг ол.
${\left| {4^{x - 1} - {\frac{{5}}{{4}}}} \right|} \le {\frac{{4^{x}}}{{8}}} + {\frac{{3}}{{4}}}$ бүхэл шийдүүдийн нийлбэрийг ол.
$\left( {2.5} \right)^{\left( {x + 1} \right)^{2}} \cdot \left( {0.4} \right)^{{\left| {4x - 4} \right|}} \ge \left( {{\frac{{25}}{{4}}}} \right)^{{\frac{{13}}{{2}}}}$
$\log _{0.5} \left( {6{\left| {x} \right|} - 3} \right) \le \log _{0.5} \left( {4 - x^{2}} \right)$
$\lg {\left| {{\frac{{x - 1}}{{2x + 1}}} > 0} \right|}$ бүхэл шийдүүдийг ол.
$\log _{5}^{2} x + {\left| {\log _{5} x} \right|} \ge 6$
$\log _{{\frac{{1}}{{3}}}}^{2} x + {\left| {\log _{{\frac{{1}}{{3}}}} x} \right|} > 2$ хамгийн бага бүхэл шийдийг ол.
$\log _{{\left| {x} \right|}} \log _{2} \left( {4^{x} - 12} \right) \le 1$
$\log_{x^{2}}{\dfrac{{4x-5}}{{{\left|{x-2}\right|}}}}\ge{\dfrac{{1}}{{2}}}$ тэнцэтгэл бишийн хамгийн их шийдийг ол.
$\log_{3}{\dfrac{{{\left|{x^{2}-4x}\right|}+3}}{{x^{2}+{\left|{x-5}\right|}}}}\ge 0$ тэнцэтгэл бишийн хамгийн их шийдийг ол.
$\left|\log_{(x+5)}(x+2)^2\right|-2\le 0$
$\log_{{\left|{x-3}\right|}}{\left|{x^{2}-5x+6}\right|}< 2$ тэнцэтгэл бишийг бод.
$\log_{{\left|{x}\right|}}\left({\sqrt{9-x^{2}}-x-1}\right)\ge 1$ тэнцэтгэл бишийг бод.
$|4x+1|< 3$ тэнцэтгэл бишийг бод.
Дараах тэнцэтгэл бишийг бод.
  1. $|x-2|<4$
  2. $|x+3|\ge 5$
  3. $|x-4|<3x$
  4. $|x-1|+2|x-3|\le 11$
Дараах тэнцэтгэл бишийг бод.
  1. $|x+3|\le 2$
  2. $|x-4|>3$
  3. $3|x+1|< x+5$
  4. $|x-5|\le\dfrac23|x|+1$
$|2x+1|+|3x+2|\le 5x+3$ тэнцэтгэл бишийг бод.
$\sqrt {x - 3} \le 3 - {\left| {x - 6} \right|}$ тэнцэтгэл бишийг бод.

A. $[7;+\infty[$     B. $]-\infty;4]$     C. $[3;7]$     D. $[4;7]$     E. $\{3\}\cup[4;7]$    
$\left|\dfrac{3x-4}{3-x}\right|\le 2$ тэнцэтгэл бишийг бод.

A. $x\le-2$     B. $-2\le x$     C. $-2\le x\le 2$     D. $-2 < x\le 2$     E. $-2\le x<2$    
$|x^2-2x-6|\le 6-x$ тэнцэтгэл бишийн бүхэл тоон шийдийг ол.

A. $-3$; $-2$; $-1$; $0$; $3$; $4$     B. $-3; -2; -1$     C. $0$; $3$; $4$     D. $3$; $4$     E. $-2$; $-1$    
$\left|x^{2} -x-6\right|< x+3$ тэнцэтгэл бишийн бүхэл тоон шийдүүдийг ол.

A. $2$; $3$; $4$     B. $-2$; $2$; $3$     C. $3$; $4$     D. $-2$; $2$; $3$; $4$     E. $-2$; $3$; $4$    
$\big||2x+4|-7\big|<9$ тэнцэтгэл бишийг бод.

A. $]-7; 4[$     B. $]3; 6[$     C. $]-10; 6[$     D. $]8; 10[$     E. $]-2; 5[$    
$|x-1|\ge 3$ тэнцэтгэл бишийг бод.

A. $[-2;4]$     B. $]-\infty;-2]\cup[4;+\infty[$     C. $]-\infty;-4]\cup[2;+\infty[$     D. $[-4;2]$     E. $]-\infty;-2]\cup]4;+\infty[$    
$|2x+3|\le 4+|x|$ тэнцэтгэл бишийн шийдийн интервалын уртыг ол.

A. 6     B. 7     C. 8     D. 9     E. 10    
$\dfrac{9}{|x-5|-3}\leq|2-x|$ тэнцэтгэл бишийн бүх шийдийг ол.

A. $[-1;2]\cup[5-3\sqrt{2};5]$     B. $[-1;2]\cup[5;5+3\sqrt{2}]$     C. $[5-3\sqrt{2};2]\cup[5;5+3\sqrt{2}]$     D. $[-1;5]\cup [5+3\sqrt{2};7]$     E. $]-\infty;-1]\cup ]2;8[\cup[5+3\sqrt2;+\infty[$    
$|x-a|< 1$ тэнцэтгэл бишийг бод.

A. $]a-1;a+1[$     B. $[a-1;a+1]$     C. $]-1;1[$     D. $]-\infty; a-1[\cup]a+1;+\infty[$     E. Шийдгүй    
$|x+4|\ge 3|x|$ тэнцэтгэл бишийн шийдийн муж ямар урттай вэ?

A. $-3$     B. $3$     C. $4$     D. $2.5$     E. $1$    
$|3x-7|< 2$ тэнцэтгэл биш хэчнээн бүхэл шийдтэй вэ?

A. нэг ч байхгүй     B. нэг     C. хоёр     D. гурав     E. зургаа    
$\ctg x\Big(\sec x-\dfrac{\cos x}{1+\sin x}\Big)=?$

A. $\tg x$     B. $\sin x$     C. $\cos x$     D. $\ctg x$     E. $1$    
$|x+2|>8-x$ тэнцэтгэл бишийг бод.

A. $x<-2;$ $x>3$     B. $x>3$     C. $x<-2$     D. $-2< x<3$     E. $x>-2$    
$|20x+12| \le 2012$ тэнцэтгэл биш хэдэн бүхэл шийдтэй вэ?

A. төгсгөлгүй олон     B. $200$     C. $201$     D. $202$     E. $203$    
$|x-1|+|2-x|>3+x$ тэнцэтгэл бишийг бод.

A. $]-\infty;6[\cup]6;+\infty[$     B. $]0;6[\cup]6;+\infty[$     C. $(-\infty;0)\cup]-\infty)$     D. $]-\infty;0[\cup]6;+\infty[$     E. $]-\infty;-4[$    
$|2x+1|+|3x+2|\le5x+3$ тэнцэтгэл бишийг бод.

A. $[-\frac12;+\infty[$     B. $]-1;+\infty[$     C. $[0;+\infty[$     D. $[1;+\infty[$     E. $]-\infty;-\frac12]$    
$|x-2|< 4$ тэнцэтгэл бишийг бод.

A. $]-\infty;6[$     B. $]2;6[$     C. $]-2;6[$     D. $]-2;2[$     E. $]0;6[$    
$|x-5|<3$ тэнцэтгэл бишийг бод.

A. $]-\infty;8[$     B. $]5;8[$     C. $]2;8[$     D. $[5;8[$     E. $]0;8[$    
$|1-x|\le 3$ тэнцэтгэл бишийн шийд аль нь вэ?

A. $]2;4[$     B. $[1;3]$     C. $[-2;4]$     D. $[-1;1]$     E. $[2;4]$    
$|x-3|< 5$ тэнцэтгэл бишийг бод.

A. $]-\infty;8[$     B. $]2;8[$     C. $]-2;8[$     D. $[2;8[$     E. $]0;8[$    
$|x-4|< 2$ тэнцэтгэл бишийг бод.

A. $]-\infty;6[$     B. $]2;6[$     C. $]-2;6[$     D. $]-2;2[$     E. $]0;6[$    
$|x-2|\geq 5$ тэнцэтгэл бишийг бод.

A. $[-3;7]$     B. $]-\infty;-7]\cup[3;+\infty[$     C. $]-\infty;-3]\cup[7;+\infty[$     D. $[-7;3]$     E. $]-\infty;-7[\cup]3;+\infty[$    
$\dfrac{|x-1|}{x^2-5x+6}\le 0$ бод.

A. $2< x<3$     B. $2< x<3$, $x=1$     C. $x>2$     D. $x<3$     E. $x<3$, $x\neq1$    
$|5x-3|<1$ тэнцэтгэл бишийн шийдийн олонлог аль нь вэ?

A. $]-\infty;-0.4[$     B. $]-0.8;-0.4[$     C. $]-0.4;0.4[$     D. $]0.4;0.8[$     E. $]0.8;+\infty[$    
$|4x+3|<8$ тэнцэтгэл биш хэдэн бүхэл шийдтэй вэ?

A. 1     B. 2     C. 3     D. 4     E. төгсгөлгүй олон    
$|x-2|+|x+1|>5$ тэнцэтгэл бишийг бод.

A. $]-1;2[$     B. $]-\infty,2[\cup]3;+\infty[$     C. $]-\infty,-2[$     D. $]-\infty,-2[\cup]3;+\infty[$     E. $]3;+\infty[$    
Дараах тоонуудын аль нь $|x-2|<5$ тэнцэтгэл бишийг хангах вэ?

A. $7$     B. $-4.5$     C. $9$     D. $-3$     E. $0$    
$|x^2-1|-2x<0$ тэнцэтгэл бишийн бүхэл тоон шийдүүдийг ол.

A. $0$; $1$     B. $1$; $2$     C. $2$     D. $0$; $1$; $2$     E. $\varnothing$    
$\dfrac{x^2-6x+5}{|x-1|}\le 0$ тэнцэтгэл бишийг бод.

A. $]1;5[$     B. $[-5;-1[$     C. $]-5;-1]$     D. $[1;5]$     E. $]1;5]$    
$(x-2)^2-6|x-2|-7\le 0$ тэнцэтгэл бишийн шийдийг ол.

A. $[-9;9]$     B. $[-7;-1]$     C. $[-9;5]$     D. $[-5;9]$     E. $[1;7]$    
$\dfrac{3}{1+|x+3|}<1$ тэнцэтгэл бишийг бод.

A. $]-1;-5[$     B. $\varnothing$     C. $]-\infty;5[$     D. $]1;5[$     E. $]-\infty;-5[\cup]-1;+\infty[$    
$|6x-5|\le 8$ тэнцэтгэл бишийн шийд нь

A. $-\dfrac12\le x\le\dfrac12$     B. $0\le x\le\dfrac56$     C. $-1\le x\le\dfrac12$     D. $-\dfrac12\le x\le\dfrac{13}{6}$     E. $-\dfrac12\le x\le\dfrac13$    
$|3-2x|\le 4+|x|$ тэнцэтгэл бишийн шийдийн интервалын уртыг ол.

A. 6     B. 7     C. 8     D. 9     E. 10    
$|x+2|>8-x$ тэнцэтгэл бишийг бод.

A. $x<-2;$ $x>3$     B. $x>3$     C. $x<-2$     D. $-2< x<3$     E. $x>-2$    
$|x-a| > 1$ тэнцэтгэл бишийг бод.

A. $]a-1;a+1[$     B. $[a-1;a+1]$     C. $]-1;1[$     D. $]-\infty; a-1[\cup]a+1;+\infty[$     E. Шийдгүй    
$\sqrt {x - 4} \le 6 - {\left| {x - 10} \right|}$ тэнцэтгэл бишийг бод.

A. $[13;+\infty[$     B. $]-\infty;4]$     C. $[4;13]$     D. $[5;13]$     E. $\{4\}\cup[5;13]$    
$|x-|x-|x-2|||>2$ тэнцэтгэл бишийг бодьё.
  1. $x\ge\fbox{a}$ үед $x>\fbox{b}$ гэсэн шийдтэй.
  2. $\fbox{c}\le x< \fbox{a}$ үед $|x-|x-|x-2|||=2-x>2$ болох тул шийдгүй.
  3. $x< \fbox{c}$ үед $|3x-2|>2$ болох ба $x>\dfrac{\fbox{d}}{\fbox{e}}$ үед $3x-2>2\Rightarrow x>\dfrac43$ тул шийдгүй, $x\le\dfrac{\fbox{d}}{\fbox{e}}$ үед $x< \fbox{f}$ гэсэн шийдтэй байна.
$y=|x|+1 , -3\leq x\leq 2 $ функцийн хувьд $\fbox{a}\leq y\leq \fbox{b} ,$ $y=|2x+4| , -3\leq x\leq 1 $ функцийн хувьд $\fbox{c}\leq y\leq \fbox{d} ,$ байна.
$y=|2x-6| , 1\leq x\leq 4 $ функцийн хувьд $\fbox{a}\leq y\leq \fbox{b} ,$ $y=1-|x+3| , -4\leq x\leq 1 $ функцийн хувьд $-\fbox{c}\leq y\leq \fbox{d} ,$ байна.
$x^2-|5x-3|-x< 2$ тэнцэтгэл бишийн шийд $]\fbox{ab};\fbox{c}+\fbox{d}\sqrt{\fbox{e}}[$ байна.
$3x^2-|x-3|>9x-2$ тэнцэтгэл бишийн шийд $$]-\infty;\dfrac{\fbox{a}-\sqrt{\fbox{bc}}}{\fbox{d}}[\cup] \dfrac{\fbox{a}+\sqrt{\fbox{bc}}}{\fbox{d}};+\infty[$$ байна.
$x|3x+5|=3x^2+4x+3$ тэгшитгэл нь $x< -\dfrac53$ бол $2x^2+\fbox{a}x+\fbox{b}=0$ тэгшитгэлд шилжих ба $x_1=-\fbox{c}, x_2=-\dfrac1{\fbox{d}}$ шийдтэй байна. Эдгээр шийдүүд нь $x< -\dfrac53$ нөхцлийг хангахгүй тул энэ тохиолдолд анхны тэгшитгэлийн шийд болохгүй. $x\geq-\dfrac53$ бол $\fbox{e}x^2+x-\fbox{f}=0$ тэгшитгэлд шилжих ба $x=\fbox{g}$ шийдтэй байна. Энэ шийд $x\geq-\dfrac53$ нөхцлийг хангах тул анхны тэгшитгэлийн шийд болно.
$x|2x+5|=2x^2+3x+2$ тэгшитгэл нь $x< -\dfrac52$ бол $\fbox{a}x^2+\fbox{b}x+1=0$ тэгшитгэлд шилжих ба $x_{12}=\dfrac12(-\fbox{c}\pm\sqrt{\fbox{d}})$ шийдтэй байна. Эдгээр шийдүүд нь $x< -\dfrac52$ нөхцлийг хангахгүй тул энэ тохиолдолд анхны тэгшитгэлийн шийд болохгүй. $x\geq-\dfrac52$ бол $\fbox{e}x^2+x-\fbox{f}=0$ тэгшитгэлд шилжих ба $x=\fbox{g}$ шийдтэй байна. Энэ шийд $x\geq-\dfrac52$ нөхцлийг хангах тул анхны тэгшитгэлийн шийд болно.
$\left|2-\dfrac{1}{x-4}\right|<3$ тэнцэтгэл бишийн шийд нь $$]-\infty;\fbox{a}[\cup\bigg]\dfrac{\fbox{bc}}{\fbox{d}};+\infty\bigg[$$ байна.

Модультай хялбар шугаман тэнцэтгэл биш


Параметртэй тэнцэтгэл биш

$x=2$ үед ${\dfrac{{x^{3}-x^{2}}}{{b^{2}x^{2}+x+2}}} \le{\dfrac{{x^{2}-3}}{{b^{2}x+b-1}}}$ тэнцэтгэл биш биелэх $b$ параметрийн бүх бүхэл утгыг ол.
$x=1$ үед $\dfrac{2x^{3}-1}{ax+2x^{4}}< \dfrac{x}{ax^{5}+5}$ тэнцэтгэл биш биелэх $a$ параметрийн бүх бүхэл утгыг ол.
$m$-ийн ямар утганд $f\left({x}\right)=\sqrt{2mx-x^{2}-5}+\sqrt{1-x}$ функцийн тодорхойлогдох муж нь нэг цэгээс бүрдэх вэ?
$n$-ын ямар утганд $f\left({x}\right)=\sqrt{x-7}-\sqrt{n-4x-x^{2}}$ функцийн тодорхойлогдох муж нь нэг цэгээс бүрдэх вэ?
$p$-н ямар утганд $y=\sqrt{\left({3p+1}\right)x-p\left({4+x^{2}}\right)}$ функц нь $x$-н бүх бодит утганд тодорхойлогдох вэ?
$a$ параметрийн ямар утганд $ax^{2}+2ax+4>0$ тэнцэтгэл биш бүх тоон шулуун дээр биелэх вэ?
$x$-н бүх утганд $ax^{2}+4x-1+2a>0$тэнцэтгэл биш биелэх $a$-гийн хамгийн бага бүхэл утгыг ол.
$a=5$ үед $x$-н бүх утганд $4^{x}+\left({a-1}\right)\cdot 2^{x}+\left({2a-5}\right)>0$ тэнцэтгэл биш биелэхийг харуул. Мөн $x$-н бүх утганд тэнцэтгэл биелэх $a$-гийн бусад утгыг ол.
$a=2$ үед $x$-н бүх утганд $9^{x}+\left({2a+4}\right)\cdot 3^{x}+8a+1>0$ тэнцэтгэл биш биелэхийг харуул. Мөн $x$-н бүх утганд тэнцэтгэл биелэх $a$-ийн бусад утгыг ол.
$a$ параметрийн ямар утганд $36^{x}+a\cdot 6^{x}+a+8\le 0$ гэсэн тэнцэтгэл биш нэг шийдтэй байх вэ?
$p$-н ямар утганд $25^{x}-p\cdot 5^{x}+3-p\le 0$ гэсэн тэнцэтгэл биш нэг шийдтэй байх вэ?
Дурын $x$-ийн хувьд $a^{2}+2a-\sin ^{2}x-2a\cos x> 2$ тэнцэтгэл биш биелэх $a$-н бүх утгыг ол.
Дурын $2< c< 4$-ийн хувьд $\left({2c-6}\right)x^{2}+\left({32-10c}\right)x-\left({8+c}\right) < 0$ байх $x$-н бүх утгыг ол.
Дурын $a\in\left(1;3\right)$-ийн хувьд $\left({4-2a}\right)x^{2}+\left({13a-27}\right)x+\left({33-13a}\right)> 0$ тэнцэтгэл биш биелэх $x$-н бүх утгыг ол.
$x^{2}+ax+a-7\le 0$ тэнцэтгэл бишийн шийд бүр $2x^{2}+5x+2\le 0$ тэнцэтгэл бишийн шийд болох $a$-н хамгийн их бүхэл утгыг ол.
$2x^{2}-9x+4\le 0$ тэнцэтгэл бишийн шийд бүр ${\left|{2x-a}\right|}\le 5$ гэсэн тэнцэтгэл бишийн шийд болох $a$-н бүх бүхэл утгуудын нийлбэрийг ол.
Дурын $x$-н хувьд $1+\log_{5}\left({x^{2}+1}\right)\ge\log_{5}\left({ax^{2}+4x+a}\right)$ тэнцэтгэл биш биелэх $a$-н утгыг ол.
Дурын $x$-н хувьд ${\dfrac{{1-ax-x^{2}}}{{x^{2}+2x+2}}}\le 2$ тэнцэтгэл биш биелэх $a$-н утгуудын нийлбэрийг ол.
Дурын $x$-н хувьд $-3< {\dfrac{{x^{2}+ax-2}}{{x^{2}-x+1}}}< 2$ тэнцэтгэл биш биелэх $a$-н хамгийн их утгыг ол.
$\left({a-x}\right)\cdot\sqrt{3+x-x^{2}}\ge 0$ тэнцэтгэл биш зөвхөн 2 шийдтэй байх $a$-н утгыг ол.
$x$-н бүх утганд $\left\{\begin{array}{c} -x^{2}+12x-a\ge 0\\ x\le 2 \end{array}\right.$ нөхцөл биелэх дор хаяж нэг $x$ олдох $a$-н утгыг ол.
$\left\{\begin{array}{c} a+\sin bx\le 1\\ x^{2}+ax+1\le 0 \end{array}\right.$ тэнцэтгэл биш яг нэг шийдтэй байх $a$ ба $b$-н утгыг ол.
$a>0$ үед $\sqrt{2ax-x^{2}}\ge a-x$ тэнцэтгэл бишийг бод.
$a$ параметрийн хувьд $3^{\sqrt{x+1}}> 2^{a-1}$ тэнцэтгэл бишийг бод.
$k$ параметрийн бүх утганд $\log_{3}\left({x-2k+1}\right)+2\le\log_{3}\left({x+k-5}\right)$ тэнцэтгэл бишийг бод.
$x^{2}+4x+6a\cdot{\left|{x+2}\right|}+9a^{2}\le 0$ тэнцэтгэл бишийн шийдүүд 1-ээс их байх $a$ параметрын бүх утгыг ол.
$3ax+2\sqrt{3x+1}-6x+a-5< 0$ тэнцэтгэл бишийг хангах $x$ нь $[1;5]$ завсарт харъяалагдах $a$-н бүх утгыг ол.
$2ax+2\sqrt{2x+3}-2x+3a-5< 0$ тэнцэтгэл бишийг хангах $x$ нь $[-1;3]$ завсарт харъяалагдах $a$-н бүх утгыг ол.
$a\cdot\left({2+\sin ^{2}x}\right)^{4}+\cos^{2}x+a>11$ тэнцэтгэл биш $x$-н бүх утганд биелэх $a$-н бүх бодит утгыг ол.
$\cos x-2\sqrt{x^{2}+9}\le-{\dfrac{{x^{2}+9}}{{a+\cos x}}}-a$ тэнцэтгэл биш нэг шийдтэй байх $a$ параметрын бүх утгыг ол.
$\left({p-x^{2}}\right)\cdot\left({p+x-2}\right)< 0$ тэнцэтгэл бишийн шийд нь $x^{2}\le 1$ гэсэн тэнцэтгэл бишийн шийд болохгүй байх $p$-н бүх утгйг ол.
$2>{\left|{x+a}\right|}+x^{2}$ тэнцэтгэл биш ядаж нэг эерэг шийдтэй байх $a$ параметрын утгыг ол.
$x$-н дурын утганд ${\left|{\sin ^{2}x-2\left({a-1}\right)\cdot\sin x\cdot\cos x+5\cos ^{2}x+2-a}\right|}\le 6$ тэнцэтгэл биш биелэх $a$ параметрын бүх утгыг ол.
$[1;3]$ хэрчимд ${\left|{2x^{2}+ax+b}\right|}>1$ тэнцэтгэл биш шийдгүй байх $a, b$ бүх хос тоонуудыг ол.
6.23.10. [МГУ, мех.-мат] $\left({a+2}\right)x^{3}-\left({1+2a}\right)x^{2}-6x+\left(a^{2}+4a-5\right)>0$ гэсэн тэнцэтгэл биш $a\in{\left[{-2;1}\right]}$ хэрчим дээр ядаж нэг шийдтэй байх $x$-н бүх утгыг ол.
$9^{x}< 20\cdot 3^{x}+a$ тэнцэтгэл биш нэг ч бүхэл шийдгүй байх $a$-н бүх утгыг ол.
$a\sqrt{a}\cdot\left({x^{2}-2x+1}\right)+{\dfrac{{\sqrt{a}}}{{x^{2}-2x+1}}}\le\sqrt[{4}]{{a^{3}}}\cdot{\left|{\sin{\dfrac{{\pi x}}{{2}}}}\right|}$ тэнцэтгэл биш нэг шийдтэй байх $a$-н хамгийн их утгыг ол.
Дурын $m$-н хувьд $x\big({\pi\cdot\left({x+1}\right)-4\arcctg\left({3m^{2}+12m+11}\right)}\big)>0$ тэнцэтгэл бишийн шийд болох ба $x\in{\left[{-3;1}\right]}$ байх $x$-н бүх утгыг ол.
$\left\{\begin{array}{c} y\ge x^{2}+2a\\ x\ge y^{2}+2a \end{array}\right.$ систем цор ганц шийдтэй байх $a$ параметрын утгыг ол.
6.23.15. [МГУ, филолог.ф-т] $\left\{\begin{array}{c} y\ge\left({x-b}\right)^{2}\\ x\ge\left({y-b}\right)^{2} \end{array}\right.$ систем цор ганц шийдтэй байх $b$ параметрын утгыг ол. ол.
$\left\{\begin{array}{*{20}c} {x^{2}+a^{2}+a^{4}\le 2a^{4}}\hfill\\ {3^{ax+2x}-3^{-a}\ge 0}\hfill\\ \end{array}\right.$ систем нэг шийдтэй байх а параметрын бүх утгыг ол.
$x^{2}-3x+3{\left|{x+c}\right|}+c\le 0$ тэнцэтгэл бишийн шийд бүхэл байх $c$ параметрын утгыг ол.
$x\in\left[\dfrac{\pi}{2};\dfrac{3\pi }{2}\right]$ хэрчим дээрх $\pi+\dfrac{6}{5}\left(x^{2}+ax \right)+\cos\left(x^{2}+ax\right)+\sin\left({2x^{2}+2ax+ {\dfrac{{\pi}}{{3}}}}\right)< 0$ тэнцэтгэл биш биелэх $a$-ын бүх утгыг ол.
$a\ge 0$ нөхцлийг хангах үед $x+\sqrt{a-x}> 0$ тэнцэтгэл бишийг бод.
$m$-ийн утгуудад $m\cdot{\left|{x-4}\right|}>x+1$ тэнцэтгэл бишийг бод.
$a$-ын бүх утганд $12\cdot 11^{\sqrt{3-x}}+a\cdot 11^{x-2}> 11^{x+\sqrt{3-x}-2}+12a$ тэнцэтгэл бишийг бод.
$c$-н бүх утганд $c+1< \left({c+2}\right)\cdot 3^{\sqrt {x-1}}$ тэнцэтгэл бишийг бод.
$a$ параметрын бүх утганд $\dfrac{2}{3}\le\log_{64}(x+a-2)+\log_{\frac{1}{4}}\sqrt[3]{x-a+8}$ тэнцэтгэл бишийг бод.
$a$ параметрын утга бүрд $\log_{2}ax+\log_{a}x\le 1$ тэнцэтгэл бишийг бод.
$f\left({x}\right) =\log _{a}\left({1-8a^{-x}} \right)$ функц өгөгджээ. < p>< /p>а) $f\left({x}\right)$ функцийн тодорхойлогдох мужийг ол. < p>< /p>б) $a ={\dfrac{{1}}{{2}}}$ үед $f\left({x}\right)> x+5$ бод. < p>< /p>в) бүх боломжит $a$-н хувьд $f\left({x}\right)+2x>0$ тэнцэтгэл бишийг бод.
Дурын $a$-н хувьд $x^{2}-3\le\left(a-\dfrac{3}{a}\right)x$ тэнцэтгэл бишийг бод.
$ax^{3}+9x\ge-3\left({a+1}\right)x^{2}$ дурын $a$-ийн хувьд бод.
$a, b> 0$ нөхцөлд $\sqrt{\dfrac{1}{x^{2}}-\dfrac{1}{a^{2}}}>\dfrac{1}{x}-\dfrac{1}{b}$ тэнцэтгэл бишийг бод.
$ax\le{\left|{x^{2}-5x+6}\right|}$ тэнцэтгэл бишийг бод.
$a$ параметрын утга бүрд ${\left|\dfrac{1}{x}+ 2a\right|}\le x$ тэнцэтгэл бишийг бод.
$x\geq 0$ үед $3a^2x-x^3\leq 16$ байх $a$-ийн бүх утгуудыг ол.
$\left\{\begin{array}{c}a(x-2)\ge x-3\\8(a+1)x\ge 8ax+9\end{array}\right.$ системийг боджээ. Хэрвээ
  1. $a\ge\dfrac{15}{7}$ үед $x\ge2+\dfrac{1}{1-a}$
  2. $\dfrac{15}{7}>a>1$ үед $x\ge1\dfrac18$
  3. $a=1$ үед $x\ge\dfrac98$
  4. $a< 1$ үед $\dfrac{9}{8}\le x\le\dfrac{2a-3}{a-1}$
гэсэн дөрвөн өгүүлбэр өгөгдсөн бол анхны системийн хариуг заа.

A. 1, 2 ба 3     B. 1, 2 ба 4     C. 1, 2, 3 ба 4     D. 2, 3 ба 4     E. 1 ба 3    
$\left\{\begin{array}{c}a(x-2)\ge x-3\\8(x-1)\ge1\end{array}\right.$ системийг боджээ. Хэрвээ
  1. $a\ge15/7$ үед $x\ge\dfrac{2a-3}{a-1}$
  2. $15/7>a>1$ үед $x\ge1\dfrac18$
  3. $a=1$ үед $x\ge 9/8$
  4. $a< 1$ үед $9/8\le x\le \dfrac{2a-3}{a-1}$
гэсэн дөрвөн өгүүлбэр өгөгдсөн бол анхны системийн хариуг заа.

A. 1, 2 ба 3     B. 1, 2 ба 4     C. 1, 2, 3 ба 4     D. 2, 3 ба 4     E. 1 ба 3    
$\dfrac x{a-4}\ge 3x-2a$ тэнцэтгэл бишийг бод.

A. $\left\{\begin{array}{ll} x\in\left]\dfrac{a(a-4)}{3a-14};+\infty\right[, & a\in\left]4;\dfrac{14}3\right[ \\ x\in\left]-\infty;\dfrac{a(a-4)}{3a-14}\right], & a\in]-\infty;4[\cup\left]\dfrac{14}3;+\infty\right[ \\ \varnothing, & a=\dfrac{14}3 \end{array}\right.$     B. $\left\{\begin{array}{ll} x\in\left]\dfrac{a(a-4)}{3a-13};+\infty\right[, & a\in\left]4;\dfrac{13}3\right[ \\ x\in\left]-\infty;\dfrac{a(a-4)}{3a-13}\right], & a\in]-\infty;4[\cup\left]\dfrac{13}3;+\infty\right[\\ x=\dfrac{2a(a-4)}{3a-13}, & a=\dfrac{13}3 \end{array}\right.$     C. $\left\{\begin{array}{ll} x\in\left]\dfrac{2a(a-4)}{3a-13};+\infty\right[, & a\in\left]4;\dfrac{13}3\right[\\ x\in\left]-\infty;\dfrac{2a(a-4)}{3a-13}\right], & a\in]-\infty;4[\cup\left]\dfrac{13}3;+\infty\right[\\ x\in\mathbb{R}, & a=\dfrac{13}3 \end{array}\right.$     D. $\left\{\begin{array}{ll} x\in\left]-\infty;\dfrac{2a(a-4)}{3a-15}\right], & a\in]-\infty;4[\cup\left]\dfrac{15}3;+\infty\right[ \\ x\in\left]\dfrac{2a(a-4)}{3a-15};+\infty\right[, & a\in\left]4;\dfrac{15}3\right[ \\ x\in\mathbb{R}, & a=\dfrac{15}3 \end{array}\right.$     E. $\left\{\begin{array}{ll} x\in\left]-\infty;\dfrac{2a(a-4)}{3a-15}\right], & a\in]-\infty;4[\cup\left]\dfrac{13}3;+\infty\right[ \\ x\in\left]\dfrac{2a(a-4)}{3a-15};+\infty\right[, & a\in\left]4;\dfrac{13}3\right[ \\ x\in\mathbb{R}, & a=\dfrac{13}3 \end{array}\right.$    
$|x+1|-|x-1|\ge a(x+1)$ тэнцэтгэл бишийг бод.

A. $\left\{\begin{array}{rl}0< a\le1 & \mbox{бол }x\in]-\infty;\frac{a}{a+2}]\cup[\frac a{a-2};\frac{a-2}a] \\a>1 & \mbox{бол }x\in]-\infty;\frac{a+2}{a}]\end{array}\right.$     B. $\left\{\begin{array}{rl}a\le0 & \mbox{бол }x\in\mathbb{R} \\0< a\le1 & \mbox{бол }x\in]-\infty;\frac{a+2}{a}]\cup[\frac a{2-a};\frac{2-a}a] \\a>1 & \mbox{бол }x\in]-\frac{a+2}a;+\infty[\end{array}\right.$     C. $\left\{\begin{array}{rl} a\le0 & \mbox{бол }x\in\mathbb{R} \\0< a\le1 & \mbox{бол }x\in]-\infty;\frac a{a+2}]\cup[\frac{2-a}a;\frac a{a-2}]\end{array}\right.$     D. $\left\{\begin{array}{rl} a\le0 & \mbox{бол }x\in[\frac{a}{2-a};+\infty[ \\ 0< a\le1 & \mbox{бол }x\in]-\infty;-\frac{a+2}{a}]\cup[\frac a{2-a};\frac{2-a}a] \\ a>1 & \mbox{бол }x\in]-\infty;-\frac{a+2}a] \end{array}\right.$     E. $x\in\left]\frac{2}{a}-1;+\infty\right[$    
$5|x|>|x-a|$ тэнцэтгэл бишийг бод.

A. $\left\{\begin{array}{rl} a\in]-\infty;-1[ & \mbox{бол }x\in]-\infty;\frac a6[\cup]-\frac a4;+\infty[ \\ a=-1 & x\in\mathbb{R} \\ a\in]0;+\infty[ & \mbox{бол }x\in]-\infty;\frac a6[\cup]\frac a4;+\infty[ \end{array}\right.$     B. $\left\{\begin{array}{rl} a\in]-\infty;1[ & \mbox{бол }x\in]-\infty;\frac a6[\cup]-\frac a4;+\infty[ \\ a=1 & \mbox{бол }x\in]-\infty;1[\cup]1;+\infty[ \\ a\in]1;+\infty[ & \mbox{бол }x\in]-\infty;-\frac a4[\cup]\frac a6;+\infty[ \end{array}\right.$     C. $\left\{\begin{array}{rl} a\in]-\infty;0[ & \mbox{бол }x\in]-\infty;\frac a6[\cup]-\frac a4;+\infty[ \\ a=0 & x\in]-\infty;0[\cup]0;+\infty[ \\ a\in]0;+\infty[ & \mbox{бол }x\in]-\infty;-\frac a4[\cup]\frac a6;+\infty[ \end{array}\right.$     D. $\left\{\begin{array} {rl}a\in]-\infty;0[ & \mbox{бол }x\in]-\infty;-\frac a4[\cup]\frac a6;+\infty[ \\ a=0 & x\in\mathbb{R} \\ a\in]0;+\infty[ & \mbox{бол }x\in]-\infty;\frac a6[\cup]-\frac a4;+\infty[ \end{array}\right.$    
$p$ параметртэй $\dfrac{x-1}{2x-p}>1$ тэнцэтгэл бишийн шийд нь $p<\fbox{a}$ үед $x\in\Big(p-\fbox{b},\dfrac{p}{\fbox{c}}\Big)$, $p=\fbox{a}$ үед шийдгүй, $p>\fbox{a}$ үед $x\in\Big(\dfrac{p}{\fbox{d}},p-\fbox{e}\Big)$

Рационал тэнцэтгэл биш

$5x+7>3x+20$ тэнцэтгэл бишийн хамгийн бага бүхэл шийдийг ол.
8.2
$x-\dfrac{1-x}{6}\le \dfrac{2x+1}{2}-\dfrac34$ тэнцэтгэл бишийн хамгийн их бүхэл шийдийг ол.
$(x-2)^2+3>(x+5)^2$ тэнцэтгэл бишийн хамгийн их бүхэл шийдийг ол.
$2x^2+1\le x(x+2)$
$(x^2+1)(x^2+x+1)^3(x+1)^5>0$ тэнцэтгэл бишийн хамгийн бага бүхэл шийдийг ол.
$(x^3-1)(x^4-16)< 0$ тэнцэтгэл биш бод.
$\dfrac{5(x^3 + 6x^2 + 12x + 8)}{(x - 1)^2(x + 8)} \ge \dfrac{x(x + 2)^3}{(x^2 - 2x + 1)(x + 8)}$ тэнцэтгэл бишийн $[-10;12]$ муж дахь бүхэл шийдийн тоог ол.
Дурын $x$-ийн хувьд $\sin x+\sin (x+\alpha)+\sin (x+\beta)=0$ нөхцөл биелдэг байх $\alpha$, $\beta$-өнцгийг ол. Үүнд $0^{\circ}< \alpha < 180^{\circ}< \beta< 360^{\circ}.$
$-3< x< 5$, $-1< y< 4$ бол дараах илэрхийллүүдийн утгын мужийг ол.
  1. $x-1$
  2. $2x$
  3. $-y$
  4. $x+y$
  5. $2x-3y$
$-1< x< 2$, $1< y< 3$ бол дараах илэрхийллүүдийн утгын мужийг ол.
  1. $x+3$
  2. $-2y$
  3. $-\dfrac{x}{5}$
  4. $5x-3y$
$x^2-9x+14\le 0$ тэнцэтгэл бишийн хамгийн их бүхэл шийдийг ол.

A. $9$     B. $7$     C. $5$     D. $2$     E. $0$    
$x^2+8x<20$ тэнцэтгэл бишийн бүхэл шийдийн тоог ол.

A. $5$     B. $10$     C. $11$     D. $15$     E. $17$    
$x^2+2x-5\ge x+7$

A. $[3;+\infty)$     B. $(-\infty;-4[\cup]3;+\infty)$     C. $\{-4;3\}$     D. $[-4;3]$     E. $(-\infty;-4]\cup[3;+\infty)$    
$\dfrac1x>\dfrac15$ тэнцэтгэл бишийг бод.

A. $(-\infty;5)$     B. $(0;5)$     C. $(5;+\infty)$     D. $(-5;0)$     E. $(-\infty;-5)$    
$\dfrac{2x+1}{2+x}\ge 2$ тэнцэтгэл бишийг бод.

A. $(-\infty;-2)$     B. $(-\infty;-2]$     C. $(-2;+\infty)$     D. $\big(-2;-\frac12\big)$     E. $\varnothing$    
$-2x\le 0$ тэнцэтгэл бишийг бод.

A. $(-\infty;0)$     B. $[0;+\infty)$     C. $(-\infty;+\infty)$     D. $(-\infty;2)$     E. $(2;+\infty)$    
$x^2+8x<20$ тэнцэтгэл бишийн бүхэл тоон шийдийн тоог ол.

A. $10$     B. $11$     C. $12$     D. $13$     E. Бүхэл шийд байхгүй    
$\dfrac{5x+1}{4}-\dfrac{2-3x}{3}< \dfrac{x}6+1$ тэнцэтгэл бишийг бод.

A. $x< \dfrac{17}{25}$     B. $x>\dfrac{17}{25}$     C. $x< 0$     D. $x< 1$     E. $x>-\dfrac{14}{25}$    
$(x-2)^2\le 0$ бол

A. $x>2$     B. $x\le 2$     C. $x>2$     D. $x=2$     E. $x\ge 2$    
$(x^2-16)(x+4)(x-5)\le 0$ тэнцэтгэл бишийн бүх бүхэл тоон шийдүүдийн нийлбэрийг ол.

A. $0$     B. $1$     C. $4$     D. $5$     E. $9$    
$\displaystyle\frac{2-3x}{x+1}\ge-3$ тэнцэтгэл бишийг бод.

A. $x\ge-1$     B. $-1< x< 1.5$     C. $x<-1$     D. $x>-1$     E. $x\le -1$    
Хэрэв $x^2-3x-4< 0$ бол шийдүүдийн олонлог нь:

A. $-4< x<1$     B. $-4< x<-3$     C. $-3< x<0$     D. $-1< x<0$     E. $-1< x<4$    
$-2(x+5)<-4$ тэнцэтгэл бишийн шийд нь

A. $x>-3$     B. $x<-3$     C. $x>3$     D. $x<3$     E. $x>7$    
$0.1< \dfrac1{10}(x+10)< 0.2$ тэнцэтгэл биш биелэх завсрын дунджийг ол.

A. $-9.4$     B. $-9.5$     C. $-8.9$     D. $-8.5$     E. $-8.3$    
$-2< \dfrac15(x+7)< 1$ тэнцэтгэл биш биелэх завсрын дунджийг ол.

A. $-9.4$     B. $-9.3$     C. $-9.5$     D. $-9$     E. $-10$    
$\dfrac{(x-3)(2x-3)}{(x+5)}\le 0$ тэнцэтгэл бишийг бод.

A. $]-\infty;-5]\cup\big[\frac32;3]$     B. $]-\infty;-5[\cup\big[\frac32;3\big]$     C. $\big]-5;\frac32\big]\cup[3;+\infty]$     D. $\big[-5;\frac32\big]\cup[3;+\infty]$     E. Шийдгүй    
$\dfrac{3}{8}x>(1.125+3)\cdot\dfrac4{11}$ тэнцэтгэл бишийг бод.

A. $x>4$     B. $x\le 4$     C. $x>4.5$     D. $x<4.5$     E. $x\ge4.5$    
$-2x\geq 0$ тэнцэтгэл бишийг бод.

A. $(-\infty;0]$     B. $[0;+\infty)$     C. $[2;+\infty)$     D. $(-\infty;2)$     E. $(2;+\infty)$    
$\dfrac{5}{(5^x+2)(4x-17)}\leq 0$ тэнцэтгэл бишийн хамгийн их бүхэл шийдийг ол.

A. $5$     B. $3$     C. $1$     D. $0$     E. $4$    
$\dfrac{x^3+3x^2}{1+x}\le 0$ тэнцэтгэл бишийг бод.

A. $[-3;-1[\cup[0;+\infty[$     B. $[-\infty;-1[\cup[0;+\infty[$     C. $[-\infty;-2[\cup[0;+\infty[$     D. $[-3;-1[$     E. $[-3;-1[\cup\{0\}$    
$\dfrac{2}{x}\ge 1$ тэнцэтгэл биш бод.

A. $(-\infty;2)$     B. $(0;2]$     C. $[2;+\infty)$     D. $(-\infty;0)\cup[2;+\infty)$     E. $(0;2)$    
$\dfrac{1}{x}\ge 1$ тэнцэтгэл бишийн шийд аль нь вэ?

A. $]-\infty;1]$     B. $]-\infty;1[$     C. $]0;1]$     D. $]0;1[$     E. $]-\infty;-1]$    
$y=-x^2-2x+3$ функцийн график өгөгдөв. Уг функцийн эерэг байх завсар аль вэ?

A. $]0;+\infty[$     B. $]0;4[$     C. $]-1;3[$     D. $]-\infty;-3[\cup]1;+\infty[$     E. $]-3;1[$    
$\dfrac{(x^2+x+1)\cdot(x+10)^2}{(x-3)^2+1}\le 0$ тэнцэтгэл бишийн шийд аль нь вэ?

A. $]-\infty;+\infty[$     B. шийдгүй     C. $[3;10]$     D. $x=3$     E. $x=-10$    
$\dfrac{1}{x}\le 1$ тэнцэтгэл биш бод.

A. $(-\infty;0)$     B. $(0;1]$     C. $[1;+\infty)$     D. $(-\infty;0)\cup[1;+\infty)$     E. $(0;1)$    
$\dfrac{2}{x-1}>\dfrac{1}{7}$ тэнцэтгэл бишийн шийдийн олонлогийг ол.

A. $]-1;15[$     B. $]1;15[$     C. $]-15;1[$     D. $]-\infty;15[$     E. $]-1;1[$    
$\dfrac{(x^2+x+1)\cdot(x-10)^2}{(x+3)^2+1}>0$ тэнцэтгэл бишийн шийдийг дугуйл.

A. $x=10$     B. $x=3$     C. $]-\infty;+\infty[$     D. $]-\infty;10[\cup]10;+\infty[$     E. $[3;10]$    
$\dfrac{(2-x)^2}{x+3}\ge 0$ тэнцэтгэл бишийн хамгийн бага бүхэл шийдийг ол.

A. $0$     B. $-1$     C. $-2$     D. $1$     E. $2$    
$\dfrac{1}{x}-\dfrac{1}{x-1}<1$ тэнцэтгэл бишийн шийд аль нь вэ?

A. $]0;1[$     B. $]-\infty;0[\cup]1;+\infty[$     C. $]-1;1[$     D. $]0;+\infty[$     E. $\varnothing$    
$\dfrac{6}{x}<1$ тэнцэтгэл бишийн шийдийн олонлог аль завсар нь вэ?

A. $(-\infty;0)$     B. $(6;+\infty)$     C. $(-\infty;0)\cup(6;+\infty)$     D. $(0;6)$     E. $\varnothing$    
$\dfrac{(x-2)(x+3)}{1+x}\ge 0$ тэнцэтгэл бишийг бод.

A. $[-3;-1[\cup[2;+\infty[$     B. $[2;+\infty[$     C. $]-\infty;-3]\cup]-1;2]$     D. $[3;+\infty[$     E. $]-1;2]$    
$\dfrac{3}{1+|x+3|}<1$ тэнцэтгэл бишийг бод.

A. $]-1;-5[$     B. $\varnothing$     C. $]-\infty;5[$     D. $]1;5[$     E. $]-\infty;-5[\cup]-1;+\infty[$    
$\dfrac{2}{x} < \dfrac{1}{x - 6} + 1$ тэнцэтгэл бишийн хамгийн бага эерэг бүхэл шийдийг ол.

A. $2$     B. $3$     C. $5$     D. $6$     E. $7$    
$\dfrac{(x-2)(x+3)}{1+x}\ge 0$ тэнцэтгэл бишийг бод.

A. $[-3;-1[\cup[2;+\infty[$     B. $[2;+\infty[$     C. $]-\infty;-3]\cup]-1;2]$     D. $[3;+\infty[$     E. $]-1;2]$    
$\dfrac{3}{x-2}\ge 1$ тэнцэтгэл биш бод.

A. $(-\infty; 5)$     B. $(2;5]$     C. $(0,2)$     D. $(-5;5]$     E. $[5;+\infty)$    
$\dfrac{2x-1}{2-x}\ge 2$ тэнцэтгэл бишийг бод.

A. $(-\infty;2)$     B. $(-\infty;2]$     C. $[1.25;2)$     D. $[1.25;+\infty)$     E. $\varnothing$    
$\displaystyle\frac{2-3x}{x+1}\le-3$ тэнцэтгэл бишийг бод.

A. $x\ge-1$     B. $-1< x< 1.5$     C. $x<-1$     D. $x>-1$     E. $x\le -1$    
  1. $\frac{2x+1}{3}-\frac{3x-1}{2}>1$
  2. $\frac{9x+2}{10}-\frac{10x-2}{9}\ge2$
  3. $\frac{8x+4}{11}-\frac{9x-5}{10}>1$
  4. $\frac{7x+3}{8}-\frac{6x-2}{5}\ge1.5$
  5. $\frac{7x+1}{9}-\frac{4x-5}{5}>1$
  6. $\frac{3x-5}{4}-\frac{5x-8}{6}\ge\frac16$
  7. $\frac{5x-2}{8}-\frac{3x-1}{4}\ge-\frac{2}{3}$
  8. $\frac{4x-3}{3}-\frac{8x-2}{5}\ge-\frac{8}{7}$
  9. $\frac{6x-2}{7}-\frac{7x-2}{8}>-\frac{4}{3}$
  10. $\frac{5x-1}{4}-\frac{8x-3}{5}\ge-\frac{3}{2}$

A. 1     B. 3     C. 4     D. 2     E. -1    
  1. $\frac{6x-5}{4x+1}<0$
  2. $\frac{2x-3}{x+1}\le0$
  3. $\frac{2-3x}{2x+5}>0$
  4. $\frac{7x-12}{1-6x}\ge0$
  5. $\frac{4x+3}{2-0.5x}\>0$
  6. $\frac{7x-15}{3x+3}\le0$

$y=ax+b , 1\leq x\leq 3, a>0$ функцийн хувьд $3 \leq y\leq 5$ бол $a=\fbox{a}, b=\fbox{b}$ байна.
$y=ax+b , -2\leq x\leq 6, a< 0$ функцийн хувьд $-9 \leq y\leq 7$ бол $a=-\fbox{a}, b=\fbox{b}$ байна.
$y=ax+a^2 , 1\leq x\leq 3 (a>0)$ функцийн хувьд $6\leq y\leq 10$ бол $a=\fbox{a}$ байна.
$y=ax+a^2+1 , -2\leq x\leq 1 (a< 0)$ функцийн хувьд $1\leq y\leq 4$ бол $a=-\fbox{a}$ байна.
$ax^2+bx+1< 0$ тэнцэтгэл бишийн шийд $x\in]-\infty;-\dfrac13[\cup ]1;+\infty[$ бол $a=\fbox{ab}, b=\fbox{c}$ байна. Энэ үед $bx^2+ax+1< 0$ тэнцэтгэл бишийн шийд $]\dfrac1{\fbox{d}};\fbox{e}[$ байна.
$ax^2+bx+3< 0$ тэнцэтгэл бишийн шийд $x\in]-\infty;-1[\cup ]2;+\infty[$ бол $a=\dfrac{\fbox{ab}}2, b=\dfrac{\fbox{c}}2$ байна. Энэ үед $bx^2+ax-3< 0$ тэнцэтгэл бишийн шийд $]\fbox{de};\fbox{f}[$ байна.
$f(x)=3x^3-6x^2-5ax+18$ олон гишүүнтийн нэг язгуур нь $x_1=3$ бол $a=\fbox{a}$ байна. Иймд $3x^3-6x^2-5ax+18=3(x-3)(x^2+x+\fbox{bc})$ үржигдэхүүнд задрах ба $f(x)>0$ тэнцэтгэл бишийн шийд $]\fbox{de};\fbox{f}[\cup]\fbox{g};+\infty[$ байна.
$f(x)=2x^3-6x^2+2ax+30$ олон гишүүнтийн нэг язгуур нь $x_1=1$ бол $a=-\fbox{ab}$ байна. Иймд $2x^3-6x^2+(a+4)x-a=2(x-1)(x^2-\fbox{c}x+\fbox{de})$ үржигдэхүүнд задрах ба $f(x)< 0$ тэнцэтгэл бишийн шийд $]-\infty;\fbox{fg}[\cup]\fbox{h};\fbox{i}[$ байна.
$2x^3-ax^2+bx+c< 0$ тэнцэтгэл бишийн шийд $x\in ]-\infty;-1[\cup]1;2[$ бол $a$, $b$, $c$ -ийн хувьд $$\left\{ \begin{array}{l} -a-b+c=\fbox{a} \\ -a+b+c=\fbox{bc}\\ -4a+2b+c=-\fbox{de} \end{array} \right.$$ тэгшитгэлийн систем гарах ба $a=\fbox{f}$, $b=\fbox{gh}$, $c=\fbox{i}$ байна.
$x^3-ax^2-bx+c>0$ тэнцэтгэл бишийн шийд $x\in]-2;1[ \cup]3;+\infty[$ бол $a,b,c$ -ийн хувьд $$\left\{ \begin{array}{l} -4a+2b+c=\fbox{a} \\ a+b-c=\fbox{b}\\ 9a+3b-c=\fbox{cd}\\ \end{array} \right.$$ тэгшитгэлийн систем гарах ба $a=\fbox{e}, b=\fbox{f}, c=\fbox{g}$ байна.
$f(x)=9x^2+2(a-13)x+16$ парабол абсцисс тэнхлэгийг $А$, $C$ хоёр цэгээр огтолдог бол $AC=\dfrac{2}{\fbox{a}}\sqrt{(a-\fbox{bc})^2-\fbox{def}}$ байна. $AC\geq 2$ бол $a\leq \fbox{gh} \mbox{эсвэл} \fbox{ij}\leq a$ байна.
$f(x)=4x^2+2(a-21)x+9$ парабол абсцисс тэнхлэгийг $А$, $C$ хоёр цэгээр огтолдог бол $AC=\dfrac{1}{\fbox{a}}\sqrt{(a-\fbox{bc})^2-\fbox{de}}$ байна. $AC\geq 4$ бол $a\leq \fbox{fg} \mbox{эсвэл} \fbox{hi}\leq a$ байна.
$y=ax^2-2(2a-1)x+3a-1$ парабол $a$-ийн ямарч утганд $(\fbox{a},\fbox{b})$ ба $(\fbox{c},\fbox{d})$ цэгүүдийг дайрна.$(\fbox{a}< \fbox{c})$ Мөн $$\dfrac{\fbox{e}-\sqrt {\fbox{f}}}{2}< a< \dfrac{\fbox{e}+\sqrt {\fbox{f}}}{2}$$ бол абсцисс тэнхлэгтэй огтлолцохгүй.
$y=ax^2+(3-5a)x+4a+2$ парабол $a$-ийн ямарч утганд $(\fbox{a},\fbox{b})$ ба $(\fbox{c},\fbox{de})$ цэгүүдийг дайрна.$(\fbox{a}< \fbox{c})$ Мөн $$\dfrac{19-2 \sqrt {\fbox{fg}}}{9}< a< \dfrac{19+2\sqrt {\fbox{fg}}}{9}$$ бол абсцисс тэнхлэгтэй огтлолцохгүй.
$y=2x^2-8x+5 , 0\leq x\leq 3$ функцийн хувьд $-\fbox{a}\leq y \leq \fbox{b};$ $ y=2x^2+3x+1 , -1 \leq x < \dfrac12$ функцийн хувьд $ -\dfrac1 {\fbox{c}}\leq y < \fbox{d}$ байна.
$y=-x^2-2x+2,-3\leq x\leq -2$ функцийн хувьд $-\fbox{a}\leq y \leq \fbox{b}$ ; $y=- \dfrac 12 x^2+2x+ \dfrac 32, 1\leq x\leq 5$ функцийн хувьд $ - \fbox{c}\leq y \leq \dfrac{\fbox{d}}2 $ байна.
$y=ax^2-(a+1)x-a-3$ функцийн абсцисс тэнхлэгийг огтлох 2 цэг нь $-1< x< 0, 1< x< 2$ интервалуудад харгалздаг бол $a< \fbox{ab}, \fbox{c}< a$ байна.
$y=ax^2-(a-5)x+3a-15$ функцийн абсцисс тэнхлэгийг огтлох 2 цэг нь $-5< x< 0, 1< x< 2$ интервалуудад харгалздаг бол $\dfrac{\fbox{ab}}{38}< a< \dfrac{\fbox{c}}2$ байна.
$(x^2-4x+1)^2-3(x^2-4x+1)+2\leq 0$ тэнцэтгэл бишийн шийд $[\fbox{a}-\sqrt{\fbox{b}}; \fbox{c}]\cup[\fbox{d};\fbox{a}+\sqrt{\fbox{b}}] $ байна.
$(x^2+2x-8)^2-2(x^2+2x-8)-35\leq 0$ тэнцэтгэл бишийн шийд $[-\fbox{a}; -\fbox{b}]\cup[\fbox{c};\fbox{d}] $ байна.
$\dfrac{1}{x^2+8x-9}\ge\dfrac{1}{3x^2-5x+2}$ тэнцэтгэл бишийг бодьё.

$x^2+8x-9=(x-\fbox{a})(x+\fbox{b})$ ба $3x^2-5x+2=(x-\fbox{a})(\fbox{c}x-\fbox{d})$ тул $$\dfrac{1}{x-\fbox{a}}\cdot\left(\dfrac{1}{x+\fbox{b}}-\dfrac{1}{\fbox{c}x-\fbox{d}}\right)=\dfrac{1}{x-\fbox{a}}\cdot\dfrac{\fbox{e}x-\fbox{fg}}{(x+\fbox{b})(\fbox{c}x-\fbox{d})}\ge 0$$ болно. Иймд тэнцэтгэл бишийн шийд нь $$x\in\bigg]-\infty;-\fbox{h}\bigg[\cup\bigg]\dfrac{\fbox{i}}{3};\fbox{j}\bigg[\cup\bigg[\dfrac{\fbox{kl}}{2};+\infty\bigg[$$ байна.

Систем тэнцэтгэл биш

${\left\{ {{\begin{array}{c} {2x + 10 < 1.5x + 20}\\ {3x + 4 < 2x + 16} \end{array}} } \right.}$ хамгийн их бүхэл шийдийг ол.
$${\left\{ {{\begin{array}{c} {{\frac{{x}}{{8}}} - {\frac{{5x - 4}}{{12}}} < {\frac{{x - 2}}{{6}}} - {\frac{{x + 1}}{{3}}} - {\frac{{3x}}{{4}}} + 6} \\ {x - {\frac{{x - 1}}{{2}}} - {\frac{{x + 2}}{{3}}} > {\frac{{x - 3}}{{4}}}} \end{array}} } \right.}$$
$\left\{\begin{array}{c} x^{2} - 9x + 14 < 0\\ x - 4 < 0 \end{array}\right.$ бүхэл шийдүүдийг ол.
$\left\{\begin{array}{c} x^2 + 6x + 5 < 0\\ x^2 + 4x + 3 > 0 \end{array}\right.$ бүхэл шийдүүдийг ол.
$\left\{\begin{array}{c} x\left( {x + 5} \right) > 6\\ 1 - \dfrac{x}{3} > 0.1 - 0.25x \end{array}\right.$
$\left\{\begin{array}{c} 2x^{2} - 10x + 5 < 0\\ x^{2} + 3x - 2 < 0 \end{array}\right.$
$\left\{\begin{array}{c} x^{2} - 5x - 6 < 0\\ x^{2} - 3x > 0 \end{array}\right.$
${\left\{ {{\begin{array}{c} {{\frac{{2x - 14}}{{x2 - x - 12}}} \le 1} \hfill \\ {1.5 < x < 2.5} \hfill \\ \end{array}} } \right.}$ хамгийн их ба хамгийн бага утгуудын нийлбэрийг ол.
${\left\{ {{\begin{array}{c} {1 \ge {\frac{{x2 + 4x + 8}}{{\left( {x + 2} \right)\left( {x + 3} \right)}}}} \hfill \\ { - 2.5 \le x \le 3.5} \hfill \\ \end{array}} } \right.}$ хамгийн их ба хамгийн бага утгуудын нийлбэрийг ол.
$1 \le \dfrac{x + 1}{2 - x} < 3$ хамгийн бага бүхэл шийдийг ол.
${\left\{ {{\begin{array}{c} {{\left| {x} \right|} \ge 1} \hfill \\ {{\left| {x - 1} \right|} < 3} \hfill \\ \end{array}} } \right.}$
${\left\{ {{\begin{array}{c} {9^{x + 0.5} - 10 \cdot 3^{x} + 3 \le 0} \hfill \\ {x \ge - 0.5} \hfill \\ \end{array}} } \right.}$ мужийн уртыг ол.
${\left\{ {{\begin{array}{c} {3^{x + 1} + 18 \cdot 3^{ - x} > 29} \hfill \\ {1 \le x \le 12} \hfill \\ \end{array}} } \right.}$ мужийн уртыг ол.
${\left\{ {{\begin{array}{c} {{\frac{{x^{2} - 13x + 40}}{{x^{2} - x - 6}}} \ge 0} \hfill \\ {\log _{2} \left( {x + 2} \right) \le 3} \hfill \\ \end{array}} } \right.}$ хамгийн их бүхэл шийдийг ол.
${\left\{ {{\begin{array}{c} {2^{\log _{{\frac{{1}}{{2}}}} x} \le 3} \hfill \\ {{\frac{{x^{2} + x - 12}}{{x^{2} - 6x + 8}}} \ge 0} \hfill \\ \end{array}} } \right.}$ хамгийн бага бүхэл шийдийг ол.
${\left\{ {{\begin{array}{c} {\log _{0.5} {\frac{{x^{2} - 2x}}{{x - 3}}} < 0} \hfill \\ {{\frac{{x - 7}}{{x + 5}}} \ge 0} \hfill \\ \end{array}} } \right.}$ хамгийн бага шийдийг ол.
${\left\{ {{\begin{array}{c} {\log _{{\frac{{1}}{{2}}}} \left( {2x - 3} \right) > - 3} \hfill \\ {x^{2} - 4x > 0} \hfill \\ \end{array}} } \right.}$ б?хэл шийдүүдийг ол.
${\left\{\begin{array}{c} \dfrac{{1}}{2 - x} \ge 1\\ 2 \cdot 4^{2x} \ge 32^{x} \end{array}\right.}$
${\left\{ {{\begin{array}{c} {\left( {x - 2} \right) \cdot \sqrt {x^{2} - 5.5x + 6} \ge 0} \hfill \\ {\left( {x + 1} \right) \cdot \sqrt {x^{2} + 0.5x - 3} \le 0} \hfill \\ \end{array}} } \right.}$
$\left\{\begin{array}{c} {2x^{2}+2y^{2}-12x+20y+65< 0}\\ {4x+2y>3} \end{array}\right.$ бүх бүхэл шийдийг ол.
$\left\{\begin{array}{c} m^{2}+n^{2}< 16m-22n-171\\ 30m-n^{2}> 252+14n+m^{2} \end{array}\right.$ бүх бүхэл шийдийг ол.
$\left\{\begin{array}{c} p^{2}+q^{2}< 18p-20q-166\\ 32p-q^{2}> p^{2}+12q+271 \end{array}\right.$ бүх бүхэл шийдийг ол.
$\left\{\begin{array}{c} \log_{\sqrt{2}}\left(x-1\right)< 4\hfill\\ \dfrac{x}{x-3}+\dfrac{x-5}{x}< \dfrac{2x}{x-3} \end{array}\right.$ тэнцэтгэл бишийн системийг бод.
$\left\{\begin{array}{c} \log_{2-x}\left({2-y}\right)> 0\\ \log_{4-y}\left({2x-2}\right)> 0 \end{array}\right.$ тэнцэтгэл бишийн системийг бод.
$\left\{\begin{array}{c} \log_{x-1}\left({5-y}\right)< 0\\ \log_{2-y}\left({4-x}\right)< 0 \end{array}\right.$ тэнцэтгэл бишийн системийг бод.
$\left\{\begin{array}{c} 4^{x+y-1}+3\cdot 4^{2y-1}\le 2\\ x+3y\ge 2-\log_{4}3 \end{array}\right.$ тэнцэтгэл бишийн системийг бод.
$\left\{\begin{array}{c} 3^{x+2y-1}+2\cdot 3^{3y-1}\le 2\\ x+5y\ge 2-\log_{3}2 \end{array}\right.$ тэнцэтгэл бишийн системийг бод.
$2^{x^{2}-x-1}=0.5\cdot 8^{2x-4}$ тэгшитгэлийн $\log_{1.1}\left(\log_{1.3}\dfrac{2x-1}{x+1}\right)>0$ нөхцлийг хангах шийдүүдийг ол.
$\left\{\begin{array}{c} 2^{-x}\cdot y^{4}-2y^{2}+2^{x}\le 0\\ 8^{x}-y^{4}+2^{x}-1=0 \end{array}\right.$ тэнцэтгэл бишийн системийг бод.
$\min{\left\{{1-x^{2},{\dfrac{{1+x}}{{2}}}}\right\}}\ge 0.5$ байх $x$-н бүх утгыг ол. Хариунд ${\dfrac{{a+b}}{{\sqrt{2}}}}$ тоог бичээрэй! Энд $a$, $b$ нь шийдийн мужийн хил.
$3x-4$ ба $\log _{2}\left({5\cdot 2^{2x-4}-1^{x-1}+1}\right)$ тоонуудын их нь эерэг тоо байх $x$-ийн утгуудыг ол.
$3x+5$ ба $\log _{2}\left({5\cdot 2^{2x+2}-2^{x+2}+1}\right)$ тоонуудын бага нь сөрөг байх $x$-ийн бүх утгыг ол.
$\left\{\begin{array}{*{20}c} {2\cdot{\left|{x+2}\right|}\cdot\arcsin\left({y-1}\right)^{2} \le\pi\left({x+2}\right)}\hfill\\ {2\cdot{\left|{y-1}\right|}-x\ge 0}\hfill\\ \end{array}\right.$ системийн шийдийн мужийн периметрийг ол.
$\left\{\begin{array}{c} y-x\le 5\\ y+4x\le-5\\ 3y+2x\ge-5 \end{array}\right.$ системийг хангах $x$, $y$-н хувьд $-4\le x\le-1$ байхыг батлаад
  1. $x^2+y^2$
  2. $\dfrac{y}{x}$
илэрхийллүүдийн авч болох утгын мужийг ол.
Дараах тэнцэтгэл бишийн системийг бод.
  1. $\left\{\begin{array}{c} 5x+1\le 8(x+2)\\ 2x-3< 1-(x-5) \end{array}\right.$
  2. $\left[\begin{array}{c} x+7< 1-2x\\ 6x+2\ge 2 \end{array}\right.$
  3. $-2x+1<3x+4<2(3x-4)$
Дараах тэнцэтгэл бишийн системийг бод.
  1. $\left\{\begin{array}{c} 2(1-x)> -6-x\\ 2x-3> -9 \end{array}\right.$
  2. $\left[\begin{array}{c} 3(x-4)\le x-3\\ 6x-2(x+1)< 10 \end{array}\right.$
  3. $x+9\le 3-5x\le 2(x-2)$
8.2
Дараах давхар тэнцэтгэл бишийг бод. $-2x+1<3x+4<2(3x-4)$
$\left\{\begin{array}{c}a(x-2)\ge x-3\\8(a+1)x\ge 8ax+9\end{array}\right.$ системийг боджээ. Хэрвээ
  1. $a\ge\dfrac{15}{7}$ үед $x\ge2+\dfrac{1}{1-a}$
  2. $\dfrac{15}{7}>a>1$ үед $x\ge1\dfrac18$
  3. $a=1$ үед $x\ge\dfrac98$
  4. $a< 1$ үед $\dfrac{9}{8}\le x\le\dfrac{2a-3}{a-1}$
гэсэн дөрвөн өгүүлбэр өгөгдсөн бол анхны системийн хариуг заа.

A. 1, 2 ба 3     B. 1, 2 ба 4     C. 1, 2, 3 ба 4     D. 2, 3 ба 4     E. 1 ба 3    
$\left\{\begin{array}{c}a(x-2)\ge x-3\\8(x-1)\ge1\end{array}\right.$ системийг боджээ. Хэрвээ
  1. $a\ge15/7$ үед $x\ge\dfrac{2a-3}{a-1}$
  2. $15/7>a>1$ үед $x\ge1\dfrac18$
  3. $a=1$ үед $x\ge 9/8$
  4. $a< 1$ үед $9/8\le x\le \dfrac{2a-3}{a-1}$
гэсэн дөрвөн өгүүлбэр өгөгдсөн бол анхны системийн хариуг заа.

A. 1, 2 ба 3     B. 1, 2 ба 4     C. 1, 2, 3 ба 4     D. 2, 3 ба 4     E. 1 ба 3    
$8-2x \leq x+2 \leq 10$ тэнцэтгэл бишийг хангах бүхэл тоонуудын нийлбэрийг ол.

A. $35$     B. $40$     C. $38$     D. $36$     E. $30$    
$1 \le \dfrac{2-x}{x+1} \le 2$ тэнцэтгэл бишийг бод.

A. $(-\infty;0]$     B. $\left[-\dfrac{1}{3};0\right]$     C. $\left[0;\dfrac{1}{2}\right]$     D. $\left[-\dfrac{1}{2};\dfrac{1}{2}\right]$     E. $[0;+\infty[$    
$\left\{\begin{array}{l} 2-x< x-4< 3-x \\ 3-x< x-2< 5-x \end{array}\right.$ тэнцэтгэл бишийн системийг бод.

A. $]\frac52;\frac72[$     B. $]3;\frac72[$     C. $[3;3.5]$     D. $]3;4[$     E. $\varnothing$    
$\left\{\begin{array}{l} 5+x< 2x-3< 12+x \\ 4+x< 3x-2< 16+x \end{array}\right.$ тэнцэтгэл бишийн системийг бод.

A. $]8;15[$     B. $]3;15[$     C. $]9;15[$     D. $]8;9[$    
$\left\{\begin{array}{l}|x-3|< 4 \\ 2-x< 3x-8< 4-x \end{array}\right.$ тэнцэтгэл бишийн системийг бод.

A. $]-1;7[$     B. $]\frac{5}2;7[$     C. $]\frac{5}2;3[$     D. $]-1;3[$    
$\left\{\begin{array}{l}|x-1|>1 \\ 5+x< 4x-3< 8+x \end{array}\right.$ тэнцэтгэл бишийн системийг бод.

A. $]-4;\frac{13}3[$     B. $]-2;\frac83[$     C. $]2;\frac{13}3$     D. $]\frac83;\frac{11}3[$    
$\left\{\begin{array}{l}|x|\le3 \\ 2x-3< 6x-5< 2x+3 \end{array}\right.$ тэнцэтгэл бишийн системийг бод.

A. $]1;2[$     B. $[-3;3]$     C. $]\frac12;2[$     D. $]\frac12;\frac32[$    
$\left\{\begin{array}{l}|x-1|\le4 \\ 23-2x< 7x+5< 60-2x \end{array}\right.$ тэнцэтгэл бишийн системийг бод.

A. $]-3;5]$     B. $]2;\frac{55}9[$     C. $[-3;\frac{55}9[$     D. $]2;5]$    
$\left\{\begin{array}{l}2(3x-1)< 3(4x+1)+16 \\ 4(2+x)< 3x+8 \end{array}\right.$ системийг хангах хамгийн бага бүхэл $x$-ийг ол.

A. $3$     B. $0$     C. $-1$     D. $-3$     E. $-4$    
$\left\{\begin{array}{l}0.5(2x-5)>\frac{2-x}2+1 \\ 0.2(3x-2)+3>\frac{4x}3-0.5(x-1) \end{array}\right.$ системийг хангах хамгийн их бүхэл $x$-ийг ол.

A. $10$     B. $9$     C. $8$     D. $7$     E. $6$    
$\left\{\begin{array}{l}|x^2-4x|< 5 \\ |x+1|< 3 \end{array}\right.$ системийг бод.

A. $]-2;2[$     B. $]-2;-1[$     C. $]-1;1[$     D. $]-1;2[$     E. $]0;1[$    
$\left\{\begin{array}{l}|x^2+5x|< 6 \\ |x+1|\le2 \end{array}\right.$ системийг бод.

A. $]-2;-1[$     B. $]-2;0[$     C. $]-3;1[$     D. $]-2;1[$     E. $]-2;2[$    
$\left\{\begin{array}{l} \frac x3-\frac43\le\frac4x \\ \frac1x>-1 \\ x^2+3x+1>0 \end{array}\right.$ системийг бод.

A. $]-\infty;\frac{3+\sqrt5}2]\cup[0;7]$     B. $]-\infty;\frac{-3-\sqrt5}2[\cup]0;8[$     C. $]-\infty;\frac{-3-\sqrt5}2[\cup]0;6]$     D. $]-\infty;\frac{-3+\sqrt5}2[\cup]0;7[$     E. $]-\infty;\frac{-3-\sqrt5}2[\cup]0;7[$    
$\left\{\begin{array}{l} \frac1{3x}< 1\\ x+\frac43\ge\frac4{3x}\\ 9x^2-9x+1< 0 \end{array}\right.$ системийг бод.

A. $]-\frac32;\frac{3+\sqrt5}6[$     B. $[\frac{-3-\sqrt5}6;1[$     C. $]-;\frac{-3+\sqrt5}6[$     D. $[\frac23;\frac{3+\sqrt5}6[$    
$\left\{\begin{array}{l}\displaystyle\frac{5x+37}{12+x-x^2}>2 \\ \displaystyle\frac{(2x+9)(7x-2)}{(9+5x)^2}\le0 \end{array}\right.$ системийг бод.

A. $]-3;-1[\cup]-1;1[$     B. $]-3;-\frac95[\cup]-\frac95;\frac27]$     C. $]-3;0[\cup]0;\frac27[$     D. $]-3;-2[\cup]-2;1[$    
$\left\{\begin{array}{l}\displaystyle\frac{(2x-11)(3x+7)}{(9-4x)^2}\le0 \\ \displaystyle\frac{71-24x}{14-5x-x^2}< 5 \end{array}\right.$ системийг бод.

A. $]2;4[\cup]5;\frac{16}3[$     B. $]2;3[\cup]3;6[$     C. $[2;3]\cup[4;6]$     D. $]2;\frac94[\cup]\frac94;\frac{11}2[$     E. $]2;\frac94[\cup]\frac94;\frac{11}2]$    
$\left\{\begin{array}{l}(x-1)(x-3)\le8 \\ \displaystyle\frac{5x-2}{7x-2}\ge2 \end{array}\right.$ системийг бод.

A. $[-1;\frac27]$     B. $[-1;\frac27[$     C. $]-2;\frac27[$     D. $]-\frac27;1[$    
$\left\{\begin{array}{l}\displaystyle\frac{2x+23}{x+3}< 5 \\ (4-x)(x+6)\le16 \end{array}\right.$ системийг бод.

A. $]-\infty;+\infty[$     B. $]-\infty;-3[\cup]\frac38;+\infty[$     C. $]-\infty;-4[\cup]3;+\infty[$     D. $]-\infty;-4]\cup]\frac83;+\infty[$    
$\left\{\begin{array}{l}x^2+y^2\le9 \\ x+y\ge0 \end{array}\right.$ тэнцэтгэл бишийн системийг хангах хавтгайн цэгүүдийн олонлог аль вэ?

A. 1)     B. 2)     C. 3)     D. 4)     E. 5)    
$\left\{\begin{array}{l}(x^2+3x+1)(x^2+3x-3)\ge5 \\ (x^2-x-1)(x^2-x-7)< -5 \end{array}\right.$ тэнцэтгэл бишийн системийг бод.

A. $]-\infty;-4]\cup[-2;-1]\cup[1;\infty[$     B. $]-\infty;-4[\cup]-2;-1]$     C. $[-2;-1]\cup[1;\infty[$     D. $]-2;-1[\cup]2;3[$    
$\left\{\begin{array}{l}(x^2-4x+1)(x^2-4x+3)\le8 \\ (x^2-2x-1)(x^2-2x+2)\le-2 \end{array}\right.$ тэнцэтгэл бишийн системийг бод.

A. $[-2-\sqrt{5};-2+\sqrt5]$     B. $[0;2]$     C. $[-2+\sqrt5;2]$     D. $[0;-2+\sqrt{5}]$    
$\left\{\begin{array}{l}x^2-4< 0 \\x+1>0\\ \frac12-x>0 \end{array}\right.$ тэнцэтгэл бишийн системийг бод.

A. $[-\frac12;\frac23]$     B. $]-1;\frac12[$     C. $[0;\frac12]$     D. $[-1;1]$    
$\left\{\begin{array}{l}\displaystyle\frac1x>-4 \\ \displaystyle\frac{x^2-1}{4x-5}< 1 \\ \displaystyle\frac{x^2-x}{2x+3}>-2 \end{array}\right.$ тэнцэтгэл бишийн системийг бод.

A. $[-\frac12;-\frac23]\cup]0;\frac54[$     B. $[-1;-\frac12]\cup]0;\frac54[$     C. $]-\frac32;-\frac14[\cup]0;1[$     D. $]-\frac32;-\frac14[\cup]0;\frac54[$    
$\left\{\begin{array}{l}\displaystyle\dfrac{x^2-7x+12}{2x^2+4x+5}>0 \\ |x^2-3x|< 4 \end{array}\right.$ тэнцэтгэл бишийн системийг бод.

A. $]-\infty;3[$     B. $]4;+\infty[$     C. $]-1;3[$     D. $]-1;4[$    
$\left\{\begin{array}{l}\displaystyle\dfrac{x^2-9}{3x-x^2-24}< 0\\ |x^2-6|< 5x \end{array}\right.$ тэнцэтгэл бишийн системийг бод.

A. $]-\infty;-3[$     B. $]3;+\infty[$     C. $]-3;3[$     D. $]3;6[$    
$0 < \dfrac{3x-1}{2x+5} < 1 $ давхар тэнцэтгэл бишийг бод.

A. $]-\infty;\frac13[$     B. $]\frac13;6[$     C. $]\frac12;6[$     D. $[2;6[$     E. $]\frac13;+\infty[$    
$1\le\dfrac{2-x}{x+1}\le2$ давхар тэнцэтгэл бишийг бод.

A. $]-\infty;0]$     B. $]-\frac13;0]$     C. $[0;\frac12]$     D. $]-\frac12;\frac12]$     E. $]-\frac13;\frac12]$    
$\bigg\{\begin{array}{c}x^2-7x+10> 0\\2x-1\ge 0\end{array}$ тэнцэтгэл бишийн системийг бод.

A. $(2;5)$     B. $\Big[\frac12;5\Big)$     C. $\Big[\frac12;2\Big)\cup(5;+\infty)$     D. $\Big(\frac12;+\infty\Big)$     E. $\Big(\frac12;2\Big)$    
$\left\{ \begin{array}{l} \dfrac{x-5}{4}-\dfrac{2x-1}2<3\\ \dfrac{2x-3}3\leq\dfrac{x+1}2 \end{array} \right. $ тэнцэтгэл бишийн бүхэл шийдийн тоог ол.

A. $11$     B. $12$     C. $13$     D. $14$     E. $0$    
$\left\{ \begin{array}{c} 2x^2-3x > 2\\ x^2-2 \leq 2x \end{array} \right.$ тэнцэтгэл биш бод.

A. $(-\infty;-\frac12)\cup[2;+\infty)$     B. $[1-\sqrt3;1+\sqrt3)$     C. $[1-\sqrt3;-\frac12)\cup(2;3+\sqrt3]$     D. $[1-\sqrt3;-\frac12)\cup(2;1+\sqrt3]$     E. $[\frac12;\sqrt3-1)\cup(2;1+\sqrt3]$    
$\left\{ \begin{array}{rcl} 2x^2-5x&<&3\\ 3x^2-4x&\leq&11 \end{array} \right.$ тэнцэтгэл биш бод.

A. $(-\frac12;\frac{2+\sqrt{37}}3]$     B. $[-\frac12;\frac{2+\sqrt{37}}3)$     C. $(-\frac12;3)$     D. $[\frac{-2-\sqrt{37}}2;3]$     E. $(-\frac12;\frac{2+\sqrt{37}}3]$    
$\left\{\begin{array}{l}|x^2+5x|< 6 \\ |x+1|\le2 \end{array}\right.$ системийг бод.

A. $]-2;-1[$     B. $]-2;0[$     C. $]-3;1[$     D. $]-2;1[$     E. $]-2;2[$    
$\left\{\begin{array}{c}r+\sin sx\le 1\\ x^2+rx+1\le 0\end{array}\right.$ тэнцэтгэл бишийн систем яг нэг шийдтэй байх $r, s$-ийг олъё. $r\le \fbox{a}$ үед дурын бодит $x$-ийн хувьд $\sin sx\le 1-r$ байх тул $x^2+rx+1\le 0$ тэнцэтгэл биш яг нэг бодит шийдтэй байна. $r\le\fbox{a}$ гэдгийг тооцвол $r=\fbox{bc}$ байна. Энэ үед $s\in\mathbb R$ байна. Одоо $r>\fbox{a}$ үед бодъё. $x^2+rx+1\le 0$ шийдтэй тул $D\ge 0$ буюу $r\ge\fbox{d}$ байна. Энэ үед $\sin sx\le 1-r\le\fbox{ef}$ болох тул шийдтэй байхын тулд $\sin sx=\fbox{ef}$, $r=\fbox{d}$, $x=\fbox{gh}$ байна. Энэ үед $s=\dfrac{\pi}{\fbox{i}}+\fbox{j}\pi k, k\in\mathbb Z$ байна.
$\displaystyle x^2-x-2\le0\quad (1),\ x^2-px+q< 0\quad (2)$ тэнцэтгэл бишүүд өгөгдөв. Тэгвэл $(1)$-ийн шийд нь $\displaystyle-\fbox{a}\le x\le\fbox{b}$ байна. Мөн $(1)$ ба $(2)$ тэнцэтгэл бишийг зэрэг хангах шийд байхгүй ба тэдгээрийн шийдийн нэгдэл $\displaystyle -1\le x< 6$ бол $p=\fbox{c}$, $q=\fbox{de}$ байна.
$\left\{ \begin{array}{l} |x^2-4x|< 5 \\ |x+1|< 3 \\ \end{array} \right.$ тэнцэтгэл бишийн системийн шийдийн олонлог $]-\fbox{a};\fbox{b}[ $ байна.
$\left\{ \begin{array}{l} |x^2+5x|< 6 \\ |x+1|< 2 \\ \end{array} \right.$ тэнцэтгэл бишийн системийн шийдийн олонлог $]-\fbox{a};\fbox{b}[ $ байна.
$x^2-(a+1)x+a< 0 , 3x^2+2x-1>0$ тэнцэтгэл бишүүдийн шийдүүдийн огтлолцлийн олонлогт 3 бүхэл тоо ордог бол $-\fbox{a}\leq a< -\fbox{b} , \fbox{c}< a\leq \fbox{d}$ байна.
$x^2-2x-8>0 , x^2+(a-3)x-3a< 0$ тэнцэтгэл бишүүдийн шийдүүдийн огтлолцлийн олонлогт 5 бүхэл тоо ордог бол $-\fbox{a}\leq a< -\fbox{b} , \fbox{c}< a\leq \fbox{de}$ байна.

Тригонометрийн тэнцэтгэл биш

$4\cos x-\sin 2x>0$ тэнцэтгэл бишийг бод.
$3\sin x+\sin 2x< 0$ тэнцэтгэл бишийг бод.
$\sin 2x+\cos ^{2}2x>1+\sqrt{2}$ тэнцэтгэл бишийг бод.
$3\sin x>2\cos ^{2}x$ тэнцэтгэл бишийг бод.
$\cos ^{2}x-\sin^{2}x+\sqrt {3}\cdot\cos x-2 \le 0;$ $ |x|\le\dfrac{\pi}{4}$ тэнцэтгэл бишийг бод.
$\sqrt{2\sin x}< 1$ тэнцэтгэл бишийг бод.
$\sqrt{\sin x}>\sqrt{\cos x}$ тэнцэтгэл бишийг бод.
$\tg^{3}x+\tg^{2}x-\tg x-1< 0$ тэнцэтгэл бишийг бод.
$2\sin x-1\le\sqrt{6\sin ^{2}x-6\sin x-12}$ тэнцэтгэл бишийг бод.
$2\cos x-1\le\sqrt{8\cos ^{2}x-8\cos x-16}$ тэнцэтгэл бишийг бод.
$x^{2}\cdot\sin x+18> 2x^{2}+9\sin x$ бүхэл шийдийн тоог ол.
$\log _{\frac{1}{7}}\left({10-x^{2}}\right)\cdot\log _{\frac{1}{2}}{\left|{\sin x}\right|}>0$ тэнцэтгэл бишийг бод.
$\log_{\frac{1}{2}}{\left|{\cos x}\right|}\cdot\log_{5}\left({x^{2}-9}\right)< 0$ тэнцэтгэл бишийг бод.
$0,5^{2\sin\left({5x-30^{0}}\right)}>0,5^{p}; p =-2^{0,5}$ тэгшитгэлийн ${\left[{0^\circ; 90^\circ}\right]}$ завсар дахь бүх шийдийг ол.
$4^{\sin^{2}x}< \dfrac{12}{4^{\sin ^{2}x}-1}$ тэнцэтгэл бишийг бод.
$\big(-5+(\lg 103)\cosec x-\cosec^{2}x\big)\cdot\dfrac{\log_{\frac{1}{3}}\left({x^{2}+x}\right)}{\pi x-1}< 0$ тэнцэтгэл бишийг бод.
$\sqrt{4x-x^{2}-3}\cdot\left(\sqrt{2}\cos x-\sqrt{1+\cos 2x}\right)\ge 0$ тэнцэтгэл бишийг бод.
$\dfrac{1}{1-\ctg x}\le\sqrt{1+\ctg x}$ тэнцэтгэл бишийг бод.
$\dfrac{1}{\log _{6\cdot\sin t}{\dfrac{1}{6}}}-\dfrac{1}{\log _{6}{\dfrac{{\sin t}}{6}}}\le 0$ тэнцэтгэл биш биелэх $t$-н бүх эерэг утгыг ол.
$\dfrac{1}{\log _{3\cdot\cos u}3}+\dfrac{{1}}{\log _{3}\dfrac{\cos u}{3}}\ge 0$ тэнцэтгэл бишийг хангах $u$-н бүх сөрөг утгыг ол.
$\dfrac{\sqrt{3}+\cos x}{2\sin ^{2}x-\cos x\cdot \sin 2x}>\dfrac{3}{2\sin 4x}$ тэнцэтгэл бишийг бод.
$\dfrac{3}{\sin 4x}< \dfrac{2\sqrt{3}-\tg x}{4\cos ^{4}x-\sin ^{2}2x}$ тэнцэтгэл бишийг бод.
  1. $\sin^2\theta-\cos \theta+a=0, (0^{\circ}\leq \theta< 360^{\circ})$ тэгшитгэл шийдтэй байх $a$ параметрийн утгын мужийг ол.
  2. Дурын $\theta$-ийн хувьд $p=\cos^2\theta-2a\sin\theta+a^2-4\leq 0$ тэнцэл биелэх $a$ параметрийн утгын мужийг ол.
Дараах тэнцэтгэл бишийг бод.
  1. $\cos2\theta+2\sin \theta+\dfrac 12\geq -\sqrt{3}$, $0^{\circ}\leq\theta< 360^{\circ}$.
  2. $\sin 2\theta< \sin \theta.$
$0^{\circ}\leq \theta< 360^{\circ}$ бол дараах тэнцэтгэл бишийг бод.
  1. $2\sin\theta>1$
  2. $2\cos \theta\geq -\sqrt{3}$
  3. $\tg \theta\leq -\sqrt{3}$
Дараах тэнцэтгэл бишийг $0^{\circ}\leq \theta< 360^{\circ}$ ба $-180^{\circ}< \theta\leq 180^{\circ}$ мужуудад бод.
  1. $2\sin\theta\geq \sqrt{2}$
  2. $2\cos \theta< 1$
  3. $\sqrt{3}\tg \theta< 1$
Дараах тэнцэтгэл бишийг батал.
  1. $|\cos\alpha\cdot \cos \beta+\sin\alpha\cdot \sin\beta\cdot \cos\theta|\leq 1.$
  2. $|\cos\alpha\cdot \cos\beta+\sin\alpha\cdot \cos\gamma\cdot \sin\beta\cdot \cos\delta+ \sin\alpha\cdot \sin\gamma\cdot \sin\beta\sin\delta\cdot \cos \theta|\leq 1.$
Тэнцэтгэл бишийг бод.
  1. $\cos2x+5\sin x-3< 0$, $0^{\circ}\leq x\leq 180^{\circ};$
  2. $2\sin 2x-2\sqrt{2}\sin x-2\cos x+\sqrt{2}\geq 0$, $0^{\circ}\leq x< 360^{\circ}.$
Тэнцэтгэл бишийг бод.
  1. $2\sin (x+60^{\circ})+2\sin (x+120^{\circ})>\sqrt{3}$, $0^{\circ}\leq x< 360^{\circ};$
  2. $\cos x+2\cos x\cdot \cos 3x< 0$, $0^{\circ}< x< 180^{\circ};$
  3. $\cos 3\theta< \cos \theta$, $0^{\circ}\leq \theta< 360^{\circ}.$
Тэнцэтгэл бишийг бод. $\cos\theta+\cos 2\theta+\cos 3\theta< 0$, $0^{\circ}\leq \theta< 360^{\circ}.$
$\triangle ABC$-ийн хувьд $2\cos A\cdot \cos B\leq 1-\cos C$ тэнцэтгэл бишийг батал.
$\frac12\le\cos x< 1$ тэнцэтгэл бишийн $\big[-\frac\pi2; 0\big]$ завсар дахь шийдийн олонлог аль нь вэ?

A. $\big[-\frac\pi2;-\frac\pi3\big]$     B. $\big]-\frac\pi2;-\frac\pi4\big]$     C. $\big[-\frac\pi2;-\frac\pi6\big[$     D. $\big[-\frac\pi3;-\frac\pi6\big]$     E. $\big[-\frac\pi3; 0\big[$    
$\cos x+\sin 2x< 0$ тэнцэтгэл бишийн $0\le x< 2\pi$ байх шийдийг ол.

A. $\big]\frac{\pi}{2};\frac{7\pi}{6}\big[\cup\big]\frac{3\pi}{2};\frac{5\pi}{3}\big[$     B. $\big]\frac{\pi}{2};\frac{7\pi}{6}\big[\cup\big]\frac{3\pi}{2};\frac{11\pi}{6}\big[$     C. $\big]0;\frac{\pi}{2}\big[\cup]\frac{7\pi}{6};\frac{3\pi}{2}\big[$     D. $]\pi;2\pi[$     E. шийдгүй    
$\sin x< \dfrac{4}{5}$ тэнцэтгэл бишийн $0\le x< 2\pi$ байх шийд аль нь вэ?

A. $0\le x\le\arccos\dfrac45$     B. $0\le x\le\arccos\dfrac35$     C. $0\le x<\arccos\dfrac35\cup \pi-\arccos \dfrac35\le x< 2\pi$     D. $0\le x<\arcsin\dfrac35\cup \pi-\arcsin \dfrac35\le x< 2\pi$     E. Шийдгүй    
$2\sin^2x>\sin 2x$ тэнцэтгэл бишийн $x\in \left[\dfrac{\pi}{2};\dfrac{3\pi}{2}\right]$ байх шийдүүдийг ол.

A. $\Big[\dfrac{\pi}{2};\pi\Big[\cup\Big]\dfrac{5\pi}{4};\dfrac{3\pi}{2}\Big]$     B. $\Big]\dfrac{\pi}{4};\pi\Big[\cup\Big]\dfrac{5\pi}{4};2\pi\Big[$     C. $\Big]\dfrac{\pi}{4};\pi\Big[\cup\Big]\dfrac{5\pi}{4};\dfrac{3\pi}2\Big]$     D. $\Big]\dfrac{\pi}{2};\pi\Big[\cup\Big]\dfrac{5\pi}{4};2\pi\Big[$     E. Шийдгүй    
$2\sin x>1$ тэнцэтгэл бишийг $0\le x< 2\pi$ мужид бод.

A. $\dfrac{\pi}{2}< x<\dfrac{3\pi}{4}$     B. $x=\dfrac{5\pi}{6}$     C. шийдгүй     D. $\dfrac{\pi}{6}< x<\dfrac{5\pi}{6}$     E. $0< x<\dfrac{\pi}{6}$    
$y=\sin(1-3x)$ функцийн тэмдэг эерэг байх завсрыг ол.

A. $\left]\frac13-\frac{2n}{3}\pi;\frac 13-\frac{n\pi}{3}\right[;$     B. $\left]\frac13+\frac{2n-1}{3}\pi;\frac 13+\frac{2n}{3}\pi\right[;$     C. $\left]\frac13-\frac{2n+1}{3}\pi;\frac 13-\frac23n\pi\right[;$     D. $\left]\frac13+\frac{2n-3}{3}\pi;\frac 13+\frac{2n-1}{3}\pi\right[; (n\in Z)$    
$y=\ctg\left(2x-\dfrac{\pi}{4}\right)$ функцийн тэмдэг сөрөг байх завсрыг ол.

A. $\left]\frac{\pi}{8}+\frac{\pi n}{2};\frac{3\pi}{8}+\frac{\pi n}{2}\right[;$     B. $\left]\frac{\pi}{4}+\pi n;\frac{3\pi}{4}+\pi n\right[;$     C. $\left]\frac{\pi}{4}+2\pi n;\frac{3\pi}{4}+2\pi n\right[;$     D. $\left]\frac{3\pi}{8}+\frac{\pi n}{2};\frac{5\pi}{8}+\frac{\pi n}{2}\right[; (n\in Z)$    
$4\cos^3x-3\cos x< \sin^23x-\cos^23x$ тэнцэтгэл биш аль тэнцэтгэл биштэй эквивалент вэ?

A. $-1<\sin 3x<\frac 12;$     B. $-1<\cos 3x<\frac 12;$     C. $-\frac 12<\sin 3x<1;$     D. $-\frac 12<\cos 3x<1;$    
$\sin (|x|-|x-1|)>\dfrac 12$ тэнцэтгэл бишийг бод.

A. $\left]\frac{\pi}{12}-\frac 16;\infty\right[;$     B. $\left]\frac{\pi}{12}+\frac 16;\infty\right[;$     C. $\left]\frac{\pi}{12}+\frac 12;\infty\right[;$     D. $\left]\frac{\pi}{12}-\frac 12;\infty\right[;$    
$\cos (|x-3|-|x-2|)< \dfrac{\sqrt{3}}{2}$ тэнцэтгэл бишийг бод.

A. $\left]-\infty;2\right]\cup \left]2,5+\frac{\pi}{12};\infty\right[;$     B. $\left]-\infty;2,5-\frac{\pi}{10}\right[\cup \left]2,5+\frac{\pi}{10};\infty\right[;$     C. $\left]-\infty;2\right]\cup \left]2,5+\frac{\pi}{18};\infty\right[;$     D. $\left]-\infty;2,5-\frac{\pi}{16}\right[\cup\left]2,5+\frac{\pi}{16};\infty\right[;$    
$|\sin x|< \cos x$ тэнцэтгэл бишийг бод.

A. $\left]-\frac{\pi}{4}+2\pi k;\frac{\pi}{4}+2\pi k\right[;$     B. $\left]\frac{\pi}{4}+2\pi k;\frac{3\pi}{4}+2\pi k\right[;$     C. $\left]-\frac{3\pi}{4}+2\pi k;\frac{3\pi}{4}+2\pi k\right[;$     D. $\left]\frac{\pi}{4}+\pi k;\frac{3\pi}{4}+2\pi k\right[; (k\in Z)$    
$\sin x>|\cos x|$ тэнцэтгэл бишийг бод.

A. $\left]-\frac{\pi}{4}+2\pi k;\frac{\pi}{4}+2\pi k\right[$     B. $\left]\frac{\pi}{4}+2\pi k;\frac{3\pi}{4}+2\pi k\right[$     C. $\left]-\frac{3\pi}{4}+2\pi k;\frac{3\pi}{4}+2\pi k\right[$     D. $\left]\frac{\pi}{4}+\pi k;\frac{\pi}{2}+\pi k\right[$     E. $\left]\frac{\pi}{4}+\pi k;\frac{3\pi}{4}+\pi k\right[$    
$\tg \dfrac{\pi}{4}\cdot \sin x+\tg\dfrac{\pi}{3}\cdot \cos x\geq 1$ тэнцэтгэл бишийг $\{-3,-2,-1,0,1,2,3\}$ тоонуудын хэд нь хангах вэ?

A. $6;$     B. $4;$     C. $2;$     D. $0;$    
$\dfrac{2\sin\frac{\pi}{3}+\tg x}{1-2\cos\frac{\pi}{6}\cdot \tg x}\geq 1$ тэнцэтгэл бишийг $\{-3,-2,-1,0,1,2,3\}$ тоонуудын хэд нь хангах вэ?

A. $1;$     B. $3;$     C. $5;$     D. $7;$    
$\left|\dfrac 12-\tg|x-1|\right|< \dfrac 12$ тэнцэтгэл бишийн шийд үеийг нь орхиход $x\in ]a,b[\cup]c,d[$ хэлбэрээр бичигдэнэ. Тэгвэл $a+b+c+d=?$

A. $4;$     B. $2;$     C. $1;$     D. $0;$    
$\left|\dfrac{\sqrt{3}}2-\ctg|x|\right|< \dfrac{\sqrt{3}}2$ тэнцэтгэл бишийн шийд үеийг нь орхиход $x\in ]a,b[\cup]c,d[$ гэж олдсон бол $a+b+c+d=?$

A. $-\frac{\pi}{3};$     B. $-\frac{\pi}2;$     C. $\frac{\pi}{3};$     D. $0;$    
$\sin\left(\dfrac{\pi}{2}-x\right)\leq \sin x$ тэнцэтгэл бишийг бод.

A. $-\frac{\pi}{4}+2\pi k\leq x\leq \frac{\pi}{4}+2\pi k$     B. $\frac{\pi}{4}+2\pi k\leq x\leq \frac{\pi}{2}+2\pi k$     C. $\frac{\pi}{4}+2\pi k\leq x\leq \frac{5\pi}{4}+2\pi k$     D. $-\frac{\pi}{4}+2\pi k\leq x\leq\frac{3\pi}{4}+2\pi k$     E. бодит шийдгүй    
$\cos\left(\dfrac{\pi}{3}-x\right)\leq \cos x$ тэнцэтгэл бишийг бод.

A. $-\frac{\pi}{6}+2\pi k\leq x\leq \frac{\pi}{6}+2\pi k;$     B. $-\frac{5\pi}{6}+2\pi k\leq x\leq \frac{\pi}{6}+2\pi k;$     C. $-\frac{7\pi}{6}+2\pi k\leq x\leq \frac{5\pi}{6}+2\pi k;$     D. $\frac{\pi}{6}+2\pi k\leq x\leq \frac{5\pi}{6}+2\pi k; (k\in Z)$    
$2\cos^2x+5\cos x+2\geq 0$ тэнцэтгэл бишийг бод.

A. $-\dfrac{\pi}{3}+2\pi k\leq x\leq \dfrac{\pi}{3}+2\pi k;$     B. $-\dfrac{\pi}{6}+\pi k\leq x\leq \dfrac{\pi}{3}+\pi k;$     C. $-\dfrac{2\pi}{3}+2\pi k\leq x\leq \dfrac{2\pi}{3}+2\pi k;$     D. $-\dfrac{5\pi}{6}+\pi k\leq x\leq\dfrac{\pi}{6}+\pi k$     E. $-\dfrac{5\pi}{12}+\pi k\leq x\leq\dfrac{\pi}{12}+\pi k$    
$\dfrac{2\sin^2x+\sin x-1}{\sin x-1}> 0$ тэнцэтгэл бишийг бод.

A. $-\dfrac{7\pi}{6}+2\pi n< x<\dfrac{\pi}{6}+2\pi n$     B. $-\dfrac{\pi}{6}+2\pi n< x< \dfrac{5\pi}{6}+2\pi n$     C. $-\dfrac{7\pi}{6}+2\pi n< x< \dfrac{5\pi}{6}+2\pi n$     D. $-\dfrac{5\pi}{6}+2\pi n< x<\dfrac{\pi}{6}+2\pi n$     E. $-\dfrac{7\pi}{6}+2\pi n< x<\dfrac{\pi}{6}+2\pi n$, $x\neq\dfrac{3\pi}{2}+2\pi n$    
$\tg^3t+2\tg^2t-\tg t-2\leq 0$ тэнцэтгэл бишийг бод.

A. $-\dfrac{3\pi}{2}+\pi n< t\leq \arctg2+\pi n$     B. $-\dfrac{\pi}{2}+\pi n< t\leq -\arctg 2+\pi n, -\dfrac{\pi}{4}+\pi n\leq t\leq \dfrac{\pi}{4}+\pi n$     C. $-\dfrac{\pi}{4}+\pi n\leq t\leq \dfrac{\pi}{2}+\dfrac{\pi n}{2}$     D. $-\dfrac{3\pi}{4}+\pi n< t\leq \arctg2+\pi n,-\dfrac{\pi}{2}+\pi n< t< \dfrac{\pi}{2}+\pi n$     E. $-\dfrac{3\pi}{4}+\pi n< t\leq \arctg2+\pi n,-\dfrac{\pi}{3}+\pi n< t< \dfrac{\pi}{3}+\pi n$    
$\dfrac1{\sqrt2}<\sin x<\dfrac{3}{2\sqrt3}$ тэнцэтгэл бишийн шийдүүдийн хувьд $\dfrac\pi2< x< \pi$ завсарт харъяалагдах шийд аль нь вэ?

A. $\dfrac{5\pi}{12}$     B. $\dfrac{11\pi}{24}$     C. $\dfrac{\pi}{5}$     D. $\dfrac{17\pi}{24}$     E. $\dfrac{7\pi}{24}$    
$\sin^2x<\dfrac{3}{4}$ тэнцэтгэл бишийн шийдийн олонлог аль нь вэ?

A. $\left]-\dfrac{\pi}{6}+\pi k;\dfrac{\pi}{6}+\pi k\right[$     B. $\left]-\dfrac{\pi}{3}+\pi k;\dfrac{\pi}{3}+\pi k\right[$     C. $\left]\dfrac{\pi}{2}+\pi k;\pi+\pi k\right[$     D. $\left]\dfrac{\pi}{2}+\pi k;\pi k+\dfrac{7\pi}{6}\right[$     E. $\left]\dfrac{\pi}{6}+\pi k;\pi k+\dfrac{7\pi}{6}\right[$    
$\sin(\cos x)<0$ тэнцэтгэл бишийн шийдийг ол.

A. $2\pi k< x<2\pi(k+1)$     B. $\dfrac{\pi}{2}+2\pi k< x<\dfrac{3\pi}{2}+2\pi k$     C. Шийдгүй     D. $\dfrac{\pi}{3}+2\pi k< x$     E. $\dfrac{3\pi}{2}+2\pi k< x<\pi(k+2)$    
$(1-\sin x)(x-2)\ge 0$ тэнцэтгэл бишийн хамгийн бага эерэг шийдийг ол.

A. $\dfrac{\pi}{2}$     B. $2$     C. $1$     D. $\dfrac{\pi}{3}$     E. $\dfrac{2\pi}{3}$    
$\sqrt{\sin x}>\sqrt{-\cos x}$ тэнцэтгэл бишийн шийд аль нь вэ?

A. $\left[\frac{\pi}{2}+2\pi k;\frac{3\pi}{4}+2\pi k\right[$     B. $\left]\frac{3\pi}{4}+2\pi k;\pi+2\pi k\right[$     C. $\left]-\frac{3\pi}{4}+2\pi k;-\frac{\pi}{2}+2\pi k\right[$     D. $\left[-\frac{\pi}{2}+2\pi k;-\frac{\pi}{4}+2\pi k\right[$     E. $\varnothing$    
$\sqrt{-\sin x}>\sqrt{\cos x}$ тэнцэтгэл бишийн шийд аль нь вэ?

A. $\left[\frac{\pi}{2}+2\pi k;\frac{3\pi}{4}+2\pi k\right[$     B. $\left]\frac{3\pi}{4}+2\pi k;\pi+2\pi k\right[$     C. $\left[-\frac{\pi}{2}+2\pi k;-\frac{\pi}{4}+2\pi k\right[$     D. $\left]-\frac{\pi}{4}+2\pi k;2\pi k\right]$     E. $\varnothing$    
$\sin\Big(x-\dfrac{\pi}{6}\Big)\cos\Big(x-\dfrac{\pi}{6}\Big)<\dfrac{\sqrt2}{4}$ тэнцэтгэл бишийг бод.

A. $\Big]k\pi-\dfrac{5\pi}{24};\dfrac{11\pi}{24}+k\pi\Big[$     B. $\Big]\dfrac{k\pi}{2}-\dfrac{7\pi}{24};\dfrac{11\pi}{24}+\dfrac{k\pi}{2}\Big[$     C. $\Big]k\pi-\dfrac{11\pi}{4};\dfrac{7\pi}{4}+k\pi\Big[$     D. $\Big]k\pi-\dfrac{11\pi}{24};\dfrac{7\pi}{24}+k\pi\Big[$     E. $\Big]\dfrac{k\pi}{2}-\dfrac{11\pi}{24};\dfrac{7\pi}{24}+\dfrac{k\pi}{2}\Big[$    
$\sin\big(x+\frac{\pi}{6}\big)\cos\big(x+\frac{\pi}{6}\big)<\frac{\sqrt2}{4}$ тэнцэтгэл бишийг бод.

A. $\big]k\pi-\frac{7\pi}{24};\frac{11\pi}{24}+k\pi\big[$     B. $\big]k\pi-\frac{7\pi}{48};\frac{11\pi}{48}+k\pi\big[$     C. $\big]k\pi-\frac{11\pi}{48};\frac{7\pi}{48}+k\pi\big[$     D. $\big]k\pi-\frac{19\pi}{24};-\frac{\pi}{24}+k\pi\big[$     E. $\big]k\pi-\frac{11\pi}{24};\frac{7\pi}{24}+k\pi\big[$    
$\sin\big(2x-\frac{\pi}{6}\big)\cos\big(2x-\frac{\pi}{6}\big)<\frac{\sqrt2}{4}$ тэнцэтгэл бишийг бод.

A. $\big]\frac{k\pi}{2}-\frac{7\pi}{24};\frac{11\pi}{24}+\frac{k\pi}{2}\big[$     B. $\big]\frac{k\pi}{2}-\frac{7\pi}{48};\frac{11\pi}{48}+\frac{k\pi}{2}\big[$     C. $\big]\frac{k\pi}{2}-\frac{11\pi}{48};\frac{7\pi}{48}+\frac{k\pi}{2}\big[$     D. $\big]\frac{k\pi}{4}-\frac{11\pi}{48};\frac{7\pi}{48}+\frac{k\pi}{4}\big[$     E. $\big]\frac{k\pi}{2}-\frac{11\pi}{24};\frac{7\pi}{24}+\frac{k\pi}{2}\big[$    
$\sin\left(3x-\dfrac{\pi}{6}\right)\cos\left(3x-\dfrac{\pi}{6}\right)<\dfrac{\sqrt2}{4}$ тэнцэтгэл бишийг бод.

A. $\left]\dfrac{2k\pi}{3}-\dfrac{7\pi}{24};\dfrac{11\pi}{24}+\dfrac{2k\pi}{3}\right[$     B. $\left]\dfrac{2k\pi}{3}-\dfrac{7\pi}{48};\dfrac{11\pi}{48}+\dfrac{2k\pi}{3}\right[$     C. $\left]\dfrac{2k\pi}{3}-\dfrac{11\pi}{72};\dfrac{7\pi}{72}+\dfrac{2k\pi}{3}\right[$     D. $\left]\dfrac{k\pi}{3}-\dfrac{11\pi}{72};\dfrac{7\pi}{72}+\dfrac{2k\pi}{3}\right[$     E. $\left]\dfrac{k\pi}{2}-\dfrac{11\pi}{72};\dfrac{7\pi}{72}+\dfrac{k\pi}{2}\right[$    
$\sin\big(2x-\frac{\pi}{6}\big)\cos\big(2x-\frac{\pi}{6}\big)<\frac{\sqrt2}{4}$ тэнцэтгэл бишийг бод.

A. $\big]\frac{k\pi}{2}-\frac{7\pi}{24};\frac{11\pi}{24}+\frac{k\pi}{2}\big[$     B. $\big]\frac{k\pi}{2}-\frac{7\pi}{48};\frac{11\pi}{48}+\frac{k\pi}{2}\big[$     C. $\big]\frac{k\pi}{2}-\frac{11\pi}{48};\frac{7\pi}{48}+\frac{k\pi}{2}\big[$     D. $\big]\frac{k\pi}{4}-\frac{11\pi}{48};\frac{7\pi}{48}+\frac{k\pi}{4}\big[$     E. $\big]\frac{k\pi}{2}-\frac{11\pi}{24};\frac{7\pi}{24}+\frac{k\pi}{2}\big[$    
$\sin^4\dfrac{x}{2}+\cos^4\dfrac{x}{2}< \dfrac58$ тэнцэтгэл бишийн шийд $\pi k+\dfrac{\pi}{\fbox{a}}< x< \dfrac{\fbox{b}\pi}{\fbox{c}}+\pi k, k\in\mathbb Z$ байна. Энэ шийдэд агуулагдах хамгийн бага эерэг бүхэл тоо $x=\fbox{d}$, хамгийн их сөрөг бүхэл тоо $x=-\fbox{e}$ болно.
$\sin^4x+\cos^4x< \dfrac34$ тэнцэтгэл бишийн шийд $$\dfrac{\pi k}{2}+\dfrac{\pi}{\fbox{a}}< x< \dfrac{\fbox{b}\pi}{\fbox{c}}+\dfrac{\pi k}{2},~k\in\mathbb Z$$ байна. Энэ шийдэд агуулагдахгүй хамгийн бага эерэг бүхэл тоо $x=\fbox{d}$, хамгийн их сөрөг бүхэл тоо $x=-\fbox{e}$ болно.
$\displaystyle 2\cos^2 2x\le\frac{3}{2}$ тэнцэтгэл бишийн $\displaystyle 0^{\circ}\le x\le90^\circ$ нөхцөл хангах шийд нь $\displaystyle \fbox{ab}^\circ\le x\le\fbox{cd}^\circ$ байна.
$3\cos 2x-4\sin2x>\dfrac{5}{\sqrt{2}}$ $(\alpha=\arccos \dfrac{\fbox{a}}{5} $ туслах өнцөг оруулбал) $\Leftrightarrow \cos(2x+\alpha)>\dfrac{1}{\sqrt{\fbox{b}}}\Leftrightarrow x\in \left]-\dfrac{\pi}{\fbox{c}}-\dfrac{\alpha}{2}+\pi k;\dfrac{\pi}{\fbox{d}}-\dfrac{\alpha}{2}+\pi k\right[ (k\in \mathbb Z).$
$\cos 3x+\sin2x\cos x+\sin x\cos 2x>-\sqrt{\dfrac{3}{2}}$ $(\alpha=\dfrac{\pi}{\fbox{a}}$ туслах өнцөг оруулбал) $\Leftrightarrow \sin(3x+\alpha)>-\dfrac{\sqrt{\fbox{c}}}{\fbox{b}} \Leftrightarrow x\in \left]-\dfrac{\fbox{d}}{36}\pi+\dfrac23\pi k;\dfrac{13}{36}\pi+\dfrac23\pi k\right[ (k\in \mathbb Z).$
$\cos(2\sin x)< 0$ тэнцэтгэл бишийн шийд нь $x\in \left]\arcsin \dfrac{\pi}{\fbox{a}}+k\pi;-\arcsin \dfrac{\pi}{\fbox{b}}+(k+\fbox{c})\pi\right[$ байна.
$\cos(1+\sin x)< 0$ тэнцэтгэл бишийн шийд нь $x\in \left]\arcsin \dfrac{\pi}{\fbox{a}}-\fbox{b}+2\pi k;-\arcsin \left(\dfrac{\pi}{\fbox{c}}-\fbox{d}\right)+(2k+\fbox{e})\pi\right[$ байна.
$\cos 2x\geq \sin x \Leftrightarrow x\in \Bigl[-\dfrac{\fbox{b}}{\fbox{a}}\pi+2\pi k;\dfrac{1}{\fbox{c}}\pi+2\pi k\Bigr] (k\in \mathbb Z).$
$3\cos x-2\geq \cos 2x \Leftrightarrow x\in \Bigl[-\dfrac{\pi}{\fbox{a}}+2\pi k;\dfrac{\pi}{\fbox{b}}+2\pi k\Bigr] (k\in \mathbb Z).$
$\left\{ % \begin{array}{l} \tg x>1 \\ x\in [0^{\circ};360^{\circ}[ \\ \end{array} % \right. \Leftrightarrow x\in \Bigl]\fbox{ab}^{\circ};90^{\circ}\Bigr[\cup \Bigl]\fbox{cde}^{\circ};\fbox{fgh}^{\circ}\Bigr[$.
$\left\{ % \begin{array}{l} \tg x< -\sqrt{3} \\ x\in [0^{\circ};360^{\circ}[ \\ \end{array} % \right. \Leftrightarrow x\in \Bigl]\fbox{ab}^{\circ};\fbox{cde}^{\circ}\Bigr[\cup \Bigl]270^{\circ};\fbox{fgh}^{\circ}\Bigr[$.
$\left\{ % \begin{array}{l} 4\cos^2 3x-1>0 \\ x\in ]0^{\circ};100^{\circ}] \\ \end{array} % \right. \Leftrightarrow x\in \Bigl[\fbox{a}^{\circ};\fbox{bc}^{\circ}\Bigr[\cup \Bigl]\fbox{de}^{\circ};\fbox{fg}^{\circ}\Bigr[$.
$\left\{ % \begin{array}{l} \cos 3x+\sin 3x>1 \\ x\in ]100^{\circ};200^{\circ}] \\ \end{array} % \right. \Leftrightarrow x\in \Bigl]\fbox{abc}^{\circ};\fbox{def}^{\circ}\Bigr[$.
$\left\{ % \begin{array}{l} \left(\cos\alpha-\dfrac 12\right)\left(\sin\alpha-\dfrac{\sqrt{3}}{2}\right)\geq 0 \\ 0^{\circ}\leq \alpha\leq 180^{\circ} \\ \end{array} % \right.$ системийн шийд нь $\alpha\in \left\{\fbox{ab}^{\circ}\right\}\bigcup \left[ \fbox{cde}^{\circ};\fbox{fgh}^{\circ}\right]$ байна.
$\bigg\{ \begin{array}{l} (\tg x-\sqrt{3})(\ctg x-\sqrt{3})\geq 0 \\ x\in [0;\pi] \end{array}$ системийн шийд нь $x\in \bigg[\dfrac{\pi}{\fbox{a}};\dfrac{\pi}{\fbox{b}}\bigg]\bigcup \bigg] \dfrac{\pi}{\fbox{c}};\pi\bigg]$ байна.
$(\sin\alpha)x^2-\sqrt{2}x-1\geq 0$ тэнцэтгэл бишийг $x$ хувьсагчийн хувьд бодоход шийдгүй байлгах $\alpha$ параметрийн утгууд $\alpha\in \left]-\dfrac{\fbox{b}}{\fbox{a}}\pi +2\pi k;-\dfrac{\pi}{\fbox{c}}+2\pi k\right[$ байна.
$(\cos\alpha)x^2-\sqrt[4]{3}x+0,5< 0$ тэнцэтгэл бишийг $x$ хувьсагчийн хувьд бодоход шийдгүй байлгах $\alpha$ параметрийн утгууд $\alpha\in \left]-\dfrac{\pi}{\fbox{a}}+2\pi k;-\dfrac{\pi}{\fbox{b}}+2\pi k\right]\bigcup \left[\dfrac{\pi}{\fbox{c}}+2\pi k;\dfrac{\pi}{\fbox{d}}+2\pi k\right[$ байна.
$\left\{ % \begin{array}{l} |\ctg 3x|< \sqrt{3} \\ x\in [120^{\circ};180^{\circ}] \\ \end{array} % \right. \Leftrightarrow \left\{ % \begin{array}{l} \fbox{a}\sqrt{3}< \ctg 3x< \sqrt{\fbox{b}} \\ x\in [120^{\circ};180^{\circ}] \\ \end{array} % \right.\Leftrightarrow x\in \Bigl[\fbox{cde}^{\circ};\fbox{fgh}^{\circ}\Bigr[$.
$\left\{ % \begin{array}{l} |4\sin 2x+3|>1 \\ x\in [90^{\circ};270^{\circ}] \\ \end{array} % \right. \Leftrightarrow \left\{ % \begin{array}{l} \sin 2x>\fbox{ab,c} \\ x\in [90^{\circ};270^{\circ}] \\ \end{array} % \right. %\Leftrightarrow \\ \Leftrightarrow x\in \Bigl[\fbox{de}^{\circ};\fbox{fgh}^{\circ}\Bigr[$.
$\left\{ % \begin{array}{l} |\tg (2x+20^{\circ})|< \sqrt{3} \\ x\in [140^{\circ};270^{\circ}] \\ \end{array} % \right. \Leftrightarrow\\ \Leftrightarrow \left\{ % \begin{array}{l} \fbox{abc}^{\circ}< 2x+20^{\circ}< \fbox{def}^{\circ} \\ 2x+20^{\circ}\in [140^{\circ};270^{\circ}] \\ \end{array} % \right. \Leftrightarrow \fbox{gh}^{\circ}< x< \fbox{klm}^{\circ}.$
$\left\{ % \begin{array}{l} |2\cos 4x-1|>2 \\ 4x\in [90^{\circ};270^{\circ}] \\ \end{array} % \right. \Leftrightarrow \left\{ % \begin{array}{l} \fbox{abc}^{\circ}< 4x< \fbox{def}^{\circ} \\ 4x\in [90^{\circ};270^{\circ}] \\ \end{array} % \right. \Leftrightarrow \fbox{gh}^{\circ}< x< \fbox{kl}^{\circ}.$
$\arccos x\leq \arccos \dfrac 14$ тэнцэтгэл бишийн шийд нь $x\in \left[\dfrac{\fbox{b}}{\fbox{a}};\fbox{c}\right]$ болно.
$\arccos x>\dfrac{\pi}{6}$ тэнцэтгэл бишийн шийд нь $x\in\left[\fbox{ab};\dfrac{\sqrt{3}}{\fbox{c}}\right]$ болно.
$\sin^4x+\cos^4x< \dfrac34$ тэнцэтгэл бишийн шийд $\dfrac{\pi k}{2}+\dfrac{\pi}{\fbox{a}}< x< \dfrac{\fbox{b}\pi}{\fbox{c}}+\dfrac{\pi k}{2}, k\in\mathbb Z$ байна. Энэ шийдэд агуулагдахгүй хамгийн бага эерэг бүхэл тоо $x=\fbox{d}$, хамгийн их сөрөг бүхэл тоо $x=-\fbox{e}$ болно.
$1$, $\left|\sin\dfrac{a}{4}\right|$, $\cos\dfrac{5\pi}{3}$ тоонууд гурвалжны талууд болдог байх $a$-г олъё. Тэгвэл $0 < \cos\dfrac{5\pi}{3} < 1$, $\left|\sin\dfrac{a}{4}\right|\le1$ тул $\cos\dfrac{5\pi}{3}+\left|\sin\dfrac{a}{4}\right|>\fbox{a}$ нөхцөл биелэхэд л хангалттай. Иймд $\dfrac{\fbox{b}}{2}<\sin\dfrac{a}{2}$, эсвэл $\sin\dfrac{a}{2}<-\dfrac{\fbox{c}}{2}$ байна. Эндээс $$\dfrac{\fbox{d}\pi}{\fbox{e}}+\fbox{f}\pi k < a <\dfrac{1\fbox{g}\pi}{\fbox{e}}+\fbox{f}\pi k$$

Тэнцэтгэл биш батлах

Хэрэв $a+b+c=1$ бол $\sqrt{2a+1}+\sqrt{2b+1}+\sqrt{2c+1}< 15$ тэнцэтгэл бишийг батал.
$x+2\cdot\sqrt{x-1}+\sqrt[{3}]{{x^{4}-2x^{2}+1}}>1+2\sqrt[{3}]{{x^{2}-1}}$ гэсэн тэнцэтгэл бишийг хангах шийд бүрийн хувьд $\sqrt{x-1}+\sqrt[{3}]{{x^{2}-1}}>2$ болохыг батал.
$\log_{3}\left({x^{2}-1}\right)\log_{3}\left({9\left({x^{2}-1}\right)}\right)>2\log_{2}\left({x-1}\right)-\log_{2}^{2}\left({x-1}\right)$ тэнцэтгэл бишийн шийд бүрийн хувьд $\log_{2}\left({x-1}\right)+\log_{3}\left({x^{2}-1}\right)>2$ байна гэж батал.
$x>0$ байх $x$ бүрийн хувьд $x^{2}+\pi x+\dfrac{15}{2}\pi\cdot\sin x>0$ гэж батал.
$y=ax^2+x+1$ $(a\ne 0)$ бол
  1. Энэ параболыг $a$-ийн утгаас хамаарахгүйгээр ямагт шүргэх шулууныг ол.
  2. $a>0$ үед энэ параболын оройн цэгийн хөдлөх муруйг зур.
$\left(\mathop{\int\limits_{0}^{1}}(ax+b)\,\mathrm{d}x\right)^2\leq \mathop{\int\limits_{0}^{1}}(ax+b)^2\,\mathrm{d}x$ тэнцэтгэл бишийг батал.
$a$, $b$, $c$ эерэг бодит тоонууд бол
  1. $a+b+c\geq 3\sqrt[3]{abc}$ болохыг батал.
  2. $a+b+c=3$ бол $2^a+2^b+2^c$-ийн хамгийн бага утгыг ол.
$a_i\in R$, $b_i>0$ бол $$\left(\dfrac{a_1^2}{b_1}+\dots+\dfrac{a_n^2}{b_n}\right)\ge\dfrac{(a_1+\dots+a_n)^2}{(b_1+\dots+b_n)}$$ тэнцэтгэл бишийг батал.

Тэнцэтгэл биш бодох графикийн арга

  1. $2x^2-9x+4<0$
  2. $x^2-6x+5\le0$
  3. $2x^2-3x-2<0$
  4. $x^2-5x+4<0$
  5. $2x^2-5x+2\le0$
  6. $x^2-2x-3<0$
  7. $2x^2-5x-3\le0$
  8. $x^2-4x+3<0$
  9. $x^2-7x+3\le0$
  10. $x^2-x-2<0$


Тэнцэтгэл бишийн шийдийн муж

$\left\{\begin{array}{c} y+8\ge x^{2}+2x\\ 12+5x\le-2y \end{array}\right.$ системийг хангах $\left({x,y}\right)$ цэгүүдийн олонлогийг координатын хавтгайд дүрсэл.
$\left\{\begin{array}{c} x+y-1>0\\ y+{\left|{4x}\right|}\le 4 \end{array}\right.$ системийг хангах цэгүүдийн олонлогийг $Oxy$ координатын хавтгайд дүрсэл.
$y-4\le\sqrt{4y-x^{2}}$ системийг хангах цэгүүдийн олонлогийг $Oxy$ координатын хавтгайд дүрсэл.
$\log_{{\left|{x-1}\right|}-2{\left|{x}\right|}+4} y>\log_{{\left|{x-1}\right|}-2{\left|{x}\right|}+4}\left({4-x}\right)$ тэнцэтгэл бишийг хангах цэгүүдийн олонлогийг координатын хавтгайд дүрсэл.
$\log_{{\left|{x+1}\right|}-2{\left|{x-1}\right|} + 6}y>\log{}_{{\left|{x+1}\right|}-2{\left|{x-1}\right|}+6}\left({x+4}\right).$ тэнцэтгэл бишийг хангах цэгүүдийн олонлогийг координатын хавтгай дээр зурж үзүүл.
$x\ge{\left|{x^{3}+xy^{2}}\right|}$ системийг хангах цэгүүдийн олонлогийг $Oxy$ координатын хавтгай дээр дүрсэл.
$y\ge{\left|{y^{3}+x^{2}y}\right|}$ системийг хангах цэгүүдийн олонлогийг $Oxy$ координатын хавтгай дээр дүрсэл.
$a$, $b$ тоонуудын ихийг нь $\max\{a,b\}$ гэж тэмдэглэе. ($a=b$ үед $\max\{a,b\}=a$ байна). $1\leq \max\{4x+4y-3, x^2+y^2\}\leq 5$ тэнцэтгэл бишийг хангах $(x,y)$ цэгийн дүрслэх мужийг зур.
  1. $|\log_2 x|+\log_4 y\leq \log_2 \sqrt{3}$ тэнцэтгэл бишийг хангах $(x,y)$-ын геометр байрыг дүрсэл.
  2. $(x,y)$ цэг нь $(1)$ мужид байх үед $\log_2 y-\log_2 x$-ийн хамгийн их утгыг ол.
$x$, $y$ нь $3x-5y\geq-16$, $3x-y\leq 4$, $x+y\geq 0$ нөхцлүүдийг хангах үед $2x+5y$-ийн хамгийн их, хамгийн бага утгыг ол.
  1. Координатууд нь дараах нөхцлийг хангах цэгүүдийн олонлогийг дүрсэл.
    1. $y-2x< 4 \boldsymbol{\cdots}(1)$
    2. $y\ge x^2-3x+2 \boldsymbol{\cdots}(2)$
    3. $(x-2)^2+y^2\ge4 \boldsymbol{\cdots} (3)$
  2. Дараах нөхцлүүдийг зэрэг хангах цэгүүдийн олонлогийг дүрсэл. $$0\le x\le 1 \boldsymbol{\cdots}(4) x^2+y^2\leq4 \boldsymbol{\cdots}(5)$$
Дараах нөхцлийг хангах $(x,y)$ координаттай цэгүүдийг координатын хавтгайд дүрсэл.
  1. $(x+y-2)(y-x^2)< 0$
  2. $|x+y|\leq 2$
$xy=a^6,\log_x\cdot \log_y=(\log_b)^2$-ыг хангах $x,y$ оршин байх эерэг $(a,b)$-ын утгын мужийг дүрсэл.
Дараах тэнцэтгэл бишүүдийн шийдийн мужаар үүсэх дүрсийн талбайг ол.

  1. $y\geq x^2, y\leq 3x+4, y\leq -3x+10$
  2. $x+2y\leq 8, y\leq-x^2+9, y\geq 0$
$\left\{\begin{array}{c}y\ge\sin x\\x^2+y^2\le 2\pi x\end{array}\right.$ систем тэнцэтгэл бишийн шийдийг илэрхийлэх мужийн талбайг ол.

A. $\frac{\pi^2}{2}$     B. $\frac{2\pi^2}{3}$     C. $\frac{\pi^3}{3}$     D. $\frac{3\pi^3}{2}$     E. $\frac{\pi^3}{2}$    
$\left\{\begin{array}{c}y< -x+2\\y+22>x\end{array}\right.$ системийн шийд аль, аль мужид байгаа вэ?

A. 1 ба 2     B. 2 ба 3     C. 3 ба 4     D. бүх мужид     E. 1, 2, 3    
$y\ge|y^3+x^2y|$ тэнцэтгэл бишийн шийдийн мужийн талбайг ол.

A. $\pi$     B. $\pi/2$     C. $2\pi$     D. $\pi/3$     E. $0$    
$0\le x\le 2$, $0\le y\le4$ тэнцэтгэл бишийн шийдийн муж болох дүрсийн талбай аль нь вэ?

A. 1     B. 2     C. 4     D. 8     E. 16    
$\left\{\begin{array}{c}x^2+y^2\le9\\ x\le y\end{array}\right.$ тэнцэтгэл бишийн шийдийн мужийн талбай аль нь вэ?

A. $4.5\pi$     B. $5\pi$     C. $3.5\pi$     D. $9\pi$     E. $2\pi$    
$\left\{\begin{array}{c} y-x \leq 2 \\ x^2+y^2 \leq 4 \end{array}\right.$ мужаар хүрээлэгдэх дүрсийн талбайг ол.

A. $\pi-2$     B. $3\pi+2$     C. $2\pi+2$     D. $4\pi-2$     E. $3\pi-2$    
$\left\{\begin{array}{l}x^2+y^2\ge1 \\ x^2+y^2\le16 \end{array}\right.$ тэнцэтгэл бишийн системийг хангах хавтгайн цэгүүдийн олонлог аль вэ?

A. зураг     B. зураг     C. зураг     D. зураг    
$\left\{\begin{array}{l}x^2+y^2-4\le0 \\ y-x^2-3< 0 \end{array}\right.$ тэнцэтгэл бишийн системийг хангах хавтгайн цэгүүдийн олонлог аль вэ?

A. зураг     B. зураг     C. зураг     D. зураг    
$\left\{\begin{array}{l}x^2+y^2\le9 \\ y\ge x^2-3 \\ |x|\le1 \end{array}\right.$ тэнцэтгэл бишийн системийг хангах хавтгайн цэгүүдийн олонлог аль вэ?

A. зураг     B. зураг     C. зураг     D. зураг    
$\left\{\begin{array}{l}\log_2(2y-x^2+1)>\log_2y \\ \sqrt{3y-3x+2}< \sqrt{8+x} \end{array}\right.$ тэнцэтгэл бишийн системийг хангах хавтгайн цэгүүдийн олонлог аль вэ?

A. зураг     B. зураг     C. зураг     D. зураг    
$\left\{\begin{array}{l}|x|+|y|< 4 \\ \log_2(2y-x^2+4)>\log_2(y+1) \end{array}\right.$ тэнцэтгэл бишийн системийг хангах хавтгайн цэгүүдийн олонлог аль вэ?

A. зураг     B. зураг     C. зураг     D. зураг    
$\left\{\begin{array}{l}y\ge x^2 \\ y\le4-x^2 \end{array}\right.$ тэнцэтгэл бишийн системийг хангах хавтгайн цэгүүдийн олонлог аль вэ?

A. зураг     B. зураг     C. зураг     D. зураг    
$\left\{\begin{array}{l}y\ge x^2-4x+3 \\ y< x^2+4x+3 \end{array}\right.$ тэнцэтгэл бишийн системийг хангах хавтгайн цэгүүдийн олонлог аль вэ?

A. зураг     B. зураг     C. зураг     D. зураг    
$\left\{\begin{array}{l}y-|\log_2x|>0 \\ y-2\ge0 \end{array}\right.$ тэнцэтгэл бишийн системийг хангах хавтгайн цэгүүдийн олонлог аль вэ?

A. зураг     B. зураг     C. зураг     D. зураг    
$\left\{\begin{array}{l}y\ge\sqrt{1-x^2} \\ y+|x|\le4 \end{array}\right.$ тэнцэтгэл бишийн системийг хангах хавтгайн цэгүүдийн олонлог аль вэ?

A. зураг     B. зураг     C. зураг     D. зураг    
$\left\{\begin{array}{l}3x+2y+1\ge0 \\ 3x+2y-3\le0 \end{array}\right.$ тэнцэтгэл бишийн системийг хангах хавтгайн цэгүүдийн олонлог аль вэ?

A. зураг     B. зураг     C. зураг     D. зураг    
$\left\{\begin{array}{l}x+y\le1 \\x-y\le1 \end{array}\right.$ тэнцэтгэл бишийн системийг хангах хавтгайн цэгүүдийн олонлог аль вэ?

A. зураг     B. зураг     C. зураг     D. зураг    
Зурагт үзүүлсэн цэгийн олонлог аль тэнцэтгэл бишийн шийд болох вэ?

A. $y>2x+1$     B. $y<2x-1$     C. $2y\ge-4+x$     D. $2y\le x+2$     E. $y< x+1$    
$|x|+|y|\le 1$ нөхцөлийг хангах цэгийн олонлог аль нь вэ?

A. 1)     B. 2)     C. 3)     D. 4)     E. 5)    
$\left\{\begin{array}{l}x^2+y^2\le9 \\ x+y\le0 \end{array}\right.$ тэнцэтгэл бишийн системийг хангах хавтгайн цэгүүдийн олонлог аль вэ?

A. 1)     B. 2)     C. 3)     D. 4)     E. 5)    
$\left\{\begin{array}{c}y\le\sin x\\x^2+y^2\le 2\pi x\end{array}\right.$ систем тэнцэтгэл бишийн шийдийг илэрхийлэх мужийн талбайг ол.

A. $\frac{\pi^2}{2}$     B. $\frac{2\pi^2}{3}$     C. $\frac{\pi^3}{2}$     D. $\frac{3\pi^3}{2}$     E. $\frac{\pi^3}{3}$    
$x>0, y>0$ тоонуудын хувьд $5x+4y=17$ тэнцэтгэл биелдэг бол $0< y< \dfrac{\fbox{ab}}4$ тэнцэтгэл биш биелнэ. Иймд энэ тэгшитгэлийн натурал тоон шийд $y=\fbox{c}, x=\fbox{d}$ байна.
$x>0,y>0$ тоонуудын хувьд $6x+7y=20$ тэнцэтгэл биелдэг бол $0< x< \dfrac{\fbox{ab}}3$ тэнцэтгэл биш биелнэ. Иймд энэ тэгшитгэлийн натурал тоон шийд $x=\fbox{c}, y=\fbox{d}$ байна.
$x, y$ нь $x^2+y^2\leq 10$, $y\geq -2x+5$ нөхцлийг хангах үед $x+y$-ийн хамгийн их утга $\fbox{a}\sqrt{\fbox{b}}$ ба хамгийн бага утга $\fbox{c}$ байна.
$(x-5)^2+(y-4)^2=36$ нь $(\fbox{a};\fbox{b})$ төвтэй $\fbox{c}$ радиустай тойргийн тэгшитгэл ба $A(x;y)$ нь тойргийн цэг үед $3x+4y$ илэрхийллийн хамгийн их утга нь $\fbox{de}$ юм.

Холимог тэнцэтгэл биш

$5x^{2}\cdot 5^{x^{2}}+12>3x^{2}+20 \cdot 5^{x^{2}}$. Бүхэл шийдийг ол.
$4x^{2}+3^{\sqrt{x}+1}+3^{\sqrt{x}}\cdot x< 2x^{2}\cdot 3^{\sqrt{x}}+2x+6$. Хамгийн бага бүхэл шийдийг ол.
$\sqrt{x^{2}-7.5x+14}\cdot\log_{2}|x-3|\le 0$ тэнцэтгэл бишийг бод.
$\sqrt{7+2^{1-x}}\ge 7-\left({{\dfrac{{1}}{{2}}}}\right)^{x-2}$ тэнцэтгэл бишийг бод.
$\sqrt{1+\left({{\dfrac{{1}}{{3}}}}\right)^{-x-1}}\ge 5-9\cdot 3^{x-1}$ тэнцэтгэл бишийг бод.
$\dfrac{{\sqrt{2-x^{2}+2x}+x-2}}{\log_{3}\left({\dfrac{5}{2}-x}\right)+\log_{3}2}\le 0$ тэнцэтгэл бишийг бод.
${\dfrac{{{\left[{\log_{\sqrt{2}}\left({x-3}\right)}\right]}^{2}}}{{x^{2}-4x-5}}}\ge 0$ тэнцэтгэл бишийг бод.
$\sqrt{0.5\cdot\left({15^{x}+9}\right)}\le\sqrt{15^{x}+12}-\sqrt{0.5\cdot\left({15^{x}-9}\right)}$ тэнцэтгэл бишийг бод.
$\left({2-5^{x-2}-5^{2-x}}\right)^{-1}\cdot\left({x^{2}-x-2}\right)\cdot\sqrt{3-x}\ge 0$ тэнцэтгэл бишийг бод.
${\dfrac{{1}}{{x}}}\cdot\log_{5}\left({{\dfrac{{10}}{{3}}}-5^{-x}}\right)>1$ тэнцэтгэл бишийг бод.
$x\cdot\log_{\frac{1}{3}}\left(\dfrac{1}{3}-x\right)>|x|$ тэнцэтгэл бишийг бод.
${\left|{x-4^{1+\sqrt{3-x}}}\right|}\le {\dfrac{{5}}{{3}}}x-4\cdot 4^{\sqrt{3-x}}$ тэнцэтгэл бишийг бод.
${\left|{x-7^{1-\sqrt{6-x}}}\right|}\le {\dfrac{{4}}{{3}}}x-7\cdot 7^{\sqrt{6-x}}$ тэнцэтгэл бишийг бод.
${\dfrac{{1}}{{x}}}\sqrt{10x-8-2x^{2}}-\left({\sqrt{x^{2}-5x+4}+{\dfrac{{1}}{{2}}}}\right)\cdot\log_{5}{\dfrac{{x}}{{16}}}\le 1$ тэнцэтгэл бишийг бод.
$3x+12 \cdot 3^{\sqrt{x}}\ge 4x \cdot 3^{\sqrt{x}}+9$ тэнцэтгэл бишийн бүхэл шийдийн тоог ол.

A. $1$     B. $2$     C. $3$     D. $4$     E. $5$    

Шугаман тэнцэтгэл биш, түүний систем

$5x+7>3x+20$ тэнцэтгэл бишийн хамгийн бага бүхэл шийдийг ол.
8.2
$x-\dfrac{1-x}{6}\le \dfrac{2x+1}{2}-\dfrac34$ тэнцэтгэл бишийн хамгийн их бүхэл шийдийг ол.
8.1
$\dfrac{2x-7}{3}<5$ тэнцэтгэл бишийг хангах $x$-ийн хамгийн их натурал утгыг ол.

A. 5     B. 10     C. 7     D. 11     E. 15    
$2x+y-11=0$, $1< y<7\Rightarrow ? < x < ?$

A. $2< x<5$     B. $-5< x<-2$     C. $-10< x<-4$     D. $1< x<7$     E. $4< x<10$    
8.1
$-2x>-7$ тэнцэтгэл бишийг бод.

A. $x < -\dfrac{7}{2}$     B. $x < \dfrac{7}{2}$     C. $x > -\dfrac{7}{2}$     D. $x > \dfrac{7}{2}$     E. $0 < x <\dfrac{7}{2}$    
$\dfrac{2x-7}{3}<5$ тэнцэтгэл бишийг хангах $x$-ийн хамгийн их натурал утгыг ол.

A. 4     B. 10     C. 9     D. 11     E. 15    
7.1
$500 < 53x < 1000$ тэнцэтгэл бишийг хангах утга аль нь вэ?

A. $9$     B. $21$     C. $13$     D. $8$     E. $19$    
$\dfrac{2x-7}{3}\ge 5$ тэнцэтгэл бишийг хангах $x$-ийн хамгийн бага натурал утгыг ол.

A. 4     B. 10     C. 9     D. 11     E. 15    
$-\dfrac{1}{5}x < -3$ тэнцэтгэл бишийг бод.

A. $15 < x$     B. $x < 15$     C. $-15 < x$     D. $x > \dfrac{3}{5}$     E. $x < \dfrac{5}{3}$    
7.1
$\left\{\begin{array}{c} x < 5\\ x \ge -1 \end{array}\right.$ тэнцэтгэл бишийн шийдийг тоон завсар хэлбэрээр бич.

A. $[-1,5]$     B. $[-1,5[$     C. $]-1,5[$     D. $[1,5[$     E. $]-1,5]$    
7.1
$0<\dfrac{2x-7}{5}\le 3$ тэнцэтгэл бишийг хангах $x$-ийн хамгийн их натурал утгыг ол.

A. 5     B. 10     C. 7     D. 11     E. 15